CLATutor Logical Reasoning 1

87
CLATutor Logical Reasoning 1

Transcript of CLATutor Logical Reasoning 1

CLATutor Logical Reasoning 1

CLATutor Logical Reasoning 2

CLATutor Logical Reasoning 3

INDEX

1. Set I 4 - 6

2. Set II 7 - 9

3. Set III 10 - 12

4. Set IV 13 - 15

5. Set V 16 - 18

6. Set VI 19 - 22

Part II Verbal Reasoning

7. Analogy and Classification,

Number Series and Ranking 23 - 25

8. Coding, Decoding and Inequalities 26 - 30

9. Syllogism 31 - 36

10. Direction & Blood Relationship 37 - 40

11. Sitting Arrangement & Puzzle 41 - 44

12. Decision making, Input and Output 45 - 51

13. Data Sufficiency 52 - 59

Part III Logical Reasoning

14. Assumption and Argument 60 - 64

15. Course of Action, Cause And Effect 65 - 70

16. Inference and Conclusion 71 - 75

17. Critical Reasoning 76 - 80

18. Non-Verbal 81 - 83

CLATutor Logical Reasoning 4

Set I When we have dreams, we have a passion. When we have a passion, we want to do it with perfection. Often, when we have some goals, we want to start it with a blast. We want to become an overnight star. We want to achieve best as soon as possible. But things take time. It gets better with the time, but the key is to start as soon as possible. What happens is that we have an idea, which we think can change our or someone’s life instantly. But we don’t work on it until we have favorable situations. We wait for more time and more resources. And, one day someone else built a product on that idea and we regret. We should look for perfection. That is fine. We should try to do things as good as possible and keep on improving along the way. We should not stop doing it because we cannot do it perfectly. 1. Which of the following statements is consistent with the author’s main idea as has been enumerated in the passage? (a) We should plan well for things and take conscious and well-informed decisions (b) We should be ready to do things when we have the perfect favorable situations (c) We should all have a passion and always live the passion perfectly (d) We should always try out the ideas we have in mind without thinking if we are perfect in it or not 2. Which of the following can’t be the most desirable attitude of a person as per the author? (a) Arindam is an MBA final year student; he wants to start his new venture, he plans to do it as soon as he completes his MBA degree and done a 1 year internship to learn the tricks of the trade (b) Rahul has a plot in mind that he wants to write a book on; he hasn’t ever written a book in his life but he sets himself on writing the book (c) Ruchi wants to be a singer. She is 25 years old and isn’t yet trained in singing. She starts taking lessons and starts doing open mic shows and karaoke events (d) Dhruv wants to become a politician. He knows he won’t be able to win in the nearby election, but still he goes full-out to win as an independent candidate 3. The author’s statement that “We should try to do things as good as possible” - (a) forms the conclusion of the author’s idea that we should not waste time to act upon our idea (b) forms the premise of the author’s idea that we should not waste time to act upon our idea (c) forms the conclusion of the author’s idea that we should follow our passion perfectly (d) forms the premise of the author’s idea that we should follow our passion perfectly

4. Which of the following quotations will actually weaken the main idea of the author in the passage? (a) Strive for continuous improvement, instead of perfection (b) Perfection is not attainable, but if we chase perfection we can catch excellence (c) Have no fear of perfection - you'll never reach it (d) You either be perfect or else you be nothing 5. Arup wants to become a chef. He quickly sets a kitchen, prepares some dishes and gives them to his friends for tasting. His friends loathe the taste and let him know that he is a bad chef, that he has no idea of the quantities of spices to add in the food. Arup too disliked the taste of his own dishes. Unfettered, Arup disregards all the criticism and goes on making the food in his own way. No negative reviews could slow him. Is his attitude correct as per the author? (a) No, Arup should have waited for his culinary skills to get perfect and then should have gotten his friends to taste the food (b) Yes, Arup didn’t bog down to negative criticism (c) No, Arup should have listened to his friends and learnt from his mistakes (d) Yes, Arup didn’t wait for his culinary skills to get perfect and set out in his journey Narendra: We should not take definite sides. It doesn’t matter whether our political leanings are rightist or leftist, we should support a party only one issue at a time. If BJP comes out with a good law, then we should support BJP on that issue. If BJP brings out a law that is bad for the country then we should protest against BJP on that issue. This will make a person logical and not a blind devotee. Amit: Your path will leave us neither here nor there. You have to have some sort of an opinion – either you are a rightist or a leftist. You should pick your side and support it all the way. This is the only way which can clear your stand. It doesn’t matter if you are called a blind devotee, at least you will have a cause in your life. 6. In support of which of the following does Narendra state his view that we should not take sides? (a) We should support a party only one issue at a time (b) We will be logical and not a blind devotee (c) We will have a fixed path (d) We will have to be firm on our stand 7. Narendra’s statement that “This will make a person logical and not a blind devotee”- (a) Forms the premise for his statement that we should not take definite sides (b) Forms the conclusion for his statement that we should support a party only one issue at a time

CLATutor Logical Reasoning 5

(c) Offers a clarification on which party to support at a given time (d) Forms the conclusion to his statement that we should not take definite sides 8. The patterns of reasoning in Amit’s argument closely resembles the pattern of reasoning in all of the following except - (a) Atul is a die-hard fan of Virat Kohli, but he wishes that Kohli would tone down his aggression on-field and not use abusive language (b) Oliver is a proud German. He is proud of the strict disciplinarian culture the country has. One day, he meets his Israeli friend Adam, who while talking brings up the issue of holocaust by the Nazi Germany. Oliver apologises for his country. (c) Fahim is a proud father of his fifteen year old son Mustafa. Mustafa is a brilliant student in his class and brings good marks in his exams. However, Fahim comes to know that Mustafa is a kleptomaniac and he has stolen the cricket-bat of his neighbor’s son Debdoot. But when Debdoot comes to complain about Mustafa to Fahim, the latter denies that his son could do such a job. (d) In this sutta, the Buddha describes the Noble Eightfold Path as the middle way of moderation, between the extremes of sensual indulgence and self-mortification. 9. Which of the following is the main conclusion of Amit’s arguments? (a) Narendra’s path won’t leave people neither here nor there (b) We should have a cause in life (c) We should have a firm stand and support it always (d) We should be a blind devotee and not care if anyone calls us that 10. Why does Amit say that we should have some sort of an opinion? (a) To be neither here nor there (b) To have a cause in our life (c) To be a blind devotee (d) To find which side we should support 11. In the last five minutes of a Maths exam-paper, Arun gets a question, where he gets the option of either choosing a geometry question or a set of three algebra questions, perfectly of same length. He decided to solve the algebra questions. Based on the information in the given passage, which of the following is most likely to be true? (a) The algebra questions involve shorter calculations than the geometry question (b) Arun has more confidence in Algebra than in geometry (c) Arun doesn’t mind not finishing the exam-paper (d) Both or either (a) or (b)

Rajeev has recently suffered a huge loss in his stock-market investments. It is not his first loss in this trade but this time he is upset. Much of his wealth has drowned in this whole exercise. He goes home with a broken-heart to give the news to his father, afraid how the old man would react. To his surprise, his father says, “No problem, God is there.” His father is unperturbed, which has amazed Rajeev. 12. Why was Rajeev broken-hearted? (a) Because he has suffered a huge loss in his stock-market investments (b) Because he has lost much of his wealth (c) Because he didn’t know how to face his father (d) Because his father is unperturbed even after hearing the news 13. What assumption has been made by Rajeev’s father when he says “No problem, God is there”? (a) God wanted Rajeev to suffer a loss (b) Rajeev believes in God (c) God will fix the problem they are in (d) Wealth is nothing as compared to God Sanjay: To become a coach, it should be made necessary that one should have been a very successful player in his younger age because if the person has not known how to handle the pressure of a live match then how can he/she give tips to present players of the game to handle pressure. Skill is something every player has at an international level, but handling the pressure is something totally different. Harsha: The coach needs to have good insights of the game. He/She must know the nook and corners of all the tricks and rules there are to the game. Handling pressure is a pressure in itself, tackling which is a skill, residing in the wiles of a person who has seen the game from its closest and to its purest form. These can only be possible if a person possesses good coaching acumen. Only he can be a coach worth of international standards. 14. Which of the following options is an incident that follows the pattern of reasoning as has been enumerated by Sanjay? (a) Mr. Shukla has appeared for becoming a teacher at a school. Though his school-grades are not excellent but he has good experience as a teacher and is quite successful. He gets the job. (b) Ravindran has done his masters in sports health and fitness and his specialty is in Marathon running. He has trained many Indian athletes. Ogoba, a Kenyan Marathon runner calls him up and asks him to be his fitness trainer. (c) Indra is a top-notch film producer. He is looking for a director who would direct a top-notch script for him. Amitabh Bachchan decides to take a shot at directing and

CLATutor Logical Reasoning 6

approaches Indra. Indra gives Mr. Bachchan the role of the direction of the film. (d) Both (a) and (b) but not (c) 15. Which of the following is a hidden assumption that has been made by Sanjay in his arguments? (a) All good players know how to manage pressure (b) Present generation players might not know how to handle pressure (c) All international standard players have the requisite skill-set (d) Handling pressure is much difficult than playing the game 16. Which of the following statements is true as per the arguments of Harsha? (a) A good coach can be a good player (b) A good coach should possess all the requisite set of skills required for a player (c) A good coach need not have been a good player (d) Skills are something that are gathered while coaching 17. Which of the following can be a suitable conclusion for Harsha’s arguments? (a) Handling pressure is not the supreme skill to judge who should be a coach (b) All good players can be good coaches if they have been good observers of the game (c) A person with in-depth knowledge of the game and good coaching skills can be a good coach (d) Handling pressure is a skill in itself, which good coaches possess We need to make rail travel more attractive to travellers. There are so many cars on the roads that the environment and human safety are under threat. Rail travel should be made cheaper. Everyone wants the roads to be less crowded, but they still want the convenience of being able to travel by road themselves. People will not abandon the car in favour of the train without some new incentive. 18. Which of the following is the conclusion of the above passage? (a) We need to make rail travel more attractive to travellers (b) Road transport leads to environmental problems (c) Everyone wants the roads to be less crowded (d) Rail travel should be made cheaper 19. Cycling is good for your general health. Research has shown that cyclists are less likely than other people to be depressed or to suffer from high blood pressure. Which of the following statements is correct? (a) The authors concludes that those who cycle have low blood pressure (b) The authors assumes that it is cycling reduces depression

(c) The above paragraph is not an argument (d) The conclusion of the author is that cycling is good for health 20. A huge chunk of public money is spent defending riverside farmland from flooding. Some of this money should be given to farmers to compensate them for taking such land out of production. This would save money and would benefit the environment, since if rivers were allowed to flood, their natural flood plains would provide wetland meadows and woodland rich in wildlife. What is the conclusion of this argument? (a) A huge chunk of public money is spent defending riverside farmland from flooding (b) If money is given to farmers for compensation then it would save money (c) Some of the money spent on defending riverside farmland from flooding should be given to farmers to compensate them for taking such land out of production

(d) Flooding of farmlands may not be a bad thing

CLATutor Logical Reasoning 7

Set II

The government’s decision to place a ban on alcohol advertisements is justified because it encourages young people to take up drinking. By young I mean really young – as in children. But even if it had no such influence on young people, it would be right to ban it because it could give existing drinkers the mistaken impression that their habit is socially acceptable. 1. Why does the author say that the government is justified

in banning alcohol advertisements? (a) Only because advertisements have the affect of

encouraging young people to start drinking (b) Only because advertisements have the affect of

encouraging drinkers to think that their habit is socially acceptable

(c) Both (a) and (b) are required to establish the reason for the ban

(d) Either (a) or (b) can establish the reason for the ban 2. What is the conclusion of the above argument? (a) Alcohol advertising encourages young people to take up

drinking (b) Ban on alcohol advertising is correct (c) Drinking is a socially unacceptable habit (d) People wrongly interpret drinking to be socially

acceptable 3. What role does the statement “By young I mean really

young – as in children” play? (a) It is the premise for establishing the conclusion that

alcohol advertising encourages young people to take up drinking

(b) It is the proof that young people take up drinking (c) It is the premise for establishing the conclusion that ban

on alcohol advertising is correct (d) None of the above 4. Which of the following options resemble the author’s line

of argument? (a) People who diet lose weight. Since Ramesh hasn’t lost

weight, he cannot have dieted. (b) It is important that we reduce the usage of mobile

phones. It reduces the creativity and also impairs vision of the users.

(c) Maintaining a healthy client-relation is important. It gives more revenue. If not then the client can refer to other clients.

(d) Ravi has seen a dip in his popularity among family-members. He must be doing drugs.

A majority of prospective parents would prefer to have sons

rather than daughters. So, if people can choose the sex of their child, it is likely that eventually there will be many more males than females in the population. So, sex-

determination of foetus must be discouraged. A preponderance of males in the population is likely to produce serious social problems.

5. Which of the following is the conclusion of the above argument?

(a) Sons are preferred over daughters (b) An skewed sex-ratio leads to social problems (c) Sex-determination leads to more males than females in

the population (d) Sex-determination of foetus must be stopped 6. According to the author which of the following will be

consequence if sex-determination is allowed? (a) It will make people prefer sons over daughters (b) It will lead to social stigma of females (c) It will empower the people to choose the foetus (d) It will lead to a skewed sex-ratio Sushila talks a lot and is very extrovert. Aniket talks little

and is highly introvert. They get married in the most unlikeliest of weddings. Their son Rustom talks moderately and is neither extrovert nor introvert.

7. Which one of the following uses flawed reasoning that most closely resembles the flawed reasoning used in the argument above?

(a) Jack went to Class 11th and enjoyed 2 games period. James went to Class 9th and enjoyed 4 games period. Therefore, Jinny enjoyed 3 games period as she went to class 10th

(b) Shreya bought an east facing house and gets sunlight till 1pm. Kailash bought a west facing house and gets sunlight from 1pm to 6pm. Therefore, Caria’s house get full day sunlight as her house is in the middle of both.

(c) Solution A is bitter in taste. Solution B is extra sweet in taste. To prepare a dish, both solutions were mixed. Therefore, the dish was perfect in taste.

(d) Surya is sitting in the leftmost side of arrow and could see only 2 lions of Ashoka’s pillar. Zubeda is sitting in the right corner of a row and could also see only 2 lions. Therefore, Meenakshi could see only 1 lions as she is sitting at the centre.

Situation: Rakesh went to a painting exhibition and saw an

abstract art. He did not understand anything but bought it by paying $10,000.

Analysis: Sometimes people gives importance to those things that they cannot comprehend.

8. The analysis provided for the situation above is most appropriate for which one of the following situations?

(a) Sohan is an illiterate. He sent his son to the English medium school. He was told by his son to buy him an

CLATutor Logical Reasoning 8

Atlas. He went to the mall and bought the costliest atlas for him, despite of the fact he had no knowledge of it.

(b) Sulekha’s native place is Kerala. She does not know any language rather than Malayalam. Being a famous doctor, she got an invitation to the biggest Hindi literature festival. She went there and bought the masterpiece of that festival.

(c) Sushil’s sister got infected with typhoid. He went to a medical store and bought medicine that cost him nearly Rs 5,000

(d) Anil’s son wedding date got finalized. 15 days before the wedding day, he bought the most updated car for him as his son was an automobile engineer.

Situation: Surya resides in a village. He got a job in metro

city. Company provided him a fully furnished house along with a washing machine. He still washes his clothes with hands.

Analysis: Many people love to be live in their habituated environment.

9. The analysis provided for the situation above is most appropriate for which one of the following situations?

(a) Karan bought a new car, but he does not know how to drive it. He still goes to office with his old scooter.

(b) Sunaina bought a microwave. She still uses stove to cook food as her aunt told her that microwave food causes cancer

(c) Ashmita’s dad had a farm. He used to do organic farming in it. Ashmita used to help her father in that. After 20 years, new technologies and plant drugs introduced. She still uses vermicompost as a manure.

(d) Kamiya bought a sweing machine. She soon found that it’s a damaged piece. So, she still uses needle and thread to stitch clothes.

Situation: Vaibhav spent all his money to see a Shahrukh

Khan’s movie. He had earlier read the reviews of movie and knew that it was a flop movie. However, he disregarded the reviews because Shahrukh was his favourite hero.

Analysis: Most of the time people go by their whims rather than doing logical thinking

10. The analysis provided for the situation above is most appropriate for which one of the following situations?

(a) Shyamlal is a diabetic patient. His doctor prescribed him not to eat sweets frequently. During Diwali, he ate 1kg laddoos, ignoring doctor’s advice.

(b) Chandralal is fond of bursting crackers. However, he knew that bursting crackers causes environmental pollution. He still burst many crackers during his brother’s wedding.

(c) Ravina runs a restaurant. She knew that smoking is injurious to health. She still made a separate room for smoking.

(d) Gaurav likes roadtrip over railway transport. But he knew roadtrip is costliest of the two. He needs to go to Mumbai from Chennai for an urgent meeting. Due to the unavailability of tickets, he went by car.

Radhika: Doctor, I read an article that claimed that the first few hours after birth are very important in establishing a mother-infant bond, which is the first step in building a healthy relationship. Can you assure me that my relationship with my baby has not been permanently harmed by our separation for several days after his birth?

Doctor: Your relationship with your child has not been harmed by the separation. Mother-infant bonding is not like “instant glue” that cements your relationship forever. Having your infant with you during the period immediately after birth does give your relationship a head start, but many factors are involved in building a strong and lasting relationship between a mother and her child.

11. Doctor’s statement that “Mother-infant bonding is not like ‘instant glue’ that cements your relationship forever”-

(A) Forms the premise for his statement that many other factors are involved in building a strong and lasting relationship between a mother and her child.

(B) Forms the conclusion for his statement that relationship with a child is not harmed by the separation.

(C) Offers a clarification on whether mothers have a strong relationship with a child or not.

(D) Forms the conclusion to his statement that the first few hours after birth are very important for establishing a mother-infant bond.

12. Which of the following is the main conclusion of Doctor’s arguments?

(A) Mother’s relationship with her child gets harmed by the separation between them.

(B) Many factors are involved in building a strong and lasting relationship between a mother and her child.

(C) There is no real connection between the mother and her child.

(D) None of the above.

13. Which of the following is a hidden assumption that has been made by Doctor in his arguments?

(A) The first impression is the last impression. (B) Initial bond is not the only thing which defines the

relationship between the mother and her child. (C) First impressions do last long. (D) There is nothing like “instant glue” that cements the

relationships forever.

CLATutor Logical Reasoning 9

14. If everything which Radhika read in the article is correct, which one of the following must be true?

(A) The best relationships between mothers and their children are caused by immediate mother-infant bonding.

(B) A strong and lasting relationship is necessary for mother-infant bonding.

(C) First few hours after birth is the first step in building a healthy relationship between mother and child.

(D) Immediate mother-infant bonding is not necessary for a strong and lasting relationship between a mother and her child.

15. The doctor does which one of the following in her reply to Radhika?

(A) She rejects an analogy in an attempt to reduce the Radhika’s concern.

(B) She cites evidence to show that the Radhika’s worry is unfounded.

(C) She misinterprets the Radhika’s explanation of her concern.

(D) She establishes that the article that patient read was in error.

A survey conducted by an NGO has showed that many

police personals in certain States are not satisfied with their jobs. The survey also showed that most of the dissatisfied police officers believe that they have little control over their job assignments. Therefore, to increase job satisfaction of the police officers, the State Government need only to concentrate on changing police officers’ beliefs regarding the degree of control they have over their job assignments.

16. Which one of the following, if true, would most seriously undermine the conclusion made by the author of the passage?

(A) The number of police officers in the state who are satisfied with their jobs is greater than the number who are dissatisfied.

(B) The police officers in the state are more dissatisfied than workers in other states.

(C) The police officers who are satisfied with their jobs believe that they have a lot control over their job assignments.

(D) The dissatisfied police officers feel that their wages are too low and working conditions are unsatisfactory.

17. Which of the following options is an incident that follows the pattern of reasoning as has been enumerated by the author in the above passage?

(A) Mr. Sharma is not satisfied with his job at company X. He quit his job at company X and joined company Y.

(B) Mr. Sharma is not satisfied with his job at company X because of low wages. Company X increased his wages to increase the job satisfaction of Mr. Sharma.

(C) Ashwin did not get selected in the Indian cricket team; therefore he started playing in Ranji matches to make his place in Indian cricket team.

(D) None of the above.

18. Authors’ statement that “the survey also showed that most of the dissatisfied police officers believe that they have little control over their job assignments”-

(A) Forms the premise for his statement many police personals are not satisfied with their jobs.

(B) Forms the conclusion for his statement that many police officers are not satisfied with their jobs.

(C) Forms the premise for his conclusion that to increase the job satisfaction of the police officers their beliefs regarding the degree of control they have over their job assignments should be changed.

(D) Forms the conclusion for his statement that many police officers are satisfied with their jobs.

19. Which of the following statement must be true? (A) All police officers are not satisfied with their jobs in

certain states. (B) All police officers are not satisfied with their jobs in certain

states because they have little control over their job assignments.

(C) Many police officers but not all police officers are dissatisfied with their jobs in certain states.

(D) All police officers are dissatisfied because they have little control over their job assignments.

20. Why was the police officers’ were dissatisfied with their jobs?

(A) Because the survey shows that. (B) Because they believe that they have little control over their

job assignments. (C) Because police officers’ are getting low wages. (D) Because there is no equal pay for equal work.

CLATutor Logical Reasoning 10

Set III

The electronic device which I bought for the garage last week has already broken. It’s just another example of the poor quality products that we are seeing more and more these days. The thing was probably manufactured in West Floret. 1. Which one of the following states the unstated premise

that underlies the author’s reasoning in the passage? (A) If a manufacturer uses poor quality products to make a

electronic device, the device is likely to break quickly. (B) If electronic device breaks quickly it was probably

manufactured in West Floret. (C) If an electronic device for garage was manufactured in

West floret, it should not be sold in this country. (D) None of the above.

2. Which of the following statement if true will actually

weaken the conclusion of the author in the passage? (A) Many but not all electronic devices which were

manufactured in West Floret are of poor quality. (B) All electronic devices which were manufactured in West

Floret are of poor quality. (C) All electronic devices which were manufactured in West

Floret are of good quality. (D) Many but not all electronic devices which were

manufactured in West Floret are of poor quality.

3. The author’s statement that “It’s just another example of the poor quality products that we are seeing more and more these days”-

(A) Forms premise of the above passage. (B) Forms conclusion of the above passage. (C) Forms assumption of the author to the above passage. (D) None of the above.

4. The patterns of reasoning in the above passage closely

resembles the pattern of reasoning in all of the following except –

(A) Atul bought hair dryer from the market, but it broke within a week, therefore he said that it must be made in China.

(B) Rajesh’s son bought sweets from market and gave it to his father, sweets were not of good quality, and Rajesh said that you must have bought it from Heeralal’s shop.

(C) Even after attending driving school Virat was not able to drive properly, Virat’s mother told him that you must have attended Shiksha driving school.

(D) Afsha bought a gadget for her kitchen and it broke within a week, and all the gadgets which break quickly must have been manufactured in China. Therefore, this gadget was probably manufactured in China.

Ankit: Trade unions are traditionally regarded by governments and economists as restraint of trade, working against the complete freedom of the economy, but I believe that unions are indispensable since they are often the worker’s only protection against exploitation.

Bhavik: I don’t agree. The exploitation of the workers and their work is a normal part of ordinary trade just like the exploitation of natural or other material resources.

5. In support of which of the following does Ankit state his view that trade unions are indispensable?

(A) They are the worker’s only protection against exploitation.

(B) They are against freedom of economy. (C) Exploitation of workers is necessary. (D) They are the restraint of trade.

6. Ankit’s statement that “unions are often the worker’s only

protection against exploitation”- (A) Forms the conclusion of the author’s argument that

trade unions are against the freedom of the economy. (B) Forms the premise of the conclusion that trade unions

are indispensable. (C) Offers a clarification on whether trade unions are

important or not. (D) None of the above.

7. Which of the following is the main conclusion of Bhavik’s

arguments? (A) Exploitation of the workers and their work is a normal

part of ordinary trade. (B) Trade unions are against the freedom of economy. (C) Exploitation of natural or other material resources are

necessary. (D) Governments and economists are against trade unions.

8. Which of the following statement if true will actually

weaken the conclusion of the Ankit? (A) Trade unions are for the worker’s protection against

exploitation. (B) Trade unions are against the complete freedom of the

economy. (C) Trade unions are not necessary for the worker’s

protection against exploitation. (D) Government is against the trade unions.

9. In the above passage, Ankit and Bhavik will not be able to

resolve their disagreement logically unless they- (A) Define a key term. (B) Rely on the opinions of established authorities. (C) Question an unproved premise. (D) Distinguish fact from opinion. There are at least three people in the room. At most two

people in the room recognize each other. At least one

CLATutor Logical Reasoning 11

person in the room recognizes everybody else in the room.

10. Which one of the following is NOT consistent with the above?

(A) Four people are in the room. (B) No two people in the room recognize each other. (C) At most one person in the room recognizes everybody

else in the room. (D) Anyone in the room who recognizes any other person in

the room is also recognized by that person.

11. Young people who imagine that the life of the writer is one of glamour, riches or fame soon discover not only the difficulties of the craft but the long odds against achieving any measure of recognition or financial security. Upon being asked, “Aren’t most editors failed writers?”, T.S. Eliot is said to have remarked, “yes, but so are most writers.”

The statement by T.S. Eliot conveys which one of the following ideas?

(A) The profession of editing can be just as creative and challenging as writing.

(B) Few writers are fortunate enough to attain real success in their profession.

(C) For a writer, success is measured more by the influence exerted than by material gain achieved.

(D) Many writers find that editorial work is more beneficial than writing.

In national surveys taken between 1970 and 1985 the

percentage of respondents who reported that they usually attended weekly services rose from 28 per cent to 34 per cent. However, statistics compiled during the same period by the nation's major religious denominations showed a gradual decline in attendance at weekly services.

12. Each the following, if true, could help explain the apparent contradiction in the statements above EXCEPT

(A) There was a sharp drop in the number of persons who attended religious services on an occasional basis.

(B) Attendance statistics compiled by the religious denominations are often highly inaccurate.

(C) As older churchgoers died, they were not replaced by an equal number of younger churchgoers.

(D) There was significant increase in attendance among religious groups outside the major denomination.

13. Which one of the following statements must be true as per the information provided in the above passage?

(A) Hinduism is a major religious denomination in the country and there is a gradual decline in attendance at weekly religious services.

(B) Hinduism is a major religious denomination in the country and there is a gradual increase in attendance at weekly religious services.

(C) Hinduism is a major religious denomination in the country and there is a gradual decline in attendance at weekly religious services between 2001 and 2015.

(D) None of the above. Technological education is worsening. People between

eighteen and twenty-four years, who are just emerging from their formal education, are more likely to be technologically illiterate than somewhat older adults. And yet issue for public referenda will increasingly involve aspects of technology.

14. Which of the following conclusions can be properly drawn from the statements above?

(A) If all young people are to make informed decisions on public reference many of them must learn more about technology.

(B) Thorough studies of technological issues and innovations should be made a required part of the public and private school curriculum.

(C) It should be suggested that prospective voters attend science courses in order to acquire a minimal competency in technical matters.

(D) If young people are not to be overly influenced by famous technocrats, they must increase their knowledge of pure science.

15. In the above passage why young people are technologically illiterate?

(A) Because teachers are not competent at school level. (B) Because education system is worsening. (C) Because technological education is worsening. (D) Because young people are not good students. Thirty years after African colonies began emerging as

nations, modern loyalties still often go first to the tribe. From Angola to Ethiopia, ethnic hatred has forced hundreds of thousands of people to flee their homes, making Africa the continent with the world's largest number of refugees.

16. Which of the following statements best summarizes the above passage?

(A) Africa is best characterized by ethnic fractionalization. (B) Angola and Ethiopia have the worst record of inter-

ethnic strife in Africa. (C) Continued warfare has made Africa a nation of refugees. (D) Africa is best characterized as a federation of many

states.

17. Which one of the following is the conclusion of the above passage?

(A) Ethnic hatred has made Africa the continent with the world's largest number of refugees.

(B) In Africa modern loyalties still often go first to the tribe.

CLATutor Logical Reasoning 12

(C) Angola and Ethiopia are the worst affected countries by ethnic hatred.

(D) African colonies are now emerging as nations.

18. The statement “ethnic hatred has forced hundreds of thousands of people to flee their homes”-

(A) Forms the conclusion of the above passage. (B) Forms premise of the conclusion that Africans are more

loyal towards their tribe. (C) Forms premise of the conclusion that Africa is the

continent with largest number of refugees. (D) Offers clarification to the author’s argument that

Africans are still loyal to their tribe. 19. ”Reservation of a short journey can be done only three

days in advance”- A note put u by Railway authorities. Which one of the following is an assumption made by the

author in the above argument? (A) People do not plan short journeys for more than three

days in advance. (B) Short journeys do not need reservations. (C) Administratively, it is not convenient for the Railways to

provide the facility of reservation of short journeys more than 3 days in advance.

(D) None of the above. 20. “Do not allow any candidate to leave the examination

hall until the examination is over, except when an emergency calls for.” –An instruction to the supervisor.

Which of the following is an assumption made by the author in the above argument?

(A) The supervisor himself is not supposed to leave the examination hall unattended.

(B) The supervisor has the authority to determine the extent of an emergency.

(C) Neither (A) nor (B). (D) Both (A) and (B).

CLATutor Logical Reasoning 13

Set IV

Dr. Mathur: Those who advocate replacing my country’s private health insurance system with nationalized health insurance because of the rising costs of medical care fail to consider the high human costs that consumers pay in countries with nationalized insurance: access to high-technology medicine is restricted. Kidney transplants and open heart surgery-familiar life saving procedures are rationed. People are denied their right to treatments they want and need. Dr. Kumar: Your country’s reliance on private health insurance denies access even to basic, conventional medicine to the many people who cannot afford adequate health coverage. With nationalized insurance, rich and poor have equal access to life-saving medical procedures. And people’s right to decent medical treatment regardless of income is not violated. Q.1In support of which of the following does Dr. Kumar state his view that a country should not have a private health insurance system? (A) Private health insurance denies access to basic,

conventional medicine to the many people who cannot afford adequate health coverage.

(B) Private health insurance provides access to basic, conventional medicine to the many people who cannot afford adequate health coverage.

(C) Neither (A) nor (B). (D) Both (A) and (B).

Q.2Dr. Mathur’s statement that “the high human costs that consumers pay in countries with nationalized insurance: access to high-technology medicine is restricted”- (A) Forms the conclusion for his statement that private

health insurance system is better than nationalized health insurance.

(B) Offers a clarification on which system is better. (C) Forms the conclusion to his statement that nationalized

health insurance is better than the private health insurance system.

(D) Forms the premise for his conclusion that in nationalized health insurance people are denied their right to treatment which they want and need.

Q.3The patterns of reasoning in Dr. Mathur’s argument closely resembles the pattern of reasoning in all of the following except – (A) Arun lives in Country X which has a private health

insurance system and he just had open heart surgery. (B) Arun lives in Country X which has nationalized health

insurance system and despite paying the high costs he cannot have access to high-technology medicine.

(C) Arun lives in Country X which has nationalized health insurance system and despite being poor he can have access to life-saving medical procedures.

(D) All of the above.

Q.4Which of the following is the main conclusion of Dr. Mathur’s arguments? (A) In nationalized health insurance system access to high-

technology medicine is restricted. (B) In nationalized health insurance system kidney

transplants and open heart surgery-familiar lives saving procedures are rationed.

(C) In nationalized health insurance system people are denied their right to treatments which they want and need.

(D) In nationalized health insurance system even after paying high costs access to high-technology medicine is restricted.

Q.5Dr. Mathur's and Dr. Kumar statement provides the most support for holding that they would disagree about the truth of which one of the following? (A) People's rights are violated less when they are denied an

available medical treatment they need because they lack the means to pay for it than when they are denied such treatment on non economic grounds.

(B) Where health insurance is provided by private insurance companies, people who are wealthy generally receive better health care than do people who are unable to afford health insurance.

(C) In countries that rely primarily on private health insurance to pay for medical costs, most people who would benefit from a kidney transplant receives one.

(D) In countries with nationalized health insurance, no one who needs a familiar medical treatment in order to stay alive is denied that treatment.

Q.6In responding to Dr. Mathur, Dr. Kumar employs which one of the following argumentative strategies? (A) Showing that the objections raised by Dr. Mathur have

no bearing on the question of which of the two systems under consideration are the superior system.

(B) Calling into question Dr. Mathur's status as an authority on the issue of whether consumers’ access to medical treatments is restricted in countries with nationalized health insurance.

(C) Producing counterexamples to Dr. Mathur's claims that nationalized health Insurance schemes extract high human costs from consumers.

(D) Showing that the force of Dr. Mathur's criticism depends on construing the key notion of access in a particular limited way.

Dolphins are regarded as the friendliest creatures in the sea and stories of them helping drowning sailors have been common since Roman times. The more we learn about dolphins, the more we realize that their society is more complex than people previously imagined. They look after

CLATutor Logical Reasoning 14

other dolphins when they are ill, care for pregnant mothers and protect the weakest in the community, as we do. Some scientists have suggested that dolphins have a language but it is much more probable that they communicate with each other without needing words. Could any of these mammals be more intelligent than man? Certainly the most common argument in favour of man's superiority over them that we can kill them more easily than they can kill us is the least satisfactory. On the contrary, the more we discover about these remarkable creatures, the less we appear superior when we destroy them. 7. Based on the information in the given passage, which of the following is most likely to be true? (A) Dolphins are quite abundant in some areas of the world. (B) Communication is the most fascinating aspect of the

dolphins. (C) It is not usual for dolphins to communicate with each

other. (D) Dolphins have some social traits that are similar to those

of humans.

8. What is the main idea of the author in the above passage? (A) Dolphins are more intelligent than man. (B) Humans are more intelligent than mammals. (C) Dolphins look after other dolphins when they are ill. (D) None of the above.

9. The author’s statement that “the more we discover about these remarkable creatures, the less we appear superior when we destroy them”- (A) Forms premise of the author’s idea that killing other

creatures does not establish humans’ superiority over other creatures.

(B) Forms conclusion of the above passage. (C) Forms conclusion of the author’s idea that humans are

superior to mammals. (D) Forms premise of the author’s idea that humans are

superior to mammals.

10. The fact that the author of the passage thinks that we can kill dolphins more easily than they can kill us-

(A) Means that they are better adapted to their environment than we are.

(B) Shows that dolphins have a very sophisticated form of communication.

(C) Proves that dolphins are not the most intelligent species at sea.

(D) Does not mean that we are superior to them. The reforms to improve the quality of public education that have been initiated on the part of suppliers of public education have been insufficient. Therefore, reforms must be demanded by consumers. Parents should be given government vouchers with which to pay for their children’s

education and should be allowed to choose the schools at which the vouchers will be spent. To attract students, academically underachieving schools will be forced to improve their academic offerings. 11. What is the underlying assumption of the author in the

above passage? (A) In selecting schools parents would tend to prefer a

reasonable level of academic quality to greater sports opportunities or more convenient location.

(B) Improvement in the academic offerings of school will be enforced by the discipline of the job market in which graduating students compete.

(C) There is a single best way to educate students. (D) Children are able to recognize which schools are better

and would influence their parents’ decision.

12. Which of the following statements is the author’s main idea as has been enumerated in the passage?

(A) Reforms to improve the quality of public education have been insufficient.

(B) Reforms to improve the quality of public education must be demanded by consumers.

(C) Quality of public education is deteriorating day by day. (D) None of the above.

13. The author’s statement that “The reforms to improve the

quality of public education that have been initiated on the part of suppliers of public education have been insufficient”-

(A) Forms the conclusion of the author’s idea that reforms must be demanded by consumers.

(B) Forms the premise of the author’s main idea that reforms must be demanded by consumers.

(C) Forms the conclusion of the author’s idea that Quality of public education is deteriorating day by day.

(D) Forms the premise of the author’s main idea that Quality of public education is deteriorating day by day.

14. Why does the author says that reforms must be demanded by consumers?

(A) To improve the public education system. (B) So that parents could pay off their children education fee. (C) Because the reforms to improve the quality of public

education that has been initiated on the part of suppliers of public education have been insufficient.

(D) So that academically underachieving schools will be forced to improve their academic offerings.

15. There is a great delay in obtaining the bank loan because a lot of time is spent in checking the creditworthiness of the applicant.

Which of the following would be the future course of action for the above statement?

CLATutor Logical Reasoning 15

(A) The credit worthiness clause should be removed completely.

(B) A part of the loan is to be sanctioned immediately with the high rate of interest while the remaining part to be sanctioned after regular checking of credit worthiness of the applicant.

(C) The system for checking the creditworthiness of the applicant is to be reviewed and simplified.

(D) None of the above.

So, we are in 2010. Remember the presentations, plans, articles, studies and forecasts from a couple of decades ago- many of them had titles like vision 2010, India superpower by 2010, Marching Towards 2010, etc. This is one of those years by when things needed to have been done. But no, we are nowhere near some of those forecasts that were made. Conveniently, those forecasts have been shifted to another nice, elegantly rounded year like 2020, or in case of a few over-eager forecasters, to 2050. 16. Which of the following statements is consistent with the

author’s main idea as has been enumerated in the passage? (A) We are in 2010. (B) 2010 is one of those years when things needed to have

been done. (C) Promises for 2010 have not been fulfilled and have been

shifted to later rounded years. (D) None of the above.

17. The author’s statement that “those forecasts have been

shifted to another nice, elegantly rounded year like 2020, or in case of a few over-eager forecasters, to 2050”-

(A) Forms premise of the author’s main idea that promises for 2010 have not been fulfilled and have been shifted to later rounded years.

(B) Forms conclusion of the author’s main idea that promises for 2010 have not been fulfilled and have been shifted to later rounded years.

(C) Offers a clarification on 2010 was a successful year. (D) Forms a premise to the author’s conclusion that 2010

was a successful year.

18. Which of the following statements will actually weaken the main idea of the author in the passage?

(A) The forecasts that were made in the year 1990 for vision 2010 are not achieved at all.

(B) All the forecasts which were made for vision 2010 have been achieved by year the 2010.

(C) The remaining targets of vision 2010 can be achieved by the year 2020.

(D) India will become superpower by the year 2050.

19. Based on the information in the given passage, which of the following is most likely to be true?

(A) India will become a superpower by the year 2050.

(B) India will never become a superpower. (C) India will achieve all its targets by the year 2050. (D) None of the above.

20. Is the author satisfied with the progress that has been

made by India till now? (A) Yes, but it should have achieved all its targets by the year

2010. (B) No, because India’s neighboring nations have made more

progress than India. (C) No, because all the presentations, plans, articles, studies

and forecasts from a couple of decades ago are not achieved.

(D) Yes, because India has made progress then Pakistan.

CLATutor Logical Reasoning 16

Set V

More and more organisations in the world today are prepared to carry out criminal acts that involve the taking of innocent people as hostages. One of the most urgent problems on the agenda of most Western European governments is terrorism and how to deal with it. In handling the situations that arise, all agree that swift and effective action must be taken to combat the terrorist, but, for some reason, when terrorism arrives on their doorstep in the form of hijacked airplanes, for example, some governments give in all too quickly to the demands of the terrorist. It is understandable that governments must act to safeguard the welfare of their citizens and other innocent people caught up in the criminal act. However, what the governments seem to ignore is that if you give in to the terrorist once, he or she will play upon your weakness and therefore be encouraged to indulge in terrorist acts on future occasions. 1. Which of the following conclusions best describes the

views of the writer of the above passage? (A) There is a subtle but certain difference between state

terrorism and terrorist acts carried out by individuals or members of illegal terrorist groups.

(B) One man's Government make a distinction between terrorism carried out for financial gain and that came out in the name of a political organisation.

(C) Surrender terrorist is another man’s freedom fighter. (D) Surrender to blackmail and you invite more blackmail. 2. As per the author which of the following must not be the

desirable attitude of the Government while dealing with terrorism?

(A) The Government should not give too quickly to the demands of the terrorists.

(B) The Government should first safeguard the welfare of their citizens.

(C) The Government should give too quickly to the demands of the terrorists.

(D) None of the above.

3. The author’s statement that “some governments give in all too quickly to the demands of the terrorist”-

(A) Forms the conclusion of the author’s idea that governments must act to safeguard the welfare of their citizens.

(B) Forms the premise of the author’s main idea that Surrender to blackmail and you invite more blackmail.

(C) Forms the premise of the author’s main idea that governments must act to safeguard the welfare of their citizens.

(D) Forms the conclusion of the author’s idea that Surrender to blackmail and you invite more blackmail.

4. Which of the following statement will actually weaken the main idea of the author in the passage?

(A) There are many pieces of evidences to show that terrorists whose demands are met with never got indulged in terrorist activities again.

(B) Once the demands of terrorists are fulfilled they came up with some new demands.

(C) Government is more pro-active towards safeguarding the welfare of their citizens.

(D) Terrorists are playing upon the weakness of the government.

5. According to the author why does the governments give in all too quickly to the demands of the terrorist?

(A) To safeguard the welfare of their citizens and other innocent people caught up in the criminal act.

(B) To safeguard the interest of the capitalist of their nation. (C) To safeguard the interest of political organisations. (D) To prevent future terrorist attacks.

Ravi: Economists emphasize on growth and every economic

theory is based on filling the gap between demand and supply. The more the consumerism, the more is the production and the more will be circulation of money and therefore the more will be the growth. However, they have failed to understand one basic fundamental of science that is “The law of conservation of energy”.

6. Which of the following will resolve the apparent paradox in above argument:

(A) Although energy remains conserved, the value of money will go down with time.

(B) Although the growth is attained, the energy cannot be enhanced.

(C) Growth is at the cost of energy. (D) Resources are always scars and compete against each

other but energy transforms from one form to other. Siberian birds fly miles to migrate to India in winter months.

Unfortunately, the migration of the birds in past few years is on the decline. The environmentalists have attributed global warming to such a change. As a lay man, I understand the cold climatic conditions in the polar region to be the reason for the migration which should be facilitated by global warming and not act as a deterrent to migration.

7. The author's understanding is flawed because: (A) He fails to relate to the concept of migration. (B) He could not follow the environmentalists concern with

respect to migration. (C) He is not able to relate to the habitat requirement of the

migratory birds. (D) None of the above. All soldiers are brave. Nikhil is brave.

CLATutor Logical Reasoning 17

8. Which of the following is the conclusion of the above argument?

(A) Nikhil is a soldier. (B) Nikhil is not a soldier. (C) Both (A) and (B) (D) Either (A) or (B).

9. Rakesh advises Mahesh that “it is desirable to keep

common medicines with you when you go on a long journey”.

Based on the above information which of the following statements must be correct?

(A) Rakesh assumes that Mahesh’s health is poor. (B) Rakesh is a medical representative. (C) One’s health is likely to get affected by the changes in

weather, water etc, while on the long journey. (D) None of the above. 10. A report has been published which provided that “In

order to increase the level of productivity and profit of the organisation, Corporate planning must be practical and known to every employee even at the lowest level.

Aslam wants to increase the productivity and profit of his organisation, based on the information provided above, what measures he should take?

(A) The policies of the organisation must be made transparent to all employees, even at the lowest level.

(B) Scientific methods must be followed to increase the production.

(C) All employees must participate in the process of corporate planning.

(D) All of the above.

There are three main factors that control the risks of becoming dependent on drugs. These factors are the types of drug; the personality of the individual and the circumstances in the drug is taken. Indeed, it could be said that the majority of the drug is taken. Indeed, it could be said that the majority of the adult population have taken alcohol, yet few have become dependent on it. Also, many strong drugs that have been used for medical purposes have not caused the patient to become addicted. However, it can be demonstrated that people who have taken drugs for fun are more likely to become dependent on the drugs. The dependence is not always physiological but may remain psychological, although the effects are still essentially the same. Those at greatest risk appear to be personalities that are psychopathic, immature or otherwise unstable.

11. Which of the following conclusion can be drawn from the text?

(A) One cannot become addicted to certain drugs if one has a strong personality.

(B) Taking drugs for “kicks” increases the possibility of becoming dependent on drugs.

(C) Psychological dependence is the greatest with heroin. (D) Alcohol is safe drug since very few people become

dependent on it.

12. Which of the following factor does not control the risks of becoming dependent on drugs?

(A) Types of drugs. (B) The personality of the individual. (C) The beneficial result of that drug. (D) Circumstances in which the drug is taken.

13. The author’s statement that “it can be demonstrated that

people who have taken drugs for fun are more likely to become dependent on the drugs”-

(A) Forms the conclusion of the author’s idea that Psychological dependence is the greatest with heroin.

(B) Forms the premise of the author’s idea that Psychological dependence is the greatest with heroin.

(C) Forms the conclusion of the author’s idea that taking drugs for “kicks” increases the possibility of becoming dependent on drugs.

(D) Forms the premise of the author’s idea that taking drugs for “kicks” increases the possibility of becoming dependent on drugs.

14. Which of the following statement will actually weaken the main idea of the author in the passage?

(A) Majority of the population has taken drugs for kicks and they do not have addiction to drugs.

(B) Psychological dependence is very strong with heroin, morphine, cocaine and amphetamines.

(C) Physiological dependence is great with heroin and morphine, but less with amphetamines, barbiturates and alcohol.

(D) Alcohol is safe drug since very few people become dependent on it.

Passengers in a boat are not allowed to stand. Because if they

are allowed to stand then this may raise the centre of gravity of the boat and the boat may topple over due to unstable equilibrium.

15. In the above passage, explanation provided for the assertion is-

(A) Correct. (B) Not correct. (C) Can’t say. (D) None of the above. The High Court located in the city of Jabalpur has ordered

the municipal corporation to repair all roads within the next fortnight.

16. Which of the following is a hidden assumption that has been made by the High Court in the above statement?

CLATutor Logical Reasoning 18

(A) The municipal corporation has the capability of carrying out the repair work within the next fortnight.

(B) The vehicle owners may start an agitation against the municipal corporation.

(C) Both A and B. (D) Neither A nor B. Cotton acreage in India during the current year has fallen by

15% as cotton growers have moved on to cultivation of other cash crops. This is the result of the cotton glut in world markets in post-September 2008 and the consequent slowdown in the world economy. But this scenario brought with it benefits to one segment of the industry yarn manufacturers as they got higher prices for their produce. Some yarn manufacturers too had stuck up on low-priced cotton last year. The combined effect of all this is evident in the rise in net profits and net margins of yarn manufacturers.

17. Which of the following is an inference which can be made from the facts stated in the above paragraph?

(A) The cotton industry grew tremendously post-September 2008.

(B) The yarn manufacturers have marginally suffered during post September 2008 period.

(C) India was the largest cotton producer earlier. (D) Cotton production will surely grow in the upcoming

years.

18. Which of the following conclusions can be drawn from the facts stated in the above paragraph?

(A) Now cotton acreage will never face a crisis like the crisis of September 2008.

(B) USA is the largest producer and consumer of cotton. (C) There has been a huge drop in the supply of cotton during

the current year. (D) Cotton growers are no more interested in the production

of cotton.

19. What we can say about the following statement? “There has been shortage of cotton in the world market last year.”

(A) Definitely true. (B) Definitely false. (C) Probably true. (D) Nothing can be said.

20. Which of the following argument will strengthen the

statement “The farmers will again grow cotton next year due to increased price of cotton yarn”?

(A) Govt plans to increase the subsidy on cash crops including cotton.

(B) There is a high alert in the entire Europe due to terrorist threats.

(C) Alternative cash crops are evergreen and unaffected by an economic slowdown.

(D) Govt is planning to import more cotton from abroad.

CLATutor Logical Reasoning 19

Set VI 1. Which of the following inferences may be drawn from the

statements in the above passage? (A) The Indian economy has been unaffected by any global

slowdown. (B) India was affected most in the world due to the global

economic slowdown. (C) The world economy witnessed an upward trend during

pre-September 2008. (D) Govt should discourage the production of cotton and

encourage shifting to other cash crops. Blood tests used to determine pregnancy can at times be

inconclusive. This means that the test has been unable to determine if the woman is pregnant or not pregnant. Regardless, some doctors refuse to perform further surgeries because of an inconclusive pregnancy test result.

2. If the statements above are true, which of the following conclusions is strongly supported by them?

(A) Pregnancy tests should always be given prior to surgery. (B) Most women with inconclusive pregnancy tests are

actually pregnant. (C) Some surgical procedures are affected by whether or not

a woman is pregnant. (D) Some doctors require a pregnancy test be given when

evaluating female candidates for any surgery.

3. What assumption has been made by doctors when they refuse to perform surgeries because of an inconclusive pregnancy test result?

(A) The woman can be pregnant. (B) Woman cannot be pregnant. (C) Pregnancy does not affect the surgery. (D) All of the above. The cost of producing radios in Country Q is ten percent less

than the cost of producing radios in Country Y. Even after transportation fees and tariff charges are added, it is still cheaper for a company to import radios from Country Q to Country Y than to produce radios in Country Y.

4. The statements above, if true, best support which of the

following assertions? (A) Labour costs in Country Q are ten percent below those in

Country Y. (B) Importing radios from Country Q to Country Y will

eliminate ten percent of the manufacturing jobs in Country Y.

(C) The fee for transporting a radio from Country Q to Country Y is more than ten percent of the cost of manufacturing the radio in Country Q.

(D) The tariff on a radio imported from Country Q to Country Y is less than ten percent of the cost of

manufacturing the radio in Country Y.

There is a growing feeling amongst the larger Asian countries that the West is passé. The news coming out of there, especially since the 2008 financial crisis, is of declining populations, big layoffs and economic meltdowns in several countries of the European Union (EU), Italy being the latest. So much of bad news over an extended period gives the impression that the rise of the West has finally halted; that the West is now in cowering retreat.

5. Which of the following weakens the argument provided in the passage above?

(A) The Western media has contributed to perpetuating the news of the decline of the West.

(B) The U.S. has a growing population and remains the most productive and innovative in the world, as well as militarily the most powerful.

(C) Both (A) and (B). (D) Neither (A) nor (B).

6. What is the main idea of the author in the above passage? (A) Economically, west is not doing well. (B) Despite all the problems west is growing again. (C) The U.S. is the most productive and an innovative

country in the world. (D) Currently growth rate of Asian countries is more than

that of the western countries. 7. Which of the following strengthens the argument

provided in the passage above? (A) Despite all the problems west is growing again. (B) GDP of all European Nations is down from 5.4% to 2.3% in

the 2017-18 financial years. (C) The Western media has contributed to perpetuating the

news of the decline of the West. (D) The U.S. has a growing population and remains the most

productive and innovative in the world, as well as militarily the most powerful.

It is perhaps no surprise that political parties are deeply divided over the idea of holding simultaneous elections for the Lok Sabha and the State Assemblies. During consultations initiated by the Law Commission of India, nine parties opposed it, arguing that it went against the constitutional fabric and that it would be impractical. Four parties backed the concept.

8. Which of the following strengthens the case for simultaneous elections?

(A) A simultaneous poll, particularly in this era where news is easily and widely disseminated, will privilege national issues over regional ones even if, arguably, the reverse may happen too.

CLATutor Logical Reasoning 20

(B) Assemblies can be bunched into two categories based on whether their terms end close to the 2019 or the 2024 Lok Sabha elections and then elections could be held for one group in 2019, and for another in 2024 so that subsequent elections could be synchronised.

(C) Election expenditure will be drastically cut and ruling dispensations will be able to focus on legislation and governance rather than having to be in campaign mode forever.

(D) None of the above.

9. Which of the following strengthens the case against the simultaneous elections?

(A) In a simultaneous election, national issues would tend to come to the fore and drown out issues of regional interest.

(B) In simultaneous elections the cost of conducting elections will come down, since the personnel required to conduct elections will be brought down.

(C) Simultaneous elections will help elected governments and ruling parties focus on governance, instead of preparing for elections somewhere or the other in the country.

(D) Simultaneous elections will stop policy paralysis because of Model Code of Conduct being in place multiple times in the five-year tenure of government, whether at the Centre or in the states.

10. Why did the nine parties opposed the idea of holding simultaneous elections for the Lok Sabha and the State Assemblies?

(A) Because in simultaneous elections the cost of conducting elections will come down, since the personnel required to conduct elections will be brought down.

(B) Because simultaneous elections went against the constitutional fabric and that it would be impractical.

(C) Because in a simultaneous election, national issues would tend to come to the fore and drown out issues of regional interest.

(D) All of the above. India will soon become a nation producing world leading

products developed by a large number of small local entrepreneurs.

11. Which of the following is/are valid argument/arguments?

(A) No. The true potential of entrepreneurship can be attained not by having too many start-ups but by ensuring that these start-ups grow into sizeable businesses.

(B) Yes. Starting up a business is a hard task and growing it is much easier.

(C) No. We support seeding of businesses but once these businesses are successful, our policies simply do not seem to help them.

(D) Only (A) and (C).

The problems of the Indian judiciary at all levels have

reached catastrophic levels. The public is losing confidence in the judiciary despite the latter’s assertions. Data show that they are acting on this belief by filing fewer cases year on year. It is likely to be a combination of delays, cost, uncertainty, inefficiency and corruption. Not one of these problems is solved to any degree by centralizing the manner of recruitment of judges. On the contrary, this endless, stagnant debate on the All-India Judicial Service (AIJS) only takes up time and energy instead of focusing attention on implementing more direct solutions to address the problems of the Indian judiciary.

12. Which of the following statements, if true, can strengthen the argument in the passage?

(A) The Bar Council of India has mismanaged legal education and barring a few islands of excellence, almost no effort has gone into improving the standard of legal education across the country.

(B) Under the AIJS, The judiciary plans to offer a lucrative pay scale and a lot of avenues for career advancement.

(C) A lot of effort has been undertaken by the Supreme Court to ensure uniformity in pay scales across States through its orders in the All India Judges’ Association case.

(D) The positions after AIJS implementation will come with sufficiently good terms and conditions of service, and a defined career progression.

13. What is the main idea of the author in the above passage? (A) Indian judicial system is not so efficient and

implementing All-India Judicial Service (AIJS) is not a solution to the problems of the Indian judiciary.

(B) There is a lot of delay in the final adjudication of the cases. (C) There is a corruption in the Indian judiciary and

corruption is making it inefficient. (D) None of the above.

14. The author’s statement that “It is likely to be a

combination of delays, cost, uncertainty, inefficiency and corruption”-

(A) Forms the premise of the author’s idea that under the AIJS, The judiciary plans to offer a lucrative pay scale and a lot of avenues for career advancement.

(B) Forms the conclusion of the author’s idea that under the AIJS, The judiciary plans to offer a lucrative pay scale and a lot of avenues for career advancement.

(C) Forms the premise of the author’s idea that the Indian judicial system is not so efficient and implementing All-India Judicial Service (AIJS) is not a solution to the problems of the Indian judiciary.

(D) Forms the conclusion of the author’s idea that the Indian judicial system is not so efficient and implementing All-India Judicial Service (AIJS) is not a solution to the

CLATutor Logical Reasoning 21

problems of the Indian judiciary.

15. Which of the following statements, if true, can weaken the argument in the passage?

(A) Creation of AIJS will attract young and bright law graduates and help build a new cadre that can enhance accountability in the governance system.

(B) Under the centralized process of recruitment, if judges from north India were to be transferred to the south, for instance, they wouldn’t be able to understand the language and hence conduct proceedings properly.

(C) Instituting an AIJS would mean that nationally dominant SC, ST and OBC groups would be at an advantage as they can compete for posts across the country, which they would otherwise be disqualified from because of the domicile requirement. Thus an AIJS will have consequences for State-level politics.

(D) District judges will be recruited centrally through an all-India examination and allocated to each State along the lines of the AIJS.

16. According to the author in the above passage what is the solution to the problems of judiciary?

(A) There should not be a delay in the delivery of the judgement.

(B) Fast track courts should be established. (C) Centralization of the judiciary. (D) None of the above.

The attraction for jihadists of an attack on the seat of

Western Christendom is certainly not. So it is remarkable that Italy should not have experienced a single deadly jihadist attack when Britain, France, Germany and Spain have all been targeted—not least because it undermines the argument for a link between illegal immigration and terrorism (in the first half of 2017, Italy accounted for 82% of unauthorized arrivals in Europe). The most colourful explanation for the Italian exception is that Italy’s mafias have quietly deterred jihadists from gaining a toehold.

17. Which of the following statements, if true, can weaken the argument in the passage?

(A) The fight against Italy’s formidably organized criminals has given its police a wealth of experience in monitoring tightly knit target groups.

(B) Organized crime and terrorism have also encouraged judges to take a more expansive attitude than in other European countries to issuing warrants for wiretaps and particularly to the electronic surveillance of suspects’ conversations.

(C) Italy’s mobsters exert greater control in the south, whereas a sizeable majority of its Muslims live in the north.

(D) Al-Qaeda the dominant force in the jihadist world used Italy as its logistical base in Europe and this is what kept

them safe.

18. Which of the following statements, if true, can strengthen the argument in the passage?

(A) The fight against Italy’s formidably organized criminals has given its police a wealth of experience in monitoring tightly knit target groups.

(B) Organized crime and terrorism have also encouraged judges to take a more expansive attitude than in other European countries to issuing warrants for wiretaps and particularly to the electronic surveillance of suspects’ conversations.

(C) Al-Qaeda the dominant force in the jihadist world used Italy as its logistical base in Europe and this is what kept them safe.

(D) All of the above.

The protests along Gaza’s border with Israel, which turned violent with Israeli troops killing 18 Palestinians, were long in the making. Gaza, the 225 sq km strip of land where over two million people live, has been under an Israeli blockade for over a decade. In recent years, Egypt has also joined the blockade, practically cutting off the strip from the rest of the world. Despite international calls and repeated warnings by rights groups, Israel has not eased its restrictions on the strip. It says they are in place for “security reasons” — the ruling Hamas is designated a terrorist group by Israel.

19. Which of the following statements shows that the fault may lie with Palestine as well?

(A) Israel could escape censure for the latest outbreak of violence in Gaza as well due to support from Trump.

(B) Gaza and the West Bank are ruled by rival factions- Hamas and Fatah- and there have been no joint efforts to mitigate the suffering of Gaza’s people.

(C) The international community remains unresponsive when it comes to the grave rights violations in this Mediterranean enclave.

(D) An UN-appointed commission probing the 2009 Gaza war accused both Israel and Palestinian militants of committing war crimes.

Palestinian government may be wrong. Option D is incorrect as it holds both Israel and Palestine to be wrong and not just Palestine. Only option B makes sense here. It states that Palestine itself is not united and faces internal squabbles and a lack of redressing domestic issues. Therefore, option (B) is the correct answer.

20. What is the main idea of the author in the above passage?

(A) Gaza strip has been wrongfully cut-off from the rest of the world.

CLATutor Logical Reasoning 22

(B) Israel is solely responsible for the human rights violation in Gaza strip.

(C) Egypt is solely responsible for the human rights violation in Gaza strip.

(D) Gaza Strip is part of the Palestine.

CLAtutor Logical Reasoning 23

VERBAL REASONING

Anology And Classification, Number Series And Ranking

1.Maharashtra: India: :Texas : ? (a) Canada (b) Mexico (c)Brazil (d) USA 2.Qualm: Nausea: : Burn: ? (a) Fresh (c) Sensible (b) Sear (d)Wet 3. Heart: Cardiologist: : Kidney : ? (a) Endocrinologist (b) Orthodontist (c) Nephrologists (d) Neurologist 4. Lamp : Oil : : Electric bulb : ? (a) Current (b)Bright (c) Holder (d) Switch 5. Poem: Poet::?:Book (a) Editor (b) Author (c) Writer (d) Publisher 6. Cataract: Eye: :Pneumonia: ? (a)Brain (b) Ear (c)Lungs (d) Nerves and Limbs 7. Whale: Mammal: : Turtle: ? (a) Amphibian (b) Reptile (c) Fish (d) Mollusc 8. Optimist : Cheerful : : Pessimist : ? (a)Petty (b) Selfish (c) Gloomy (d)Mean 9. Money : Misappropriation : : Writing:? (a) Deception (b) Mistake (c) Plagiarism (d) Theft 10. Saint: Meditation:: Scientist: ? (a) Research (b) Knowledge (c) Spiritual (d) Rational 11. King: Palace: : Eskimo : ? (a) Caravan (b) Asylum (c) Monastery (d)lgloo

12. Marathon: Race: : Hibernation: ? (a) Winner (b)Beat (c) Dream (d)Sleep 13. Restaurant: Menu: : ? (a)Library: Catalogue (b) Journal: Newspaper (c) Book : Encyclopedia (d) College: Account 14. Chisel: Sculptor: : Harrow: ? (a) Gardener (b) Mason (c) Blacksmith (d) Guard 15. Moon: Satellite: : Earth: ? (a) Sun (b) Planet (c) Solar system (d) Round 16. Umpire: Game: : ? (a) Prodigy: Wonder (b) Chet: Banquet (c) Legislator : Election (d) Moderator: Debate 17. Tanning: Leather: : Pyrotechnics:? (a) Bombs (c) Wool (b) Fireworks (d) Machinery 18, King: Throne: : Rider: ? (a) Chair (b) Horse (c) Seat (d) Saddle 19. Head: Hair: Hand:? (a) Finger (b) Ear (c) Neck (d) Knee 20. Walking : Running : : Wind : ? (a) Weather (b) Air (c) Rain (d) Storm 21, Cobbler: Leather: : Carpenter: ? (a) Furniture (b) Wood (c) Hammer (d) Chair 22. Foot : ? : : Hand : Wrist (a) Length (b) Shoe (c) Ankle (d) Leg 23. Picture : See : : Book: ? (a)Buy (b)Read (c)Listen (d)Library

CLAtutor Logical Reasoning 24

24. Dark: Light:: ? (a) Polluted : Contaminated (b) Accrued : Accumulated (c) Ill : Diseased (d) Hot: Cold 25. Swimming: River ::Hinking: ? (a) Road (b) Pond (c) Mountain (d) Sea 26. House: Door:: Compound: ? (a) Gate (b) Fence (c) Foundation (d) Wall 27. Hongkong : China: : Vatican: ? (a) France (b) Mexico (c) Canada (d) Rome 28. Virus : Smallpox: : Bacteria: ? (a) Chickenpox (b) Typhoid (c) Malaria (d) Sleeping sickness 29. Home: Kitchen: :Plant: ? (a) Root (b) Soil (c) Leaf (d) Stem 30. Book: Shelf : : Clothes : ? (a) Hanger (b) Cupboard (c) Wardrobe (d) Bureau 31. Procession: Route: : Earth: ? (a) Space (b) Sun (c) Orbit (d) Highway 32. Fertilizers : Crops : : ? (a) Teacher: Education (b) Chlorine : Water (c) Tonic: Body (d) Pesticide: Rats 33. Fish: Scales: : Bear: ? (a) Feathers (b) Leaves (c) Fur (d) Skin 34. Writer: Pen: : ? (a) Needle : Tailor (b) Artist : Brush (c) Painter: Canvas (d) Teacher: Class 35. Country: President: : State: ? (a) Chief Minister (b) Prime Minister (c) Speaker (d) Governor 36. Mirage: Desert: : ? (a) Sky: Illusion (b) Rainbow: Sky

(c) Rain: Rainbow (d)Image: Mirror 37. Timid: Ass::Cunning:? (a) Ant (b) Fox (c)Rabbit (d) Horse 38.Ecstasy: Gloom: : ? (a) Congratulations : Occasion (B) Diligent: Successful (c) Measure: Scale (d) Humiliation : Exaltation 39. Stethoscope : Heartbeat : : ? : Temperature (a) Heat (b) Mercury (c) Scale (d) Thermometer 40. Spring: Summer: : (a) Sunday: Monday (b) Thursday: Wednesday (c) Tuesday: Friday (d) Friday: Monday

41. What should come in the following number series?

02462302464502466702?

(a)4 (b)2 (c)6 (d)5

42. What should come next in the following letter series?

AZABYABCXABCDWABCDEVABCDE

(a)U (b)T (c)A (d)None of these

43. Four of the following of five are alike in a certain

way and form a group. Which is the one does not belong

to that group?

(a)215 (b)143 (c)247 (d)91

44. Which of the following should come next in the

given sequence of numbers?

243243124312243123243123

(a)2 (b)3 (c)4 (d)5

45. Which of the following should come next in the

following letter series?

PPOPONPONMPONMLPONML

(a)P (b)K (c)J (d)I

46. What should come in next in the following number

series?

34434453445634456734456

(a)4 (b)5 (c)6 (d)7

CLAtutor Logical Reasoning 25

47. What should come in next in the following number

series?

987654321123456789987654322345678

(a)8 (b)1 (c)9 (d)2

48. Which of the following should come next in the

following letter series?

AABACABCDABCDEABCDEF

(a)A (b)G (c)H (d)B

49. What should come in next in the following number

series?

975311864229753186422975318642975

(a)1 (b)8 (c)3 (d)5

50.What should come in the place of question mark(?) in

the following letter sequence?

AZB, CYD, EXF, ? , IVJ,KUL

(a)FYH (b)GWG

(c)HWH (d)None of these

CLAtutor Logical Reasoning 26

Coding, Decoding And Inequalities

1.If E is replaced by 2, R by 5, T by 7, D by 3, S by 4, I by 6 and N by 9, then how is RESIDENT written, if its letters are reversed? (a)79326425 (b)79236425 (c)52463927 (d)s4263927 2.In a certain code language, DEAR is written as %#$?, RED is written as ?#%, MORE is written as @*?#. How is DREAM written in that code? (a) #?%$@ (b) #%?$@ (c)%?#$@ (d)?%#$@ 3. In a certain code language, BEAM is written as 5%LK and COME is written as $7K%. How is 'BOMB' written in that code? (a) 5%K5 (b)57K5 (c) $7K$ (d) 5$%5 Directions (Q. Nos. 4-6)Study the given information carefully to answer the given questions ‘Cinderalla shouted for rescue’ is written as ‘ pr co ly bu’ ‘Rescue all the bugs’ is written as ‘ ke mt co rx’ ‘Bugs ate all carrots’ is written as ‘vg rx ke sh’ ‘Carrots for pretty Cinderella’ is written as ‘ ly pr vg as’ 4. What is the code for 'bugs' in the given code language? (a) Other than those given as options (b) co (c) sh (d) Either 'ke' or 'rx' 5. What may be the possible code for 'pretty' in the given code language? (a) ly (b) pr (c)vg (d)as 6. What may be the possible code for 'shouted and ate' in the given code language? (a) bu sh mt (b) rx co gy (c) ly rx vg (d) gy sh as Directions (Q. Nos. 7-11) Study the given information carefully to answer the given questions. In a certain code language. 'good work definitely pays' is written as 'bd jq hl pr' 'you did good job' is written as 'on pr dm st' Job pays good amount 'is written as 'pr mi on jq'

'did he work properly' is written as 'hl zy st ex' (all codes are two letter codes only) 7. What is the code for 'properly' in the given code Language? (a)bd (b) Either dm or st (c) hl (d)Either zy or ex 8. In the given code language, what does the code 'jq' stand for? (a) did (c) good (b) job (d) pays 9. What is the code for 'amount' in the given code language? (a) sl (b) on (c) pr (d)mi 10. What is the code for 'did meet definitety' in the given code language? (a) cv on bd (b) st hl cv (c) bd st jq (d) cv bd st 11. What is the code for 'good job' in the given code language? (a) Other than those given as options (b) on pr (d) pr on (c) bd st (Q.Nos. 12-16) Study the given information carefully to answer the given questions. In a certain code language, 'few organic farming techniques' is written as li gs da cr' 'fertilizer products few available' is written as'fo pz nb gs' 'organic waste into fertilizer' is 'wrritten as 'nb cr pt mk' 'disposal of farming waste' is written as 'hu mk li yu' (all codes are two letter codes only) 12.What will be the code for 'few waste' in the given code language? (a) mk gs (b) gs Ii (c) pt da (d) da mk 13.What is the code for 'organic' in the given code language? (a) ls (b) cr (c) da (d)pt

CLAtutor Logical Reasoning 27

14. In the given code language, what does the code 'yu' stand for? (a) Farming (b) Techniques (c) Either 'of' or 'disposal' (d) Waste 15. If 'waste management techniques’ is coded as 'ax da mk' in the given code language, then how will 'farming fertilizer management' be coded as ? (a) ax nb ct (b) li ax pt (c) nb li ax (d) gs li nb 16. What is the code for 'available' in the given code language? (a) Either ‘pz’ or 'lo' (b) either 'nb or mk' (c) li (d)hu Directions (Q. Nos. 17-21) Study the following information to answer the given questions. In a certain code, '8 2 9'means'how art thou', '9 5 8' means 'thou art good' and '1 5 8 7 3' means ‘thy good and thou bad'. 17.What may be the possible code for 'thy'? (a)1or 7 (b)7 (c)3 (d)1 or 7 or 3 18.What is the code for 'thou'? (a)9 (b)8 (c)2 (d)5 19. What is the code for 'how'? (a)5 (b)8 (c)2 (d)9

20. Which of the following may possibly be the code for 'thou no good'? (a)5 0 8 (b)7 8 0 (c)5 0 7 (d)7 8 0 21. What is the code for 'how good thou art'? (a)7 5 8 9 (b) 8 2 9 5 (c)7 1 8 3 (d)8 7 9 5 Directions (Q.Nos. 22-25) Study the following information to answer the given questions. 'he is waiting there' is written as 'la pa ro ta', 'there is the train 'is written as 'zo ro ji la', 'waiting at the station' is written as 'ma ta fu ji ' ' is this a station' is written as 'fu bi ro vi'. 22.What is the code for 'he'? (a)la (b) pa (c) ro (d) ta 23..What does 'la' stand for? (a) Is (b) train (c) waiting (d) there 24.Which of the following represents 'the train station'? (a) zo la ma (b) fu ji ta (c) lu li zo (d) ro zo lu 25. What is the coded for 'at' ? (a)ma (b) ji (c) tu (d) ta

Directions (Q. Nos. 26-30)Below are given letters A to Z. Under each capital letter a small letter is written which is to be used a code for the capital letter.

A B C D E F G H I J K L M

o z f t g l q n a v s w c

N O P Q R S T U V W X Y Z

b y h u d j r p x m e k i

In each of the following questions, a group of six capital letters is given and its equivalent code is given in one of the columns (a), (b), (c) or (d,). Study the group of letters given in each question and with the help of code given above, choose the code equivalents from amongst (a), (b), (c) or (d) as your answer. (a) (b) (c) (d) qvscjx ctloig zrmtis qvsliz tdjiwr wguxzg ataydx nxadjw afchij gjbfer tdihwr gjbrfl wgoxgz dmthpr wgurvs ataydp qujoxs ataynp wgoxgz tdiwrh giavyu naxowd grpmil qvpirp 26.IDIORV 27. LEQVEB 28. HIVALR 29. ESNTCF 30.DRZPLT 31.lf M = 13 and MAT = 34, then WAX=? (a)47 (b)25 (c)48 (d)23

CLAtutor Logical Reasoning 28

32.If RARE can be written as SBSF. How AREA can be written in that code? (a) FSBS (b) BSBF (c) SBFB (d) BSFB 33.In a code language, MASTER is coded as OCUVGT and RANGE is coded as TCPIG. How MANAGERS can be coded?

(a) OCPICGTU (b) OCPC]CTU (c) OCICPGTV (d) OCPICGTV

Direction (Qs. Nos. 34-35) below are letters A to Z. Under each capital letter a small letter is written which is to be used as a code for the capital letter.

A B C D E F G H I J K L M

f o i l q y b m t v g e r

N O P Q R S T U V W X Y Z

u x a w z j n p c h k s d

With the help of given codes (small letters), select the equivalent capital letters for the following. 34.efsogc (a) LZIOYK (b) TABKVY (c,) LAYBKV (d) TPCPVK

35.gulpbh (a) KNGWUD (b) KNDUGI (c) KDUGWN (d) KNDGUA

Directions (Q. Nos. 36-37) Below are given letters A to Z. Under each capital letter, a small letter is written which is to be used as a code for the capital letters.

A B C D E F G H I J K L M

f g k v w a o l z s u b H

N O P Q R S T U V W X Y Z

p r t c i x m y d j n q e

In each of the following Questions, a group of six capital Letters is given and its code equivalent is given in one of the columns (a), (b), (c) or (d). Study the group of letters given in each question and with the help of code given above choose the code equivalent from amongst (a), (b), (c) or (d) as your answer. 36.GSUWVM (a) ckdqaz, mcfywp, osagbi, ilkwzs, zrvlytr, gionfu (b)whiukf, biliqm, adrtiy, volsfy, reicxd, mbuoaz (c)ulkpad, foiznj, vswgfo, apixyt, amidak, fijpxf (d)zomyak, ycpath, lowvxr, oxyjdh, kphvsr, iyzrtwo 37. TLKGFD (a)ckdqaz, mcfywv, osagbi, jlkwsz, zrblyh, gionfu (b)whiuK, biliqm, adrtry, volsfy, reicxd, nbuoav (c)ulkpad, foizni, vswgfo, aPixYt, arnidal fipax (d)zomyak, ycpath, lowvxr, oxfldh, irpsvhr, yzuwo Directions (Q.Nos.38-40): Below are given letters A to Z. Under each capital letter a small letter is written which is to be used as a code for the capital letters.

A B C D E F G H I J K L M

f g k v w a o l z s u b h

N O P Q R S T U V W X Y Z

CLAtutor Logical Reasoning 29

p r t c i x m y d j n q e

(a) (b) (c) (d) grtmo bumtso dgrsto umyro tszlxm dgretq fzslxm dgrers buvrst tzsqxy dgrpst fzsacx dgtrlo bumrst bumvho burady bumlfo tzsgrp tzphxo tzslxm dgrlwx dgrwxy bumgrs bvmyst 38.TTLLPQZ 39.JGRIOX 40.WSNUDR Directions (Q. Nos.41-45) In these questions, relationship

between different elements is shown in the statements.

The statements are followed by conclusions. Study the

conclusions based on the given statements and select the

appropriate answer.

41. Statements S≤L≤I =P≥E > R; L > Q Conclusions I. P>S II. I>R (a) Only conclusion II is true (b) Only conclusion I is true (c) Both conclusions I and II are true (d) Either conclusion I or II is true 42. Statements

G>R≥E=A<T≤S; D≤A≤J Conclusions I. T≥D II. R>S (a) Only conclusion II is true (b) Only conclusion I is true (c) Both conclusions I and II are true (d) Neither conclusion I nor II is true 43. Statement A≥B >C≤D≤E < F Conclusions I. A≥E II. C>F (a) Only conclusion II is true (b) Only conclusion I is true (c) Both conclusions I and II are true (d) Either conclusion I or II is true 44. Statements

G> R≥E=A≤T≤S;D≤A≤J

Conclusions

I.J> G II. JG

(a) Only conclusion II is true

(b) Only conclusion I is true

(c) Both conclusions I and H are true

(d) Neither conclusion I nor II is true

45. Statements

S≤L≤I=P≥E>R;L>Q

Conclusions

I. L > R II. E≥Q

(a) Only conclusion II is true

(b) Only conclusion I is true

(c) Both conclusions I and II are true

(d) Neither conclusion I nor II is true

Directions (Q- Nos. 46-50) In these questions,

relationship between different elements is shown in the

statements. The statements are followed by conclusions.

Study the conclusions based on the given statements and

select the appropriate answer.

46. Statements

C>W≥E=R<B<S;H<R≥D

Conclusions

I. C > D II. H < S

(a) Either conclusion I or II is true

(b) Only conclusion I is true

(c) Both conclusions I and II are true

(d) Neither conclusion I nor II is true

47. Statements

V=D<Y <A≥G≥E;P=A;R<V

Conclusions

I. E > P II. P > E

(a) Either conclusion I or ll is true

(b) Only conclusion I is true

(c) Both conclusions I and II are true

(d) Only conclusion II is true

48. Statements

S >J=T < O; T < P

Conclusions

I. S > O II. P < S

(a) Either conclusion I or II is true

(b)Only conclusion I is true

(c) Both conclusions I and II are true

(d) Neither conclusion I nor II is t-rue

CLAtutor Logical Reasoning 30

49. Statements

V=D<Y< A≥G≥E;P=A;R<V

Conclusions

I. R < A ll. V > G

(a) Either conclusion I or II is true

(b) Only Conclusion I is true

(c) Both conclusions I and II are true

(d) Neither conclusion I nor II is true

50. Statements

C>W≥E=R<B<S;H<R≥D

Conclusions

I. B > C ll. W≥D

(a) Either conclusion l or II is true

(b) Only conclusion I is true

(c) Both conclusions I and II are true

(d) Neither conclusion I nor II is true

CLAtutor Logical Reasoning 31

Syllogism

Directions (Q. Nos. 1-5) In this question two/three statements followed by two conclusions numbered I and II have been given. You have to take the two/ three given statements to be true even if they seem to be at variance from commonly known facts and then decide which of the given conclusions logically follows from the given statements disregarding commonly known facts. 1. Statements: All races are sprints. Some races are contests. Conclusions I. Some contests arc sprints. ll. Al contests are sprints. (a) Only conclusion ll is true (b) Only conclusion l is true (c)Both conclusions I and II are true (d) Either conclusion I or II is true 2. Statements: No bunk is a locker. All banks are stores. No store is a panel. Conclusions I. No store is a locker. ll. No panel is a bank. (a) Only conclusion II is true (b) Only conclusion l is true (c) Both conclusions I and II are true (d) Either conclusion 1 or II is true 3. Statements Some strikes are hits. No strike is a raid. All attacks are raids. Conclusions I. Some hits are definitely not raids. II. All hits being strikes is a possibility. (a)Only conclusion II is true (b) Only conclusion I is true (c) Both conclusions I and II are true (d) Either conclusion I or II is true 4. Statements Some equations are formulae. All equations are terms. All terms are symbols. Conclusions l. All equations are symbols. II. No symbol is a formula. (a) Only conclusion II is true

(b) Only conclusion I is true (c) Both conclusions I and II are true (d) Either conclusion I or II is true 5. Statements: Some strikes are hits No strike is a raid. All attacks are raids. Conclusion I. No attack is a strike. II. All attacks being hits is a possibility. (a) Only conclusion II is true (b) Only conclusion I is true (c) Both conclusions I and II are true (d) Either conclusion I or II is true 6. Which of the following is definitely true if the statement given below is considered to be true? (You have to take the given statements to be true even if they seem to be at variance from commonly known facts and then decide which of the given conclusions logically follows from the given statements disregarding commonly known facts). Statements No kite is a bird. All reptiles are birds. All kites are amphibians. All amphibians are Plants. (a) All amphibians being is a possibility (b) All plants being reptiles is a possibility. (c) At least some reptiles are kites (d) Ail kites are plants Directions (Q. Nos. 7-10): In each of the questions below are given three statements followed by two conclusions numbered I and II You have to take the given statements to be true even if they seem to be a at variance from commonly known facts. Read all the conclusions and then decide which of the given conclusions logically follows from the given statement s disregarding commonly known facts. Give Answer (a) if only conclusion I follows (b) if only conclusion II follows (c) if either conclusion I or II follows (d) if neither conclusion I nor II follows

CLAtutor Logical Reasoning 32

7. Statements Some ratios are percent. All percent are fractions. No fraction is a section. Conclusions I. No section is a percent. II. All ratios being fractions is a possibility. 8. Statements All metals are plastics. All plastics are ores. Some ores are wood. Conclusions I. All wood being metals is a possibility. II. No ore is a metal. 9. Statements Some ratios are percent. All percent are fractions. No fraction is a section. Conclusions I. All sections being ratios is a possibility. II. Atleast some fractions are ratios. 10. Statements All metals are plastics. All plastics are ores. Some ores are wood. Conclusions I. Atleast some metals are wood. II. All plastics being wood is a possibility. Directions (Q. Nos. 11-15)In each question below are two statements followed by two conclusions numbered I and II. You have to take the three given statements to be true even if they seem to be at variance from commonly known facts and then decide which of the given conclusions logically follows from the three given statements disregarding commonly known facts. Then, decide which of the answer (a), (b), (c), (d) and (e) is the correct answer. 11. Statements Some huts are mansions. No mansion is a palace. Conclusions I. All palaces being huts is a possibility. II. All huts are palaces (a) Either conclusion l or II is true (b) Neither conclusion I nor II is true (c) Only conclusion I is true (d) Both conclusions I and II are true 12.Statements Some lakes are ponds Some ponds are rivers. Conclusions I. No river is a lake.

II. Atleast some rivers are lakes. (a) Either conclusion I or II is true (b) Neither conclusion l nor II is true (c) Only conclusion l is true (d) Both conclusions I and II are true 13.Statements All leaves are flowers No flower is a branch. Some branches are trees. Conclusions l. Some trees are leaves. II. No tree is a flower. (a) Either conclusion I or Ills true (b) Neither conclusion I nor II is true (c) Only conclusion I is true (d) Both conclusions I and ll are true 14. Statements All pins are staplers. Some staplers are rulers. Conclusions I. Some pins are rulers. ll. All rulers are pins. (a) Either conclusion l or II is true (b) Neither conclusion I nor II is true (c) Only conclusion I is true (d) Both conclusions I and II are true 15. Statement All mouses are cursors. All cursors are pointers. Conclusions I. All mouses are pointers. II. All pointers are cursors. (a) Either conclusion I or II is true (b) Neither conclusion I nor II is true (c)Only conclusion I is true (d) Both conclusion l and H are true Directions (Q. Nos. 16-21) In this question are given four statements followed by five conclusions, one of which definitely does not logically follow (or is not a possibility of occurrence) from the given statements. That conclusion is your answer. Note You have to take the four given statements lo be true even if they seem to be at variance with commonly known facts and then decide which of the given conclusions logically does not follow from the given statements disregarding commonly known facts. 16. Statements

No toy is a doll. All guns are toys.

CLAtutor Logical Reasoning 33

All houses are dolls. All dolls are baskets. Conclusions (a) All baskets are toys (b) No gun is a house (c) All guns being baskets is a possibility (d) All houses are baskets 17.Statements Some logics are reasons All reasons are arguments All arguments are fights No fight is a discussion Conclusions (a) All discussions being logic is a Possibility (b) No discussion is an argument (c) All logics being discussion is a Possibility (d) All reasons are fights 18. Statements All references are mails AII mentions are references All comments are mentions No mail is a declaration' Conclusions (a) No reference is a declaration (b) Ail comments are mails (c) No mention is a declaration (d) All declarations being comments is a possibility 19. Statements

Some moments are flashes All moments are seconds All flashes are instances No instance is an hour. Conclusions (a)All hours being seconds is a possibility (b) No second is a flash (c) No hour is a flash (d) Atleast some moments are instances 20. Statements All circles are spheres. All spheres are rectangles Conclusions No rectangle is a pyramid. No pyramid is a triangle.. Conclusions (a)Atleast some circles are pyramids (b)All .triangles being circles is a possibility (C)All rectangles being triangles is possibility (d)Atleast some rectangles are circles

21, Statements

No hotel is a motel. All motels are apartments. AII apartments are inns. No inn is a guesthouse. Conclusions (a) All hotels being inns is a possibility (b) No motel is a guesthouse (c) All hotels being apartments is a possibility (d) No motel is an inn Directions (Q. Nos. 22-24) In these questions two/three statements followed by two conclusions numbered I and II have been given. You have to take the given statements to be true even, if they seem to be at variance from commonly known facts and then decide which of the given conclusions logically follows from the given statements disregarding commonly known facts. 22. Statements

All kites are birds. No bird is an animal. All animals are clouds. Conclusions I. Atleast some birds are clouds. II. All clouds being birds is a possibility. (a) Only conclusion II is true (b) Neither conclusion I nor II is true (c) Both conclusions I and II are true (d) Either conclusion I or II is true 23. Statements

Some forces are energies. All energies are powers. All powers are strengths. Conclusions I. Atleast some forces are strengths. II. AII energies are strengths.. (a) Only conclusion II is true (b) Either conclusion I or II is true (c) Only conclusion I is true (d) Both conclusions I and II are true 24. Statements

Some circles are rectangles. All squares are rectangles. Conclusions I. Atleast some squares are circles. ll. All rectangles are circles. (a) Only conclusion I is true (b) Either conclusion I or II is true

CLAtutor Logical Reasoning 34

(c) Both conclusions land II are true (d) Neither conclusion I nor II is true 25. Some statements are given followed by two Conclusions I and II. You have to consider the statements to be true, even if they seem to be at variance from commonly known facts. You are to decide which of the given conclusions can definitely be drawn from the given statements. Indicate your answer. Statements All files are papers. Some papers are books. Some books are journals. Conclusions I. AII files are journals. II. All books are not journals. (a) Both conclusions i and II follows. (b) Neither conclusion I nor II fo1lows. (c) Only conclusion I follows. (d) Only conclusion II follows.

Instruction for Question 26-28: Mark (a) Conclusion I is true (b) Conclusion II is true (c) Either I and II are true (d) Neither conclusion I nor II is true (e) Both I and II follow 26. Statement : All pilots are brave men. All astronauts are pilots. Conclusion:

I.All astronauts are brave men. II.All pilots are astronauts.

27. Statement : All hill stations have a sunset point. X is a hill station. Conclusion : I.X has a sunset point. II. Places other than hill stations do not have sunsets. 28. Statement: Some dancers are poets Some poets are singers Conclusion :

I.Some poets are not singers. II. Some dancers are not poets.

Directions (Q. Nos. 29-30) In the given questions two statements are given followed by two conclusions I and

II. You have to consider the statements to be true, even if it seems to be at variance from commonly known facts. You are to decide which of the given conclusions can definitely be drawn from the given statements. Indicate your answers. 29. Statements

I. Meena is older than Roma. II. Rashmi is older than Meena III. Roma is older than Rashmi. If 1st two statements are true 3rd statement is (a) False (b) Uncertain (c) Rogue (d) True 30. All athletes are brave. All girls are athletes. Conclusions I. All girls are brave. II. Some athletes are girls. (a) Only Conclusion II follows. (b) Both Conclusions I and II follow. (c) Neither Conclusion I nor II follows. (d) Only Conclusion I follows. Directions (Q. Nos. 31-35) Mark: (a) Conclusion I is true (b) Conclusion II is true (c) Either I and II are true (d) Neither conclusion I nor II is true (e) Both I and II follow 31. Statement : Some hens are cows All cows are horses Conclusions: I.Some horses are not hens II.Some hens are horses 32. Statement : Some bricks are sticks. No sticks are red. Conclusions: I.No brick is red. II.Some brick are not red 33. Statement : All poets are readers. No reader is wise. Conclusions: I.No poet is wise. II.All readers are poets.

CLAtutor Logical Reasoning 35

34. Statement : All stones are diamonds. Some diamonds are pearls. Conclusions: I.All pearls are stones. II.All diamonds are pearls. 35. Statement : Alcoholic drinks are injurious to health. All old women drink whisky Conclusions: I.All old women have poor health. II.All yound women are in good health. Directions (Q. Nos. 36-37)Three statements are given followed by two/four conclusions I, II III and, IV. You have to consider the three statements to be true even if they seem to be at variance from commonly known facts. you have to decide which of the given conclusions, if any, follow from the given statements. 36. Statements

1. Rabindranath Tagore wrote many poems. 2. Every poet has aesthetic knowledge. 3. Aesthetic is a part of axiological study. Conclusions I. Rabindranath Tagore did different axiological study. II. He followed the base of logic and ethics. (a) Only conclusion I (b) Both conclusions I and II (c) Only conclusion II (d) None of the above 37. Statements

1. All clerks are superintendents. 2. All superintendents are managers. 3. All managers are supervisors. Conclusions I. All supervisors are clerks. II. Some clerks are supervisors. III. Some managers are clerks. IV. All superintendents are clerks. (a) Only conclusion I (b) Only conclusion II (c) Only conclusion III (d) Only conclusion IV Directions (Q. Nos. 38-40) some statements are given followed by three / two conclusions / assumptions, respectively. You have to consider the statements to be true even if they seem to be at variance from commonly known facts. You have to decide which of the given

conclusions /assumptions if any, follow from the given statements. 38.Statement Pictures can tell a story. All story books have pictures. Some story books have words. Conclusions I. Pictures can tell a story better than words can. II. The stories in story books are very simple. III. Some story books have both words and pictures. (a) Only Conclusion I follows (b)Only Conclusion II follows (c) Only Conclusion III follows (d) Only Conclusions I and II follow, 39. Statement. Some merchants are customers. Some customers are employees. Some employees are owners. Conclusions: I. Some owners are merchants. II. Some customers are merchants. III. Some employees are merchants. IV. Some owners are customers. (a) Only II follows (b) Only I & II follows (c) III & IV follow (d) Only I & IV follow. 40. Statement. All myths are fictions. No fiction is novel. All novels are stories. Conclusion. I. No myth is novel. II. Some fictions are novels. III. Some fictions are myths. IV. Some myths are novels. (a) All follow. (b) Either I or II and Both III & IV (c) Either I or IV and II follow (d) Both I and III follow 41. I. Apples cost more than oranges. II. Apples cost less than mangoes. III. Mangoes cost more than apples and oranges. If the 1st two statements are true, then third is (a) True (b) False (c) Uncertain (d)Vague Direction (Q. No. 42) One statements is given followed by two conclusions I and II. You have to consid.er the statement to be true, even if it seems to be at variance from commonly known facts. You are to decide which of the given conclusions can be definitely drawn from the given statement. Include your answer

CLAtutor Logical Reasoning 36

42. Most Indians are aware that they have a great heritage, but few would include science in it. Conclusions I. Many Indians consider science to have made Indian heritage great. II. Many Indians are not aware that India has a great scientific heritage. (a) Only Conclusion I follows (b) Only Conclusion II follows (c) Both Conclusions I and II follow (d) Neither Conclusion I nor II follows Directions (Q. Nos. 43-44) In the following questions two statements are given. followed by two conclusions I and II. You have to consider the statements to be true even if they seem to be at variance from commonly known facts. You have to decide which of the given conclusions, if any, follows from the given statements. Indicate Your answer. 43. Statements 1. All students in my class are intelligent. 2. Sunil is not intelligent. Conclusions I. Sunil is not a student of my class. II. Sunil must work hard (a) Only Conclusion I follows (b) Only Conclusion II follows (c)Both Conclusions I and II follows (d)Neither Conclusion I nor II follows 44. Statements

1. Education is a process of lighting 2. Mind requires light to enlighten the core of cognitive aspect. Conclusions I. Education is light which the darkness of mind. II. Education is a static mind (a) Only Conclusion I follows (b) Only Conclusion II follows (c) Both Conclusions I and II follow (d) Neither Conclusion I nor II follows Directions (Q. Nos. 45-46) following question one/two statements are given followed by two / three conclusions I, II and III. You have to consider the statement to be true even if they seem to be at variance from commonly known facts. You have to decide which of the given conclusion, if any, follow from the given statements. 45. Statement Sick People need medicine.

Conclusions I. Healthy people do medicine. II. People keep medicine in their home. (a) Only Conclusion I follows (b) Only Conclusion II follows (c) Both Conclusions I and II follow (d) Neither Conclusion I nor II follows 46.Statements I. Some years are decades. II. All centuries are decades. Conclusions I. Some centuries are years. Il. Some decades are years. III. No century is a year. (a)Conclusions I and II follow (b)Neither Conclusion I nor II follow (c)Only Conclusion I follows (d) Only Conclusion II follows Direction (Q. No. 47) One statement is given, followed by two Conclusions I and II. You have to consider the statement to be true even if it seems to be at variance from commonly known facts. You have to decide which of the given conclusion, if any follows from the given statements. Indicate your answer. 47. Statement Medals are awards. Conclusions I. All awards are not medals. II. Al1 medals received are called awards. (a)Only Conclusion I follows (b)Only Conclusion II follows (c)Both Conclusions I and II follow (d)Neither Conclusion I nor II follows 48. Two statements are given followed by two inferences I and II. You have to consider the statements to be true even if they seem to be at variance from commonly known facts. You are to decide which of the given inferences, if any follow from the given statements. Indicate your answer. Statements 1. All players are educated. 2. All educated are cultured. Inferences I. All players are cultured. II. All cultured persons are players. (a) Only inference I follows (b) Only inference II follows (c) Both inference I and II follow (d) Neither inference I nor II follows

CLAtutor Logical Reasoning 37

49. Two statements are given followed by two inferences I and II. You have to consider the statements to be true even if they seem to be at valiance from commonly known facts. You are to decide which o{ the given inferences, if any follows from the given statement. Indicate your answer. Statements 1. All animals are dogs. 2. All dogs are birds. Conclusions I. All animals are birds. II. All birds are animals. (a) Only Conclusion I follows

(b) Only Conclusion II follows (c) Both Conclusions I and II follow (d) Neither Conclusion I nor II follows 50. Statements l. Some books are mobiles. 2. Some calculators are mobiles. Conclusions I. Some mobiles are calculators. II. Some mobiles are books. (a) Only Conclusion I follows (b) Only Conclusion II follows (c) Both Conclusions I and II follow (d) Neither Conclusion I nor II follows

CLAtutor Logical Reasoning 38

Direction & Blood Relationship

1. Raju was to go to the planetarium. So he walked 1.5 km towards East from the place and then turned to right and walked 2.5 km and then turned towards East walked 1 km and turned to South and walked 4 km and reached the place by walking 2.5 km towards West. What distance is he from the starting point? (a) 6.5 km (b) 9.5 km (c)10 km (d) 9 km 2. A cyclist rides 40 km to the East, turns North and rides 20 km, again turns left and rides 20 km. How far is he from the starting point?

(a) 0 km (b) 10 km (c)20√2km (d)30km 3. One morning Savita set out from home to fetch water in the direction of the Sun. After some time she turned to her left and then to her right. After some time she turned to her right again. Now, which direction is she facing? (a) South (b) West (c)East (d) North 4. Jaffar starting from a fixed point goes 15 km towards North and then alter turning to his right he goes 15 km. Then, he goes 10, 15 and 15 km after turning to his left each time. How far is he from his starting point? (a) 10 km (b)20 km (b) 15km (d) 5 km 5. Going 40 m to the South of his house, Ramdev turns left and goes another 10 m. Then, turning to the North, he goes 20 m and then starts walking to his house. In which direction is he walking now? (a) North-West (b) North-East (c) South-East (d) South-West 6. Ramesh walks 2 km to West and turns to his right and walks 3 km and turns right and walks 2 km and finally turns to his right. Which direction is he facing? (a) East (b) West (c) North (d) South 7. Pratap starts from school and walks 7 km towards East. He takes a left and walks 4 km, then takes a right and walks Z km, again takes a right and walks 3 km. Which direction is he facing now? (a) South (b)North (c) East (d) West 8. Vijay travelled 12 km Southward, then turned right and travelled 10 km, then turned right and travelled 12 km. How far was Vijay from the starting point? (a) 22 km (b) 44 km (c) 12 km (d) 10 km

9. A person walks towards his house at 8:00 am and observes his shadow to his right. In which direction he is walking? (a)North (b) South (c) East (d) West 10.A boat moves from a jetty towards East. After sailing for 9 nautical miles, she turns towards right and covers another 12 nautical miles. If she wants to go back to the jetty, what is the shortest distance now from her present position? (a) 21 nautical miles (b) 20 nautical miles (c) 18 nautical miles (d) 15 nautical miles 11.A cyclist goes straight 7 km Eastwards, then turns right and goes straight 3 km and turns right again and goes straight 10 km. In which direction is he from the starting point? (a) South-West (b) North-West (c) North-East (d) South-East 12. A man walks 20 in towards East and then he turns to South and walks 50 m. Again, he turns to East and walks 30 m. After this he turns to North and walks 170 m. Now, how far is he from his starting point? (a)130m (b)150 m (c) 140m (d) 160 m 13. Ram and Sam start walking towards North and cover 20 m. Ram turns to his left and Sam to his right. After sometime, Ram walks 10 m in the same direction in which he turned. On the other hand, Sam walks only 7 m. Later, Ram turns towards his left and Sam to his right. Both walk 25 m forward. How far is Ram from Sam now? (a) 10m (b) 20m (c) 17m (d)5m 14. A man travels 4 km due North, then travels 6 km due East and further travels 4 km due North. How far he is from the starting point? (a) 6 km (b) 14 km (c) 8 km (d)10 km 15. A direction pole was situated on the road crossing. Due to an accident, the pole turned in such a manner that the pointer which was showing East, started showing South. Sita, a traveller went to the wrong direction thinking it to be West. In which direction actually she was travelling? (a) North (b) West (c) East (d) South

CLAtutor Logical Reasoning 39

16.A man starts from his house and walked straight for 10 m towards North and turned left and walked 25 m. He then turned right and walked 5 m and again turned right and walked 25 m. Which direction is he facing now ? (a) North (b) East (c) South (d) West 17. A house faces North. A man coming out of his house walked straight for 10 m, turned left and walked 25 m. He then turned right and walked 5 m and again turned right and walked 25 m. How far is he from his house? (a) 15m (b)55m (c)60m (d)65m 18. Kiran moved a distance of 50 m towards the North. He then turned to the left and walked 25 m, turned left again and walked 40 m. Finally he turned to the right. In which direction is he moving now ? (a) West (b) East (c) South (d) North 19. John’s house is 100 m North of his uncle's office. His uncle's house is located 200 m West of his (uncle's ) office. Kabir is the friend of John and he stays 100 m East of John's house. The office of Kabir is located 100 m South of his house. Then, how far is his uncle’s house from Kabir's office? (a)200m (b)300m (c)400m (d)500m 20. A boy walks Northwards. After a while he turns towards his right and a little further to his left. Finally after walking a distance of 1 km, he turns to his left again. In which direction he is moving now ? (a) North (b) South (c) East (d) West 21. Ganesh is standing at a point. He walks 20 m towards the East and further 10 m towards the South, then he walks 35 m towards the West and further 5 m towards the East. What is the straight distance in metres between his starting point and the point where he reached last? (a) 20 m (b) 5 m (c) 10m (d) 35m 22. A man travels 3 km in the East and turns to South and moves 4 km. How far is he from the starting point? (a) 5 km (b) 6 km (c) 2 km (d) 10 km 23. A cyclist goes 30 km to North and then turning to East he goes 40 km. Again, he turns to his right and goes 20 km. After this he turns to his right and goes 40 km. How far is he from his starting point? (a) 20 km (b) 10 km (c) 25 km (d) 40 km

24. Ram starts from his house and walks 4 km North, then 3 km West, then 8 km South. How many kilometers away from his home was he? (a) 6km (b) 7km (c) 5km (d) 8km 25. Khan travels 8 km from East to West and turns right to travel 2 km. Now, he turns right and travels 5 km. In which direction is he now positioned with reference to the starting point ? (a) South-East (b) North-West (c) West (d) North Directions (Q. Nos. 26-27) Study the following information

and answer the given questions.

R is married to U.U is the mother of I.L is the sister of D.

U has only one daughter. D is married to J. K is the son

of J. F is the mother of J.

26. How is D related to F?

(a) Cannot be determined (b) Daughter

(c) Daughter-in-law (d) Son-in-law (e) Son

27. How is R related to K?

(a) Cannot be determined ' (b) Father-in-law

(c) Grandmother (d) Father (e) Uncle

Directions (Q.No.28-30) Study the following information

and answer the given questions.

T is the sister of D. D is married to P. P is the son of M.

T is the mother of J. Y is the father of U. Y has only one

son and only one daughter. U is the daughter of T. Q is

the son of D.

28. How is P related to T?

(a) Brother (b) Cannot be determined

(c) Brother-in-law (d) Cousin brother

(e) Uncle

29. How is I related to D?

(a) Son (b) Niece (c) Son-in-law

(d) Nephew (e) Daughter

30. How is Q related to M?

(a) Son-in-law (b) Grandson (c) Nephew

(d) Son (e) Cannot be determined

Directions (Q.No.31-33) Study the following information

and answer the given questions.

S is the sister of B. K is the brother of B.

CLAtutor Logical Reasoning 40

K is the son of L. L is married to Y.

Y is the daughter of J.

Y has only one daughter. J is married to C.

K is the brother of D. D is married to T.

31. If P is the brother of C, the related to P?

(a) Brother (b) Cousin (c) Sister

(d)Uncle (e) Cannot be determined

32. How is T related to S?

(a) Brother-in-law ( b) Cousin (c) Sister-in-law

(d) Sister (e) Brother

33. If Z is the daughter of T, then how is B related to Z?

(a) Uncle (b) Father (c) Aunt

(d) Cannot be determined (e) Mother

34. R is uncle of A. R is grandfather of Q.Q's brother

name is P, then what is relation of P to A ?

(a) Brother (b) Grandfather (c) Father

(d) Uncle (e) None of these

35.Pointing to a girl, Subodh said, "She is the daughter of

my mother only brother". How is Subodh related to the

girl ?

(a)Cousin brother (b) Maternal uncle

(c)Brother (d) Data inadequate

(e)None of these

36. Pointing to a boy Sunita said, “He is the son of my

grandfather’s only son”. How is boy related to Sunita?

(a)Son (b)Nephew (c)Uncle

(d)Data inadequate (e)None of these

37. Pointing to a boy in a photograph Sudhir said, "He is

the son of my maternal grandfather's only child”. How

is the boy related to Sudhir?

(a) Self (b) Brother

(c) Cousin brother (d) Data inadequate

(e) None of these

38. Pointing to a girl, Nidhi said, “She is the daughter of

my grandmother's only child". How is the girl related to

Nidhi?

(a) Sister (b) Self (c) Cousin Sister

(d) Data inadequate (e) None of these

39. D said, “A's father is the only brother of my sister’s

son. ”How is A’s father related to D?

(a) Cousin (b) Nephew (c) Aunt

(d) Data inadequate (e)None of the above

40. Pointing to a woman, Nirmal said, "She is the

daughter of my wife’s grandfather's only child”. How is

the woman related to Nirmal ?

(a) Wife (b) Sister-in-law

(c) Sister (d) Data inadequate

(e) None of these

41. K is brother of T.M is mother of K. W is brother of M.

How is W related to T?

(a) Maternal uncle (b) Paternal uncle

(c) Grandfather (d) Data inadequate

(e) None of the above

42. M is sister of K. D is brother of K. F is mother of M.

How is K related to F?

(a) Son (b) Daughter (c) Son or Daughter

(d) Data inadequate (e) None of these

43.Nandini is the only daughter of Madan’s sister,

Sangita's brother. How is Nandini related to Madan ?

(a) Daughter (b) Niece (c) Cousin

(d) Niece/Daughter (e) None of these

44. R is sister of M who is brother of H. D is mother of K

who is brother of M. How is R related to D ?

(a) Sister (b) Daughter (c) Mother

( d) Data inadequate (e) None of these

45. B is brother of D. D is sister of E. E is brother of F.

How is F related to B?

(a) Brother (b) Sister (c) Brother or Sister

(d) Data inadequate (e) None of the above

46. D, the son-in-law of B is the brother-in-law of A who

is the brother of C. How is A related to B?

(a)Brother (b) Son (c) Father

(d) Data inadequate (e) None of these above

Directions (Q. Nos. 47-48) Mohan is son of Arun's father's sister' Prakash is son of Reva who is mother of Vikash and grandmother of Arun. Pranab is father of Neela and grandfather of Mohan. Reva is wife of Pranab. 47. How is Mohan related to Reva?

CLAtutor Logical Reasoning 41

(a) Grandson (b) Son (c) Nephew (d) Data inadequate (e) None of the above 48. How is Vikash’s wife related to Neela ? (a) Sister (b) Niece (c) Sister-in-law (d)Data inadequate (e) None of the above 49. Introducing Sarita, Meena said. “She is the only daughter of my father’s only daughter". How is Meena related to Sarita ? (a) Niece (b) Cousin (c) Aunt (d) Data inadequate (e) None of the above Directions (Q. No. 50) Read the following information carefully and answer the questions which follow: 'Ax B 'means A is son of B'. 'A+ B' means .A is father of B'. 'A> B' means ’A is daughter of B'. 'A<B' means 'A is wife of B'' 50. Which of the following pairs of persons represent first cousins with regard to the relations given in the expressions, if it is provided that A is the sister of J? 'L> V< J+ P' and 'S x A< D+ F< E + K' ? (a)LP (b)SP (c) SK (d)SF (e) Cannot be determined

CLAtutor Logical Reasoning 42

Sitting Arrangement & Puzzle

Directions (Q. Nos. 1-5) Study the given information carefully and answer the given questions. Ten people are sitting in two parallel rows containing five people each, in such a way that there is equal distance between adjacent persons. In row-1 J,K, L, M and N are seated (not necessarily in the same order) and all of them are facing south. In row-2 V, W, X, Y and Z and seated ( not necessarily in the same order) and all of them are facing north. Therefore in the given seating arrangement each member seated in a row faces another member of the other row. Z sits third to the right of W. V sits second to the left of Z. The persons facing V sits to the immediate right of K. Only one person sits between K and M. J is not an immediate neighbour of K. Only to people sit between J and L. Neither K nor J faces Y. 1. Who amongst the following is facing N ? (a) Y (b)Z (c)V (d)X (e) W 2. Which of the following statements is true regarding M? (a) M faces one of the immediate neighbours of X (b) K is one of the immediate neighbours of M (c) None of the given statements is true (d) L sits to the immediate right of M (e) Only one person sits between M and N 3. Who amongst the following is facing X? (a) K (b)L (c)M (d)J (e) N 4. What is the position of Z with respect to Y? (a) Third of the right (b) Second to the right (c) Immediate left (d) Immediate right V (e) Second to the left 5. Four of the given five are alike in a certain way based on the given arrangement and hence form a group. Which of them does not belong to that group? (a) M (b)J (c)N (d)W (e)N Directions (Q. Nos. 6-10) Study the following information and answer the questions. Seven friends, namely A, B, C, D, E, F and G are standing in a straight line facing North but not necessarily in the same order. D stands exactly in the centre of the line. Only two persons stand between D and F. A stands to the immediate left of C.

A stands to the right of D (may or may not be the immediate right). Only one person stands between A and B. G stands second to the left of B. 6. Which of the following is true regarding E? (a) F stands second to the right of E (b) E stands third to the left of C (c) Only one person stands between E and A (d) E is an immediate neighbour of G (e) E stands at one of the extreme ends of the line 7. Four of the following five form a group as per the given arrangement. Which of the following does not belong to that group? (a)FD (b)DG (c)GE (d) CB (e) EA 8. What is the position of C with respect to F ? (a) Second to the right (b) Second to the left (c) Third to the left (d) Immediate left (e) Immediate right 9. Which of the following pairs stand at the extreme ends of the line? (a) B, F (b) G,F (c) A, G (d) E,F (e) B, E 10. Which of the following represents the position of B from the left end of the line? (a) Fifth (b) Fourth (c) First (d) Third (e) Sixth Directions (Q. Nos. 11 to 15) Study the following information to answer the given questions. Six people — C, D, E, F, G and H are standing in a straight line facing North not necessarily in the same order. D is standing second to the right of F. C is standing fourth to the left of H and H is not standing on the extreme end of the line. E is standing second to the right of D. 11. What is the position of G with respect to E? (a) Immediate left (b) Second to the left (c) Third to the left (d) Third to the right (e) None of these 12. Which of the following pairs represents the people standing at the extreme ends of the line ? (a) FH (b)CE (c) DE (d) CH (e) None of these

CLAtutor Logical Reasoning 43

13. Who is standing second to the right of C ? (a) F (b) D (c) G (d) E (E) None of these 14. Four of the following five are alike in a certain way based on their positions in the above arrangement and so form a group. Which of the following does not belong to the group? (a) CG (b) GE (c) GH (d) DE (e) FD 15. If all the people are asked to stand in an alphabetical order from left to right, the positions at how many will remain unchanged? (a) One (b) Two (c) Three (d) None (e) Can’t be determined Directions (Q. Nos. 16-19) Study the following information to answer the given questions. Seven friends, T, U, V, W, X, Y and Z are sitting in a straight line facing North. W sits fifth to the right of T. W does not sit at any of extreme ends. Two people sit between Z and X. Y sits third to the left of U. Y sits exactly in the middle. Z is not an immediate neighbour of Y. 16. What is Z's position with respect to W ? (a) Second to the left (b) Third to the right (c) Fourth to the left (d) Third to the left (e) Fourth to the right 17. Who is second to the right of T? (a) Y (b) X (c) U (d) V (e) None of these 18. Four of the following five are alike in a certain way based on their seating positions in the above line and so form a group. Which is the one that does not belong to the group? (a)UW (b)XV (c)ZT (d)YV (e) WX 19. If all the seven friends are made to sit alphabetically from right to left, positions of how many will remain unchanged? (a) None (b) One (c) Two (d) Three (e) Four Directions (Q. Nos. 20-24) Study the following information to answer the given questions. P, Q, R, S, T, V, X and Y are seated in a straight line facing North. P sits fourth to the left of V. V sits either sixth from the left end of the line or fourth from the right end of the line. S sits second to right of R. R is not an immediate neighbour of V. T and Q are immediate neighbours of each other but neither T nor Q sits at extreme ends of the line.

Only one person sits between T and X. X does not sit at the extreme end of the line. 20. What is the position of Q with respect to P? (a) Fifth to the right (b) Immediate neighbour (c) Second to the right (d) Third to the left (e) None of the above 21. Which of the following represents persons seated at the two extreme ends of the line? (a) P, V (b) Y, S (c) R, V (d) Y, P (e) R, Y 22. How many persons are seated between R and T? (a) One (b) Two (c) Three (d) Four (e) None of these 23. If P is related to Q and S is related to T in a certain way, to which of the following would V be related to following the same pattern? (a) Y (b) P (c) R (d) S (e) X 24. Who amongst the following sits exactly in the middle of the persons who sit second from the left and the person who sits fifth from the right? (a) V (b)Q (c) T (d) S (e) P 25. Five friends A, B, C, D and E are standing in a row facing south but not necessarily in the same order. Only B is between A and E, C is immediate right to E and D is immediate left to A. On the basis of above information, which of the following statements is definitely true? (a) B is to the right of E (b) A is second to the left of C (c) D is third to the left of E (d) B is to the left of A (e) None of the above Directions (Q. Nos. 26-30) Study the following information and answer the questions. Seven people P, Q, R, S, T, U and V have a seminar but not necessarily in the order, on seven different months (of the same year) namely January, February, March, June, August, October and December. Each of them also likes a different fruit namely Banana, Grapes, Papaya, Orange, Mango, Litchi and Apple but not necessarily in the same order. R has a seminar in a month which has less than 31 days. Only two people have a seminar between R and S. The one who likes Banana has a seminar immediately before S. Only one person has a seminar before the one who likes Papaya. Q has a seminar immediately after the one who likes Papaya. Only three people have a seminar between Q and the one who likes Mango.

CLAtutor Logical Reasoning 44

T likes neither Mango nor Papaya. P has a seminar immediately before T.V likes Apple. The one who likes Grapes has a seminar in the month, which has less than 31 days. The one who has ,a seminar in March does not like Orange. 26. Which of the following represents the month in which S has a seminar? (a) January (b) Cannot be determined (c) October (d) December (e) June 27. Which of the following represents the people who have a seminar in January and June, respectively? (a) V,S (b) U,S (c) Q, T (d) V, R (e) U,R 28. How many people have a seminar between the months in which V and R have a seminar? (a) None (b) Two (c) Three (d) One (e) More than three 29. As per the given arrangement, R is related to Banana and Q is related to Orange following a certain pattern, which of the following is V related to following the same pattern? (a) Mango ( b) Litchi (c) Apple (d) Papaya (e) Grapes 30. Which of the following fruits, does U like? (a) Orange (b) Papaya (c) Mango (c) Banana (e) Grapes Directions (Q. Nos. 31-33) Read the given information carefully and answer the questions. Each of the six buildings P, Q, R, S, T and R.U houses different number of offices. S has more offices than R. Q has more number than P but less than U. R does not house the least number of offices. The building which houses the least number of offices has 5 offices. The building which has second highest number of offices has 23 offices. S has 11 less number of offices than Q. 31. Which of the following building has the second least number of offices? (a) Q (b) U (c) R ( d) P (e) T 32. If the number of offices in P is an even number which is divisible by 2 as well as 3. How many does P have? (a) 20 (b)24 (c)16 (d)18 (e)12 33. Which of the following is the number of offices in building R? (a) 25 (b) 12 (c) 13 (d) 14 (e) 11 Directions (Q. Nos. 34-38) Read the given information

carefully to answer the given questions.

J, K, L, M, O, Q, R and S are sitting around a circular table

facing the centre with equal distances between each other

(but not necessarily in the same order). Each one of them is

also related to M in some way or the other. Only two

people sit between Q and L. M sits second to the left of Q.

Only three people sit between L and M’s sister. M’s son sits

second to the right of M’s sister. Only one person sits

between M’s son and S. J sits to the immediate right of R. R

is neither the son nor the mother of M.S is an immediate

neighbour of M’s mother. Only three people sit between

M’s mother and M’s brother. M’s daughter sits second to

the left of M’s brother. M’s father is not an immediate

neighbour of M. M’s wife sits third to the right of K.

34. Who sits second to the right of R?

(a) M’s brother (b) M (c) S (d) K

(e) M's daughter

35. How many people sit between K and L, when counted

from the left of L?

(a) Six (b) One (c) None (d) Two (e) Four

36. Which of the following statements is true with respect

to the given information?

(a) R sits second to the right of M's wife

(b) K is an immediate neighbour of R

(c) M sits second to the left of L

(d) All the given options are true

(e) S is the daughter of L

37. How is K related to R?

(a) Son-in-law (b) Uncle (c) Niece

(d) Brother (e) Daughter

38. Who amongst the following is the wife of M?

(a) R (b) L (c) O (d) Q (e)J

Directions (Q. Nos. 39-43) Study the given information

carefully to answer the given questions.

J, K, L, M, N, O and P live on seven different floors of a

building but not necessarily in the same order. The lower

most floor of the building is numbered 1, the one above

that is numbered 2 and so on till the topmost floor is

numbered 7. Each one of them also likes a different subject

namely— English, History, Commerce, Biology, Accounts,

Geography and Computer, (but not necessarily in the same

order.)

CLAtutor Logical Reasoning 45

• J lives on an odd numbered floor but not on the floor

numbered 3.

• The one who like Accounts lives immediately above J.

Only two people live between M and the one who likes

Accounts.

• The one who likes History lives on one of the odd

numbered floors above M. Only three people live

between L and the one who likes History. The one who

likes Commerce lives immediately above L.

• The one who likes English lives immediately above the

one who likes Computer. P lives on an odd numbered

floor.

• Only one person lives between K and N. K lives on one of

the floors above N. J does not like Biology. N does not

like Commerce.

39. Which of the following subjects does J like? (a) Geography (b) Computer (c) Commerce (d) History (e) English

40. Which of the following combinations is true with respect to the given arrangement? (a) Geography-L (b) History-O (c) Computer-P (d) Accounts-M (e) Commerce-K

41. If all the people are made to sit in alphabetical order from top to bottom, the positions of how many people will remain unchanged? (a)Four (b) None (c)Two (d) One (e) Three 42. Which of the following statements is true with respect to the given arrangement? (a) The one who likes Computer lives immediately below J (b) O likes History (c) None of the given options is true (d) Only four people live between M and P (e) P lives immediately below N

43. Who amongst the following lives on the floor numbered 6? (a) K (b) The one who likes English

(c)The one who likes Computer (d) M (e) O

Directions (Q. Nos. 44-49) Study the following information

and answer the question.

K, L, M, N, O, P, Q and R live on eight different floors of a

building but not necessarily in the same order. The lower

most floor of the building is numbered one, the one above

that is numbered two and so on till the topmost floor is

numbered eight. Each of them also lives a different

superhero namely, Batman, Superman, Captain America,

Thor, Hulk, Wolverine, Nova and Superman, but not

necessarily in the same order.

• The one who likes Thor lives on an even numbered

floor. Only three people live between the one who likes

Thor and M.

• Only two people live between M and N. N does not live

on the lower most floor. Only three people live between

N and the one who likes Hulk.

• O lives immediately above K. O lives on an even

numbered floor. K does not like Hulk. K lives neither on

floor numbered three nor five.

• Only two people live between K and the one who likes

Nova.

• Only one person lives between the one who likes Nova

and Ironman. The one who likes

Ironman lives below the one who likes Nova.

• L lives immediately above Q. Only one person lives

between Q and the one who likes Captain America.

• The one who likes Wolverine lives immediately above

the one who likes Batman.

• P does not like Thor. K does not like Superman.

44. Which of the following pairs represent those who live

immediately above and immediately below N?

(a) R, L (b) Other than those given as options

(c) L, M (d) K, Q (e) R, Q

45. Four of the following five arc alike in a certain way as

per the given arrangement and so form a group. Which of

the following does not belong to that group?

(a) K- Floor numbered five

(b) Q- Floor numbered six

(c) L-Floor numbered two

(d) N-Floor numbered seven

(e) R-Floor numbered eight

46. Which of the following superheroes does R like? (a) Other than those given as options (b) Superman (c) Nova (d) Thor (e)Batman 47. Which of the following superheroes does P like? (a) Ironman (b)Wolverine (c) Batman (d)Captain America (e) Other than those given as

CLAtutor Logical Reasoning 46

48. N lives on which of the following floor numbers? (a) Five (b) Three (c) Seven (d)One (e) Other than those given as 49. As per the given arrangement, P is related to Nova and N is related to Wolverine in a certain way. To which of the following is M related to in the same way? (a) Superman (b)Batman (c) Captain America (d)Thor (e) Hulk

50. 794 736 782 775 718

If the position of the 1st and 2nd digit of each number is

interchanged, which of the following will now be the 3rd

digit of the 2nd lowest number?

(a)2 (b)4 (c)5 (d)6 (e)8

CLAtutor Logical Reasoning 47

Decision making, Input and Output

Decision making

Directions: (Q. Nos. 1-5) Study the following information

carefully and answer the questions given below.

Following are the conditions for selecting Manager HR

in an organisation.

The candidate must

(i) be atleast 30 yr and not more than 35 yr as on 1.

3.2012.

(ii) have scored atleast 60% marks in graduation in any

discipline.

(iii) have scored atleast 65% marks in the post graduate

Degree/Diploma in Personnel Management/HR.

(iv) have post qualification work experience of atleast 5

yr in the Personnel/HR Department of an organisation.

(v) have scored atleast 50% marks in the selection

process.

In the case of a candidate who satisfies all the above

conditions except

(A) (ii) but has scored atleast 55% marks in graduation in

any discipline and atleast 70% marks in post graduate

Degree/Diploma in Personnel Management/HR, the

case is to be referred to GM-HR.

(B) (iv) but has post qualification work experience of

atleast 4 yr, out of which atleast 2 yr as Deputy Manager

HR, the case is to be referred to President-HR.

In each question below are given details of one

candidate. You have to take one of the following courses

of actions based on the information provided and the

conditions and sub-conditions given above and mark the

number of that course of action as your answer. You are

not to assume anything other than the information

provided in each question. All these cases are given to

you as on 1.3.2012.

Give Answer

(a) if the candidate is not to be selected

(b) if the data provided are not adequate to take a

decision

(c) if the case is to be referred to President-HR means

condition follows

(d) if the case is to be referred to GM-HR

(e) if the candidate is to be selected

1. Rita Bhatt was born on 25th July 1978. She has scored

62% marks in graduation and 65% marks in post

graduate Diploma in Management. She has been

working for the past 6 yr in the Personnel Department of

an organization after completing her post graduation.

She has scored 55% marks in the selection process.

2. Ashok Pradhan was born on 8th August 1980. He has

been working in the Personnel Department of an

organization for the past 4 yr after completing his post

graduate Degree in Personnel Management with 67%.

Out of his entire experience, he has been working for the

past 2 yr as Deputy Manager-HR. He has scored 62%

marks in graduation and 58% marks in the selection

process.

3. Alok Verma was born on 4th March 1976. He has been

working in the Personnel Department of an organization

for the past 6 yr after completing his post graduate

Diploma in Personnel Management with 66% marks. He

has scored 57% marks in the selection process and 63%

marks in graduation.

4. Swapan Ghosh has been working in the Personnel

Department of an organisation for the past 5 yr after

completing his post graduate Degree in HR with 72%

marks. He has scored 56% marks in graduation. He was

born on 12th May 1977. He has scored 58% marks in the

selection process.

5. Seema Behl has been working in the Personnel

Department of an organisation for the past 7 yr after

completing her post graduate Diploma in Personnel

management with 70% marks. She was born on 5th July

1979. She has scored 65% marks in graduation and 50%

marks in the selection process.

Directions (Q. Nos. 6-15) Study the following information

carefully and answer the questions given below. Following are

the conditions for selecting Marketing Manager in an

organisation.

The candidate must

(i) Be a graduate in any discipline with atleast 55%

marks.

(ii) Have a post graduate degree/diploma in Marketing

Management with atleast 60% marks.

(iii) Have post qualification work experience of atleast 5

yr in the marketing division of an organisation.

(iv) Have secured atleast 45% marks in the selection

examination.

(v) Have secured atleast 40% marks in the selection

CLAtutor Logical Reasoning 48

interview.

In the case of a candidate who satisfies all the

conditions except

(A) at (iii) above, but has -post qualification work

experience of atleast 3 yr as Deputy Marketing Manager,

the case is to be referred to GM Marketing.

(B) at (v) above, but has secured atleast 60% marks in the

selection examination, the case is to be referred to VP

Marketing.

In each question below, details of one candidate are

given. You have to take one of the following courses of

actions based on the information provided and the

conditions and sub-conditions given above and marks

the number of that course of action as your answer. You

are not to assume anything other than the information

provided in each question. All these cases are given to

you as on 01.05.2010.

Give Answer

(a) if the candidate is to be selected

(b) if the candidate is not to be selected

(c) if the case is to be referred to GM Marketing

(d) if the case is to be referred to VP Marketing

(e) if the data provided are not adequate to take a

decision

6. Nidhi Agarwal has secured 60% marks in the selection

interview and 40% marks in the selection examination.

She has been working in the marketing division of an

organisation for the past 8 yr after completing her post

graduate degree in Marketing Management with 65%

marks. She has secured 59% marks in B.Sc.

7. Navin Desai has secured 56% marks in B.A. He has

been working in the marketing division of an

organisation for the past 7 yr after completing his post

graduate degree in marketing with 62% marks. He has

secured 62% marks in the selection examination and 38%

marks in the selection interview.

8. Sabina Handa has been working for the past 4 yr as

Deputy Marketing Manager in an organisation after

completing her post graduate diploma in Marketing

Management with 65% marks. She has secured 45%

marks in both selection examination and selection

interview. She has also secured 58% marks in B.Com.

9. Manoj Malhotra has secured 65% marks in B.Sc. and

60% marks in post graduate degree in Marketing

Management. He has also secured 50% marks in both

selection examination and selection interview. He has

been working in the marketing division of an

organisation for the past 6 yr after completing his post

graduation in marketing.

10. Varsha Akolkar has secured 59% marks in B.A. She

has secured 42% marks in the selection interview and

48% marks in the selection examination. She has been

working in the Marketing division of an organisation for

the past 7 yr after completing her post graduation in

Marketing Management with 57% marks.

11. Utpal Goswami has been working in the marketing

division of an organisation for the past 5 yr after

completing his post graduate diploma in Marketing

Management with 65 % marks. He is a first class Science

graduate with 60% marks. He has secured 45% marks in

the selection examination and 40 % marks in the

selection interview.

12. Anindita Chosh has been working for the past 8 yr in

an organisation after completing her postgraduate

degree in Marketing Management with 70% marks. She

has secured 56% marks in B.A. She has also secured 50%

marks in the selection examination and 45% marks in the

selection interview.

13. Samir Phukan has been working in the Marketing

division of an organisation for the past 5 yr after

completing his post graduate diploma in Management

with 65% marks. He has secured 60% marks in B.Com.

He has also secured 50% marks in both selection

examination and selection interview.

14. Nimisha Patil has secured 59% marks in B.C0m. She

has also secured 50% marks in both selection

examination and the selection interview. She has been

working as Deputy Marketing Manager in an

organisation for the past 3 yr after completing her post

graduate degree in Marketing Management with 63%

marks.

15. Sadashiv Ghatge has secured 60% marks in B.Com.

He has been working for the past 5 yr in the Marketing

division of an organisation after completing his post

graduate degree in Marketing Management with 68 %

marks. He has secured 35% marks in the selection

interview and 62% marks in the selection examination.

Directions (Q- Nos. 16-20) Study the following information

carefully and answer the questions given below.

Following are the conditions for selecting Senior

CLAtutor Logical Reasoning 49

Manager Credit in bank.

The candidate must

(i) be a graduate in any discipline with atleast 60%

marks.

(ii) have post qualification work experience of atleast 10

yr in the Advances Section of a bank.

(iii) be atleast 30 yr and not more than 40 yr as on

1.4.2010.

(iv) have secured atleast 40% marks in the group

discussion.

(v) have secured atleast 50% marks in interview.

In the case of a candidate who satisfies all the conditions

except

(A) at (i) above but has secured atleast 50% marks in

graduation and atleast 60% marks in post graduation in

any discipline the case is to be referred to the General

Manager Advances.

(B) at (ii) above but has total post qualification work

experience of atleast 7 yr out of which atleast 3 yr as

Manager Credit in a bank, the case is to be referred to

Executive Director.

In each question, below details of one candidate is given.

You have to take one of the following courses of action

based on the information provided and the conditions

and sub-conditions given above and mark the number of

that course of action as your answer. You are not to

assume anything other than the information provided in

each question All these cases, are given to you as on

01.04.2010.

Give Answer

(a) if the case is to be referred,to Executive Director

(b) if the case is to be referred to General Manager

Advances

(c) if the data are inadequate to take a decision

(d) if the candidate is not to be selected

(e) if the candidate is to be selected

16. Prakash Gokhale was born on 4th August 1977. He

has secured 65% marks in post graduation and 58%

marks in graduation. He has been working for the past

10 yr in the Advances Department of a bank after

completing his post graduation. He has secured 45%

marks in the group discussion and 50% marks in the

interview.

17. Amit Narayan was born on 28th May 1974. He has

been working in the Advances Department of a bank for

the past 11 yr after completing his B.Sc. degree with 65%

marks. He has secured 55% marks in the group

discussion and 50% marks in the interview.

18. Shobha Gupta has secured 50% marks in the

interview and 40% marks in the group discussion. She

has been working for the past 8 yr out of which 4 yr as

Manager Credit in a bank after completing her B.A.

degree with 60% marks. She was born on 12th

September, 1978.

19. Rohan Maskare was born on 8th March 1974. He has

been working in a bank for the past 12 yr after

completing his B.Com. degree with 70% marks. He has

secured 50% marks in both the group discussion and the

interview.

20. Sudha Mehrotra has been working in the Advances

Department of a bank for the past 12 yr after completing

her B.Com. degree with 60 % marks. She has secured

50% marks in the group discussion 40 % marks in the

interview. She was born on 15th February 1972.

Directions (Q. Nos. 21-25) Study the following information

carefully and answer the questions given below.

Following are the conditions for selecting Senior

Manager-General Banking in a bank.

The candidate must

(i) have secured atleast 60% marks in Std. XII.

(ii) have secured atleast 55% marks in graduation in any

discipline.

(iii) have secured atleast 60% marks in post graduate

degree/diploma in Management/ Economicsl Statistics.

(iv) be atleast 25 yr and not be more than 35 yr as on

1.3.2010.

(v) have post qualification work experience of atleast 2

yr as General Banking Officer in a bank.

(vi) have secured atleast 50% marks in written

examination.

(vii) have secured atleast 40% marks in personal

interview.

In the case of a candidate who satisfies all the above

conditions except

(A) at (iii) above, but has secured atleast 60% marks in

CA or ICWA, the case is to be referred to VP

recruitment.

(B) at (vii) above, but have secured atleast 65% marks in

the written examination and atleast 35% marks in the

personal interview, the case is to be referred to President

recruitment.

In each question below are given details of one

CLAtutor Logical Reasoning 50

candidate. You have to take one of the following courses

of actions based on the information provided and the

conditions and sub-conditions given above and mark the

number of that course of action as your answer. You are

not to assume anything other than the information

provided in each question. All these cases are given to

you as on 1.3.2010.

Give Answer

(a) if the data provided are inadequate to take a decision

(b) if the case is to be referred to VP recruitment

(c) if the case is to be referred to President recruitment

(d) if the candidate is to be selected

(e) if the candidate is not be selected

21. Sohan Majhi has secured 65% marks in B.Sc. and 70%

marks in M.Sc. Statistics. He has been working in a bank

as generalist officer for the past 3 yr after completing his

post graduation. He has secured 55% marks in the

written examination and 50% marks in the personal

interview. He was bom on 8th July 1982.

22. Neeta Iaiswal was born on 2nd June 1980. She has

been working in a bank as generalist officer for the past

3 yr after completing her post graduate degree in

Economics with 60% marks. She has secured 68% marks

in Std. XII and 58% marks in B.Com. She has also

secured 50% marks in both the written examination and

personal interview.

23. Arindam Ghosh has been working in a bank as

generalist officer for the past 4 yr after completing his

post graduate diploma in management with 60% marks.

He has secured 50% marks in the written examination

and 40 % marks in the personal interview. He has also

secured 70 % marks in Std. XII. He was born on 25th

February 1975.

24. Keshav Vora was born on 8th November 1978. He

has secured 65% marks in Std. XII and 60% marks in

graduation. He has secured 58% marks in M.A.

Economics and 60% marks in ICWA. He has been

working in a bank as generalist officer for the past 2 yr

after completing his education. He has also secured 50%

marks in the written examination and 45% marks in

personal interview.

25. Neha Salve has been working in a bank as generalist

officer for the past 4 yr after completing her post

graduate degree in Economics with 60% marks. She has

secured 60% marks in both graduation and Std. XII. She

was born on 24th August 1979. She has secured 70%

marks in the written examination and 38% marks in the

personal interview.

Input- Output

Directions (Q. Nos. 26-29) Following are the steps of an

input. Rearrange them and answer the questions.

(A) metal offer arrange blue kite

(B) arrange blue kite metal offer

(C) kite metal offer arrange blue

(D) offer arrange blue kite metal

(E) kite metal blue offer arrange

26. Which of the following is Step III?

(a)A (b)B (c)C (d)D (e)E

27. Which of the following is first from the right end of

Step II?

(a) arrange (b) kite (c) metal

(d) blue (e) None of these

28. Which of the following is Step V?

(a)A (b)B (c)C (d)D (e)E

29. Which of the following is Step IV?

(a)A (b)B (c)C (d)D (e)E

Directions (Q. Nos. 30-34) study the given information and answer the questions. A number arrangement machine, when given an input Iine of number rearranges their following a particular rule in each step. The following is the illustration of the input and steps of arrangement. (All the number are two-digit number)

Input prepare 14 21 a new method 72 38 97 for studies

68

Step II 21 14 for a prepare new method 72 38 97 studies

68

Step III new method 21 14 for a prepare 72 38 97 studies

68

Step IV 68 38 new method 21 14 for a prepare 72 97

studies

Step V studies prepare 68 38 new method 21 14 for a 72

97

Step VI 97 72 studies prepare 68 38 new method 21 14

for a Step VI is the last step of the above arrangement as

the intended arrangement is obtained. As per the rules

followed in the given steps, find out the appropriate

CLAtutor Logical Reasoning 51

steps for the given input.

Input developing 44 markets 38 27 economic practice

required 16 83 72 today

30. Which of the following is step five of the given

input?

(a) today 44 required 38 practice 27 markets 83

developing 72 economic 16

(b) required 44 today 38 markets 27 practice 83 economic

72 developing 16

(c) required today 44 38 markets practice 83 72 economic

27 developing 16

(d) 44 38 required today 27 16 markets practice 83 72

developing economic

(e) today required 44 38 practice markets 27 16 economic

developing 83 27

31. In which step are the elements '72 today required 44'

found in the same order?

(a) Sixth

(b) The given order of elements is not found in any step

(c) Second (d) Fourth (e) First

32. What is the position of ‘markets’ from the left end in

the first step?

(a) Fourth (b) Third (c) Eighth

(d) Ninth (e) Fifth

33. Which element is exactly between '38’ and ‘required’

in the second step of the given input?

(a) 27 (b) There is no element in between

(c) practice (d) economic (e) 44

34. Which element is fifth to the left of the element

which is ninth from the left end of the fourth step?

(a) 16 (b) practice (c) 27 (d) economic (e) markets

Directions (Q.Nos. 35-40) Read the given information and

answer the questions.

When a word and number arrangement machine is

given an input line of words and numbers it arranges

them following a particular rule. The following is an

illustration of input and rearrangement. (All the

numbers are two-digit numbers)

Input left 46 burn 82 95 part 72 vibe bold 49 mint 59

Step I 95 left 46 burn 82 part 72 vibe 49 mint 59 bold

Step II 82 95 left 46 part 72 vibe 49 mint 59 bold burn

Step III 72 82 95 46 part vibe 49 mint 59 bold burn left

Step IV 59 72 82 95 46 part vibe 49 bold burn left mint,

Step V 49 59 72 82 95 46 vibe bold burn left mint part

Step VI 46 49 59 72 82 95 bold burn left mint part vibe

Step VI is the last step of the above arrangement as the

intended output of arrangement is obtained.

As per the rules followed in the given steps, find the

appropriate steps for the given input.

Input 29 cone 42 pale fear 39 67 fame 32 Weld 77 turn

35. Which step number is the following output?

77 29 42 pale fear 39 67 fame 32 weld turn cone

(a)I (b) III (c) VI (d) IV

(E) There is no such step

36. What is the position of ‘fame’ from the right of '67’ in

the second last step?

(a) Eighth (b) Third (c) Fifth

(d) Ninth (e)Seventh

37. Which of the following is the fifth element to the

right of '29’ in Step ll? .

(a) cone (b) turn (c) fame

(d) 39 (e) 32

38. How many elements are there between ‘77’ and

‘weld’ in the last step?

(a) Five (b) Three (c) One (d) Four (e) Two

39. In step ll, which element(s) appear(s) exactly between

‘pale’ and '32’?

(a) Only ’weld' (b) Both ‘weld’ and '42’

(c) Both ‘fear’ and '39’ (d) Only ‘fear’ (e) Only 39'

40. Which of the following represents the first two and

the last two elements in the third last step?

(a) 32, 39, pale, weld (b) 39, 42, fear, pale

(c) 29, 32, pale, turn (d) 29, 32, pale, weld

(e) 32, 39, fear, pale

Directions (Q.Nos. 41-45) Study the given information and

answer the following questions.

When a word and number arrangement machine is

given an input line of words and numbers, it arranges

them following a particular rule. The following is an

illustration of input and rearrangement. (All the

numbers are two-digit numbers.) .

Input 40 made butter 23 37 cookies salt extra 52 86 92 fell

CLAtutor Logical Reasoning 52

now 19

Step I butter 19 40 made 23 37 cookies salt extra 52 86 92

fell now

Step II cookies 23 butter 19 40 made 37 salt extra 52 86

92 fell now

Step III extra 37 cookies 23 butter 19 40 made salt 52 86

92 fell now

Step IV fell 40 extra 37 cookies 23 butter 19 made salt 52

86 92 now

Step V made 52 fell 40 extra 37 cookies 23 butter 19 salt

86 92 now

Step VI now 86 made 52 fell 40 extra 37 cookies 23

butter 19 salt 92

Step VII salt 92 now 86 made 52 fell 40 extra 37 cookies

23 butter 19

Step VII is the last step of the above arrangement as the

intended arrangement is obtained. As per the rules

followed in the given steps, find out the appropriate

steps for the given input.

Input 32 proud girl beautiful 48 55 97 rich family 61 72

17 nice life

41. How many steps will be required to complete the

given input?

(a) Five (b) Six (c) Seven (d) Eight (e) Nine

42. Which of the following is the third element from the

left end of Step VI?

(a) beautiful (b) life (c)61 (d) nice (e)17

43. Which of the following is Step III of the given input?

(a) proud 72 girl 48 family 32 beautiful 17 55 97 rich 61

nice life

(b) life 55 girl 48 family 32 beautiful 17 proud 97 rich 61

72 nice

(c) girl 48 family 32 beautiful 17 proud 55 97 rich 61 72

nice life

(d) family 32 beautiful 17 proud girl 48 55 97 rich 61 72

nice life

(e) girl 48 life 55 family 32 beautiful 17 proud 97 rich 61

72 nice

44. What is the position of ‘nice’ from the left end in the

final step?

(a)Fifth (b) Sixth (c)Seventh (d) Eighth (e)Ninth

45. Which element is third to the right of ‘family’ in Step

V?

(a)beautiful (b) 17 (c)proud (d) 97 (e)32

Directions (Q. Nos. 46-48) Study the following information

to answer the given questions.

A word and number arrangement machine when given

an input line of Words and numbers rearranges them

following a particular rule. The following is an

illustration of input and rearrangement. (All numbers in

these questions are two-digit numbers)

Input 16 today 32 waiting 21 are 11 people 46 bus 66

long

Step I 16 today 32 waiting 21 11 people 46 bus 66 long

are

Step II 16 today 32 waiting 21 people 46 bus 66 long 11

are

Step III 16 today 32 waiting 21 people 46 66 long bus 11

are

Step IV today 32 waiting 21 people 46 66 long 16 bus 11

are

Step V today 32 waiting people 46 66 21 long 16 bus 11

are

Step VI today 32 waiting 46 66 people 21 long 16 bus 11

are

Step VII today Waiting 46 66 32 people 21 long 16 bus

11 are

Step VIII Waiting 46 66 today 32 people 21 long 16 bus

11 are

Step IX waiting 66 46 today 32 people 21 long 16 bus 11

are

Step X 66 waiting 46 today 32 people 21 long 16 bus 11

are

Step X is the last step of the arrangement of the above

input as the intended arrangement is obtained. Now,

answer the questions based on the following input.

Input 23 you 13 wake 81 me 43 before 72 go 34 up

46. Which of these words / numbers would be fourth

(from left side) in Step IV for the input?

(a) me (b) 43 (c) 81 (d) wake (e) None of these

47. The following stands for which step of the

rearrangement? you wake 81 43 72 34 up me 23 go 13

before

(a) Step IX (b) Step IV (c) Step VI

(d) Step V (e) None of these

48. Which of the following would be Step II for the

above input?

CLAtutor Logical Reasoning 53

(a) 23 you wake 81 me 43 72 34 up go 13 before

(b) 23 you 13 wake 81 me 43 72 go 34 up before

(c) 23 you wake 81 me 43 72 go 34 up before 13

(d) 23 you wake 81 me 43 72 go 34 up 13 before

(e) None of the above

Directions (Q. Nos. 49-50) Study the following information to answer the given questions. A word and. number arrangement machine when given an input line of words and numbers rearranges them following a particular rule in each step. The following is an illustration of input and various steps of rearrangement. (All the numbers are two-digit numbers.) Input 11 day 34 night 93 pace 27 easy 44 joy

Step I 93 11 day 34 night pace 27 easy 44 joy

Step II 93 11 34 night pace 27 easy 44 joy day

Step III 93 44 11 34 night pace 27 easy joy day

Step IV 93 44 11 34 night pace 27 joy day easy

Step V 93 44 34 11 night pace 27 joy day easy

Step VI 93 44 34 11 night pace 27 day easy joy

Step VII 93 44 34 27 11 night pace day easy joy

Step VIII 93 44 34 27 11 pace day easy joy night

Step IX 93 44 34 27 11 day easy joy night pace

Step IX is the last step of the rearrangement as the

desired arrangement is obtained. As per rules followed

in the above steps, find out in each of the questions the

appropriate step for the given input.

Input class 25 war 15 race 73 heap 58 just 88 take 38

49. What is the position of ‘war’ in Step VII?

(a) Seventh from the left end

(b) Eighth from the right end

(c) Fifth from the left end

(d) Fifth from the right end

(e) Sixth from the left end

50. Which of the following is ninth term from the right in

Step VI ?

(a) race (b) 25 (c) war (d) 58 (e) 15

CLAtutor Logical Reasoning 54

Data Sufficiency

Directions (Q. Nos. 1-5) Each of the questions below

consists of a question and two statements numbered I

and II given below it. You have to decide whether the

data given in the statements are sufficient to answer the

questions. Read both the statements and give answer.

(a) If the data in Statement I alone are sufficient to

answer the question, while the data in Statement II alone

are not sufficient to answer the question

(b) If the data in Statement II alone are sufficient to

answer the question, while the data in Statement I alone

are not sufficient to answer the question

(c) If the data either in Statement I alone or in Statement

II alone are sufficient to answer the question

(d) It the data given in both Statements I and II together

are not sufficient to answer the question

(e) It the data in both Statements I and ll together are

necessary to answer the question

1. How is J related to K ?

I. J’s father P is brother of N. N is K’s wife.

II. I is son of P. P is brother of N. N is K’s wife.

2. On which floor of the building does G stay? (The

building has five floors 1,2,3,4,5)

I. Only the even numbered floors are occupied and G

does not stay on the second floor.

II. G does not stay on an odd numbered floor.

3. How many days did Raju take to complete his

assignment?

I. Mohit correctly remembers that Raju took more than 3

days but less than 9 days to complete his assignment.

II. Meena correctly remembers that Raju took more than

7 days but less than 11 days to complete his assignment.

4. How is the word ‘GATES’ coded?'

I. ‘BRICK’ is coded as ’LDJSC' ‘PIN’ is coded as ’OJQ’.

II. ‘WATER’ is coded as ‘SFUBX ‘DISH’ is coded as

’ITJE’.

5. Among A, B, C, D, which school has the highest

number of students?

I. School A has fewer students than school D.

II. School C has fewer students than school D.

Directions (Q. Nos. 6-10) These questions consist of a

question and two statements numbered I and II given

below it. You have to decide whether the data given in

the statements are sufficient to answer the question.

Read both the statements and choose the most

appropriate option.

6. In which month (of the same year) did Ravi join

office?

I. Ravi’s mother correctly remembers that Ravi joined

office after June but before October and that month had

less than 31 days.

II. Ravi’s father correctly remembers that Ravi joined

office after August but before December and the month

had only 30 days.

(a) The data in Statement II alone are sufficient to

answer the question while the data in Statement I alone

are not sufficient to answer the question

(b) The data either in Statement I alone or Statement II

alone are sufficient to answer the question

(c) The data in Statement I alone are sufficient to answer

the question while the data in Statement ll alone are not

sufficient to answer the question

(d) The data even in both Statements I and II together

are not sufficient to answer the question

(e) The data even in both Statements I and II together are

necessary to answer the question

7. Is M the mother of L?

I. T is the mother of M. M is married to J. K is the son of

J.F is the sister of K. L is the sister of F.

II. K is the grandson of T.K is the brother of F. F is the

sister of L. L is the daughter of I. I is the son-in-law of T.

(a) The data either in Statement I alone or Statement II

alone are sufficient to answer the question

(b) The data in both Statements I and II together are

necessary to answer the question

(c) The data in Statement II alone are sufficient to answer

the question while the data in Statement I alone are not

sufficient to answer the question

(d) The data in Statement I alone are sufficient to answer

the question while the data in Statement 11 alone are not

sufficient to answer the question.

(e) The data even in both Statements I and II together are

not sufficient to answer the question

CLAtutor Logical Reasoning 55

8. Among six people A, B, C, D, E and F each having a

different height who is the second tallest?

I. Only two people are shorter than D. Only two people

are taller than A. E is shorter than C. D is taller than F.

II. Only three people are shorter than A. B is shorter than

D but not the shortest. C is the tallest amongst them.

(a)The data in both Statements I and II together are

necessary to answer the question

(b) The data in Statement II alone are sufficient to

answer the question while the data in Statement l alone

are not sufficient to answer the question

(c)The data even in both Statements I and H together are

not sufficient to answer the question

(d)The data either in Statement I alone or Statement II

alone are sufficient to answer the question

(e)The data in Statement I alone are sufficient to answer

the question while the data in statement II alone are not

sufficient to answer the question

9. What is position of P?

I. T and V are immediate neighbours of P. Only Q stands

between V and U. Only R stands between S and T.

II. U and S stand at the extreme ends of the row. Only

three people stand between U and T. R stands exactly

between S and T. Q stands to the immediate left of U.

(a) The data either in Statement I alone or Statement H

alone are sufficient to answer the question

(b) The data in both Statements I and II together are

necessary to answer the question

(c) The data in Statement II alone are sufficient to answer

the question while the data in Statement I alone are not

sufficient to answer the question

(d) The data even in both Statements I and II together

are not sufficient to answer the question

(e) The data in Statement I alone are sufficient to answer

the question, while the data in Statement II alone are not

sufficient to answer the question

10. How is the word ‘plant’ written in the given code

language?

I. ‘Buy a new plant today’ is written as '4 $ % 7 2’ and

‘not the plant she said’ is written as ’@ # 5 $ 9 ‘

II. ‘Plant is always good’ is written as ‘> 6 3 $' and ‘is

good buy always’ is written as ‘3 > 2 6'.

(a) The data in Statement I alone are sufficient to answer

the question while the data in Statement II alone are not

sufficient to answer the question

(b) The data in Statement II alone are sufficient to

answer the question while the data in Statement I alone

are not sufficient to answer the question

(c) The data in both Statements I and II together are

necessary to answer the question

(d) The data even in both Statements I and II together

are not sufficient to answer the question

(e) The data either in Statement I alone or Statement II

alone are sufficient to answer the question

Directions (Q. Nos. 11-15) questions consist of a

question and two statements numbered I and II given

below it. You have to decide whether the data given in

the statements are sufficient to answer the questions.

Read both the statements and choose the most

appropriate option.

11. In which month of the year did Rahul go abroad for a

vacation?

I. Rahul correctly remembers that he went for a vacation

in the first half of the year.

II. Rahul's son correctly remembers that he went for a

vacation after 31st March but before 1st May.

(a) The data either in Statement l alone or in Statement II

alone are sufficient to answer the question

(b) The data in both Statements I and II together are

necessary to answer the question

(c) The data in Statement I alone are sufficient to answer

the question while the data in Statement II alone are not

sufficient to answer the question

(d) The data in Statement II alone are sufficient to

answer the question while the data in Statement I alone

are not sufficient to answer the question.

(e) The data even in both Statements I and II together are

not sufficient to answer the question

12. Among friends M, N, O, P, Q and R, who is the

second heaviest?

l. O is heavier than only two friends. P is heavier than O

but lighter than N. R is the heaviest.

II. M is lighter than only two friends. N is heavier than O

but lighter than R. P is heavier than only Q.

(a) The data either in Statement I alone or in Statement II

alone are sufficient to answer the question

(b) The data even in both Statements I and H together

are necessary to answer the question

(c) The data in Statement I alone are sufficient to answer

the question while the data in Statement II alone are not

sufficient to answer the question

CLAtutor Logical Reasoning 56

(d) The data in Statement II alone are sufficient to

answer the question while the data in Statement I alone

are not sufficient to answer the question

(e) The data even in both Statements I and II together are

not sufficient to answer the question

13. How many marks did Suman score in the twenty

marks exam?

I. Suman scored two digit marks and her marks was not

in odd numbers.

II. Suman scored more than 14 but less than 18 marks.

(a) The data in Statement II alone are sufficient to

answer the question, while the data in Statement I alone

are not sufficient to answer the question

(b) The data either in Statement l alone or in Statement Il

alone are sufficient to answer the question

(c) The data in Statement I alone are sufficient to answer

the question while die data in Statement H alone are not

sufficient to answer the question

(d) The data even in both Statements I and H together

are not sufficient to answer the question

(e) The data in both Statements I and II together are

necessary to answer the question

14. Among six people E, F, G, H, I and J standing around

a circle facing the centre, what is the position of G with

respect to F?

I. E stands second to the right of G. Only one person

stands between E and I. F is an immediate neighbour of

G.

II. Only two people sit between G and H, H is an

immediate neighbour of both I and E. F is not an

immediate neighbour of I.

(a) The data in Statement II alone are sufficient to

answer the question while the data in Statement] alone

are not sufficient to answer the question

(b) The data either in Statement I alone or in Statement II

alone are sufficient to answer the question

(c) The data in Statement I alone are sufficient to answer

the question while the data in Statement Il alone are not

sufficient to answer the question

(d) The data even in both Statements I and ll together are

not sufficient to answer the question

(e) The data in both Statements I and H together are

necessary to answer the question

(e) We have to find the position of G with respect to F . I

15. Who among P, Q, R, S and T is the tallest?

I. P is taller than Q,T is not the tallest.

II. R is taller than P, S is not the tallest.

(a) The data either in Statement I alone or in Statement H

alone are sufficient to answer the question

(b) The data in both Statements I and H together are

necessary to answer the question

(c) The data in Statement I alone are sufficient to answer

the question

(d) The data in Statement II alone are sufficient to

answer the question

(e) The data even in both Statements I and II together are

not sufficient to answer the question

Directions (Q. Nos. 16-20) Each of the questions below

consists of a question and two statements numbered I

and II given below it. You have to decide whether the

data given in the statements are sufficient to answer the

questions. Read both statements and choose the most

appropriate option.

16. How many persons are standing between L and K in

a straight line of 19 persons?

Note All are standing in a straight line, facing north

I. Y stands on the extreme left end of the line. Only five

persons stand between Y and K. Only six persons stand

between K and R. Only four persons stand between R

and L.

II. J stands exactly in the middle of the line. Only two

persons stand between I and J. Only five persons stand

between I and L. I stand to the left of L. K stands third to

the left of J.

(a) The data even in both statements I and II together are

not sufficient to answer the question

(b) The data in both statements I and II together are

necessary to answer the question

(c) The data in statement H alone are sufficient to

answer the question while the data in Statement I alone

are not sufficient to answer the question

(d) The data either in statement I alone or in Statement II

alone are sufficient to answer the question

(e) The data in statement I alone are sufficient to answer

the question while the data in Statement II alone are not

sufficient to answer the question.

17. Among six persons A, B, C, D, E and F standing around a circle, some of them are facing the centre while others are facing outside i.e. opposite to the centre,. What is the position of A with respect to E?

CLAtutor Logical Reasoning 57

Note Facing the same direction means, if one is facing the centre, then the other is also facing the centre and vice-versa, Facing the opposite directions means, if one is facing the centre, then the other is facing outside and vice-versa. I. C stands second to the right of E. E faces outside. C is an immediate neighbour of both D and B' F stands second to the left of D. D faces the same direction as E. II. Only two persons stand between B and E. Both B and

E face outside. E is an immediate neighbour of both D

and F. B is an immediate neighbour of both C and A. A

is not an immediate neighbour of D.

(a) The data in both Statements I and II together are

necessary to answa the question

(b) The data in Statement II alone are sufficient to

answer the question while the data in Statement I alone

are not sufficient to answer the question

(c) The data even in both Statements l and II together are

not sufficient to answer the question

(d) The data either in Statement I alone or in Statement ll

alone are sufficient to answer the question

(e) The data in Statement I alone are sufficient to answer

the question while the data in Statement II alone are not

sufficient to answer the question

18. How is X related to N?

I. X is mother of J. T is married to Z. N is daughter of T.Z

is brother of J.

II. X is married to Y. Y is father of J. J is married to L.J is

uncle of N.

(a) The data even in both Statements I and II together are

not sufficient to answer the question

(b) The data in Statement I alone are sufficient to answer

the question while the data in Statement II alone are not

sufficient to answer the question

(c) The data either in Statement I alone or in Statement II

alone are sufficient to answer the question

(d) The data in both Statements I and II together are

necessary to answer the question

(e) The data in Statement II alone are sufficient to answer

the question while the data in Statement I alone are not

sufficient to answer the question

19. Among mobiles R, S, T, U, V and W. which is the

costliest ?

I. T is costlier than only two mobiles. S is costlier than R

but not the costliest. V is costlier than only W.

II. R is cheaper than only two mobiles. V is costlier than

W but cheaper than T. T is cheaper than R. S is cheaper

than U.

(a) The data in Statement II alone are sufficient to

answer the question while the data in Statement I alone

are not sufficient to answer the question

(b) The data even in both Statements I and II together are

not sufficient to answer the question

(c) The data either in Statement I alone or in Statement II

alone are sufficient to answer the question

(d) The data in Statement I alone are sufficient to answer

the question while the data in Statement II alone are not

sufficient to answer the question

(e) The data in both Statements I and II together are

necessary to answer the question

20. In a six storey building (consisting of floors number 1

to 6, where in the top most floor is number 6 and the

ground floor is number 1) each of the six friends, namely

M, N, O, P, Q and R, lives on a different floor (not

necessarily in the same order). Who amongst them lives

on the lower most floor?

I. M lives on floor number five. Only two persons live

between M and N. Q lives immediately above P.

II. P lives on floor number three. Only two persons live

between P and O. N lives immediately above R. N lives

on an even-numbered floor.

(a) The data even in both Statements I and II together are

not sufficient to answer the question

(b) The data in Statement Il alone are sufficient to

answer the question while the data in Statement I alone

are not sufficient to answer the question

(c) The data in Statement I alone are sufficient to answer

the question while the data in Statement H alone are not

sufficient to answer the question

(d) The data either in Statement I alone or in Statement II

alone are sufficient to answer the question

(e) The data in both Statements I and II together are

necessary to answer the question

Directions (Q. Nos. 21-23) Each of the questions below

consists of a question and two statements numbered I

and II given below it. You have to decide whether the

data given in the statements are sufficient to answer the

questions. Read both the statements and give answer.

(a) If the data in Statement I alone are sufficient to

answer the question while the data in Statement II alone

are not sufficient to answer the question

(b) If the data in Statement II alone are sufficient to

answer the question while the data in Statement I alone

CLAtutor Logical Reasoning 58

are not sufficient to answer the question

(c) If the data either in Statement I alone or Statement II

alone are sufficient to answer the question

(d) If the data in Statements I and II together are not

sufficient to answer the question

(e) If the data in both Statements I and II together are

necessary to answer the question

21. Seventeen people are standing in a straight line

facing South. What is Bhavna’s position from the left

end of the line?

I. Sandeep is standing second to the left of Sheetal. Only

five people stand between Sheetal and the one who is

standing at the extreme right end of the line. Four

people stand between Sandeep and Bhavna.

II. Anita is standing fourth to the left of Sheetal. Less

than three people are standing between Bhavna and

Anita.

22. Five letters A, E, G, N and R are arranged from left to

right according to certain conditions. Which letter is

placed third?

I. G is placed second to the right of A. E is on the

immediate right of G. There are only two letters between

R and G.

II. N is exactly between A and G Neither A nor G is at

the extreme end of the arrangement.

23. Six people S, T, U, V, W and X are sitting around a

circular table facing the centre. What is T's position with

respect to X ?

I. Only two people sit between U and W. X is second to

the left of W, V and T are immediate neighbours of each

other.

II. T is to the immediate right of V There are only two

people between T and S. X is an immediate neighbour of

S but not of V.

24. Who is the daughter-in-law of B?

l. I is brother of D. S is wife of J's nephew.

II. R is brother of N. T is son of N. S is mother of T.

III. B is wife of D. D is father of N. D has two children.

(a) Only I and III

(b) All I, II and III are required to answer the question

(c) Only II and III

(d) Question cannot be answered even with I, II & III

(e) Only I and II

25. How many students are there in the class?

I. There are more than 22 but less than 36 students in the

class.

II. If the students of the class are divided into groups,

each group has exactly 11 students.

III. There are more than 29 but less than 45 students in

the class.

(a) Only I and II

(b) Only II and either I or III are required to answer the

question

(c) Only II and III

(d) All I, II and II are required to answer the question

(e) All I, II and III are not sufficient the question

Directions (Q. Nos. 26-30) Each of the questions below

consists of a question and three statements numbered I,

II and III given below it. You have to decide whether the

data provided in the statements are sufficient to answer

the questions. Read all the three statements and give

answer.

(a) If the data in Statement I and II are sufficient to

answer. The question, while the data in Statement III are

not required to answer the question

(b) It the data in Statements I and III are sufficient to

answer the question while the data in Statement II are

not required to answer the question

(c) It the data in Statements II and III are sufficient to

answer the question while the data in Statement I are not

required to answer the question

(d) If the data in Statement I alone or in Statement II

alone or in statement III alone are sufficient to answer

the question

(e) If the data in all the Statements I, II and III together

are necessary to answer the question

26. Among six people P, Q, R, S, T and V, each lives on a

different floor of a six storey building having its six

floors numbered one to six (the ground floor is

numbered 1, the floor above it, number 2 and so on and

the topmost floor is numbered 6. Who lives on the

topmost floor?

I. There is only one floor between the floors on which R

and Q live. P lives on an even numbered floor.

II. T does not live on an even numbered floor. Q lives on

an even numbered floor. Q does not live on the topmost

floor.

III. S lives on an odd numbered floor. There are two

floors between the floors on which S and P live. T lives

CLAtutor Logical Reasoning 59

on a floor immediately above R's floor.

27. There are six letters W, A, R, S, N and E. Is

‘ANSW'ER' the words formed after performing the

following operations using these six letters only?

I. E is placed fourth to the right of A. S is not placed

immediately next to either A or E.

II. R is placed immediately next (either left or right) to E.

W is placed immediately next (either left or right) to S.

III. Both N and W are placed immediately next to S. The

word does not begin with R. A is not placed

immediately next to W.

28. Point D is in which direction with respect to point B?

I. Point A is to the West of point B. Point C is to the

North of point B. Point D is to the South of point C.

II. Point G is to the South of point D. Point G is 4 m

from point B. D is 9 m from point B

III. Point A is to the West of point B. Point B is exactly

mid way between points A and E. Point F is to the South

of point E. Point D is to the West of point F.

29. How is ‘one’ coded in a code language?

I. ‘one of its kind’, is coded as ’zo pi ko fe' and ‘in kind

and cash’ is coded as ’ga to ru ko’.

II. ’its point for origin’ is coded as ’ba Ie fe mi‘ and ‘make

a point clear’ is coded as ‘yu si mi de'.

III. ’make money and cash’ is coded as ‘to mi ru hy’ and

‘money of various kind’ is coded as 'qu ko zo hy’.

30. Are all the four friends A, B, C and D who are sitting

around a circular table?

I. B sits second to the right of D. D faces the centre. C sits

on the immediate right of both B and D.

II. A sits on the immediate left of B. C is not an

immediate neighbour of A. C sits on the immediate right

of D.

III. D is an immediate neighbour of both A and C.B sits

on the immediate left of A. C sits on the immediate right

of B.

Directions (Q. Nos. 31-33) Each of the following

questions below consists of a question and two

statements numbered I and II given below it.

You have to decide whether the data provided in the

statements are sufficient to answer the questions. Read

both the statements and give answer.

(a) If the data in Statement I alone are sufficient to

answer the question while the data in Statement II alone

are not sufficient in answer the question

(b) If the data in Statement II alone are sufficient to

answer the question while the data in Statement I alone

are not sufficient to answer the question

(c) If the data either in Statement I alone or in Statement

II alone are sufficient to answer the question

(d) If the data in both the Statements I and II together are

not sufficient to answer the question

(e) If the data in both the Statements I and II are together

necessary to answer the question

31. Who amongst P, Q, R, S and T is the tallest?

I. P is taller than Q. T is not the tallest.

II. R is taller than P. S is not the tallest.

32. In which direction is point E, with reference to point

S?

I. Point D is to the East of point E. Point E is to the South

of point F.

II. Point F is to the North-West to point S. Point D is to

the north of point S.

33. On which day of the same week is Ramesh’s exam

scheduled (Monday being the first day of the week)?

I. Ramesh correctly remembers that his exam is

scheduled on a day after Tuesday but before Thursday

of the same week.

II. Ramesh’s father correctly remembers that Ramesh's

exam is scheduled on the third day of the week.

Directions (Q. Nos. 34-38) Each of the questions below

consists of a question and two‘ statements numbered I

and II given below it. You have to decide whether the

data provided in the statements are sufficient to answer

the questions give answer.

(a) If the data in Statement I alone are sufficient to

answer the question while the data in Statement II alone

are not sufficient to answer the question

(b) If the data in Statement II alone are sufficient to

answer the question while the data in Statement I alone

are not sufficient to answer the question

(c) If the data either in Statement I alone or in Statement

II alone are sufficient to answer the question

(d) If the data given in both the Statements I and II

together are not sufficient to answer the question

(e) If the data in both the Statements I and II together are

CLAtutor Logical Reasoning 60

necessary to answer the question

34. Among M, P, T, R and W, each of a different age,

who is the youngest?

I. T is younger than only P and W.

II. M is younger than T and older than R.

35. How is ’gone’ written in a code language?

I. ‘you will be gone’ is written as ’ka pa ni sa' in that

code language.

II. ‘he will be there’ is written as ‘ja da ka ni’ in that code

language.

36. On which day of the week (starting from Monday

and ending on Sunday of the same week) did Sushant

visit Chennai.

I. Sushant visited Chennai two days after his brother

visited Chennai.

II. Sushant did not visit Chennai either on Wednesday or

on Friday.

37. Towards which direction is P with respect to the

starting point?

I. P walked 20 m, took a right turn and walked 30 m,

again took a right turn and walked 20 m towards West.

II. P walked 30 m, took a left turn and walked 20 m,

again took a left turn and walked 30 m towards East.

38. How is K related to Z?

I. Z and P are the only sisters of D.

II. D's mother is the wife of K's father.

Directions (Q. Nos. 39-43) Each of the questions below

consists of a question and two statements numbered I

and II given below it. You have to decide whether the

data provided in the statements are sufficient to answer

the questions. Read both the statements.

(a) If the data in Statement I alone are sufficient to

answer the question while the data in Statement II alone

are not sufficient to answer the question

(b) If the data in Statement II alone are sufficient to

answer the question while the data in Statement I alone

are not sufficient to answer the question

(c)If the data either in Statement I alone or in Statement

II alone are sufficient to answer the question

(d)If the data in both the Statements I and II are not

sufficient to answer the question

(e)If the data in both the Statements I and II together are

necessary to answer the question

39. In a six storey building (consisting of floors

numbered 1, 2, 3,4, 5 and 6. The ground floor is

numbered 1, the floor above it is numbered 2 and so on)

the third floor is unoccupied. The building houses

different people viz. P, Q, R, S and T, each living on a

different floor. On which of the floors does T live?

I. S lives between the floors on which R and T live.

II. There are two floors between T's floor and Q's floor.

40. How is ‘see’ written in the code

I. ‘hope to see you’ is written as ’3692', ‘do you see that’

is written as ’1973'.

II. ‘to pray and hope‘ is written as '0286' and ‘hope I do

well’ is written as ‘5467’.

41. Among five friends A,B, C, D and E sitting around a circular table and facing the centre, who is sitting to the immediate left of A? I. A sits third to the right of B. D is not an immediate neighbour of B. II. B is an immediate neighbour of C.

42. Is X the wife of Y? I. X's daughter M is the only sister of R and R is the son of Y. II. The mother of Y has only one grandson R. 43. Among P, Q, R, S and T, which bag is the lightest? I. P is heavier than Q, R is as heavy as Q. T is lighter than R. II. S is lighter than Q but heavier than T. Directions (Q. Nos. 44-48) Each of the questions below consists of a question and two statements numbered I and II given below it. You have to decide whether the data provided in, the statements are sufficient to answer the questions. Read both the statements and give answer (a) If the data in Statement I alone are sufficient to answer the question while the data in Statement II alone are not sufficient to answer the question (b) If the data in Statement II alone are sufficient to answer the question while the data in Statement I alone are not sufficient to answer the question (c) If the data either in Statement I alone or in Statement II alone are sufficient to answer the question (d) If the data even in both Statements I and II together are not sufficient to answer the question (e) If the data in both Statements I and II together are necessary to answer the question

CLAtutor Logical Reasoning 61

44. Who amongst L, M, N, O and P is the shortest? I. O is shorter than P but taller than N. II. M is not as tall as L. 45. Are all the five friends viz. Leena, Amit, Arury Ali and Ken who are seated around a circular table facing the centre? I. Leena sits second to left of Amit. Amit faces the centre. Arun sits second to right of Leena. II. Ali sits third to the left of Ken. Ken faces the centre. Amit sits to the immediate left of Ali but Ken is not an immediate neighbour of Amit. 46. Is T grandmother of Q? I. P is the mother of Q. Q is the son of R. R is the son of T. II. L is father of N and N is daughter of T. 47. Point A is towards which direction from point B? I. If a person walks 4 m towards the North from point A and takes two consecutive right turns, each after walking 4 m, he would reach point C, which is 8 m away from point B. II. Point D is 2 m towards the East of point A and 4m towards the West of point B. 48. How many brothers does Bharat have? I. Sheela, the mother of Bharat has only three children. II. Meena the grandmother of Bharat has only one granddaughter. 49. How many employees are enrolled with the company? I. The employee engagement survey was administered to all employees in the company. II. A total of 946 employee engagement surveys were returned to the HR department. 50. What was the grand total of team A? I. Joseph correctly remembers that Team A scored a grand total of above 85 but below 94 points. II. Surekha correctly remembers that Team A scored a grand total of above 80 and below 87 points.

CLAtutor Logical Reasoning 62

LOGICAL REASONING Assumption and Argument

Statement Assumption Directions for questions 1 to 25: in each question below is given a statement followed by two assumptions/inferences numbered l and ll. An assumption is something supposed or taken for granted and an inference is something which can be directly inferred from the given facts. You have to consider the statement and the following assumptions l inferences and decide which of those is/are implicit in the statement. Give answer: (a) if only I is implicit (b) if only ll is implicit (c) if either I or ll is implicit (d) if neither l nor ll is implicit (e)if both l and ll are implicit 1. Statement : ‘Do not enter-avoid the risk of getting infected with the ABC disease- written outside the quarantine ward no. 2 (meant only for ABC disease) of a hospital. Assumptions: I. Disease ABC is contagious. II.. All the patients in ward no.2 suffer from disease ABC.

2. Statement : In city Z, people prefer to buy Car X instead of Car Y as Car X has German technology which is very advanced. Assumptions: I. Cars witty; .German' technology are perceived to be better than other cars in city Z. II. Had German technology been present in Car Y also, its sales would have crossed car X's sales.

3. Statement : Railway does not provide concession to any one for travelling to certain holiday destinations. Assumptions : I. Railway services are available for travelling to these holiday destinations. II. Railways provides concession to certain persons for travelling to places other than these holiday destinations.

4. Statement : “Travelers with a ticket for the second class if found travelling in the first class compartments would be penalized”- Notice in the compartments of a

train Assumptions: I. Travelers with a ticket for the first class are also not allowed to travel in the second class compartments. II. Inspections are carried out in the train to check the tickets. 5. Statement: The Prices of Petrol and have remained unchanged only in Nigeria since the past three years Assumptions: I.Petrol and diesel prices have changed elsewhere in the world during these three years. II. Before this three years period, petrol and diesel, were available at a Price different from the present rates. 6.Statement lf parking space is not available in office, park your vehicles in the mall and walk to office. Assumptions: I. The mall is at a walkable distance from the office. II. The office does not allow visitors' in its premises. 7.Statement : Farmers must immediately switch over to organic fertilizers from chemical fertilizers for better yield. Assumptions: I. All the farmers use only chemical fertilizers. II. Organic fertilizers are readily available to the farmers. 8. Statement : An advertisement by X-’Our interest rates for education loans are lower than any other bank’. Assumptions: I. Some other banks also provide education II. Interest rates charged on education loans are different for different banks 9.Statement : For any kind of problem with your mobile phone, contact our help desk immediately. Assumptions : I. Help desk has a solution to all kinds of problems related to mobile phones or will guide accordingly. II. Unless the problem is reported immediately, it cannot be solved.

10. Statement : Use our medicine to fight the problem of obesity. Assumptions : I. Other slimming medicines available in the market do

CLAtutor Logical Reasoning 63

not reduce weight. II. Obesity cannot be controlled withou medicines.

11. Statement: The number of people living below poverty line in urban areas has increased since last year. Assumptions: I. People living in rural areas are not below the poverty line. II. A similar survey was conducted last year.

12. Statement: Travelling by trains rather than cabs is more convenient and economical in Paris. Assumptions: I. Paris is an expensive city. II. Train services are reasonably good in Paris.

13. Statement: A government advertisement in public interest-For a child's better mental health, admit him/ her to a school only after five years of age. Assumptions: I. A child cannot learn before he/she turns five. II. Some school admit children who are below five years of age.

14. Statement : Although he has done MBA through a correspondence course, he is as smart as a person from college X. Assumptions : I. Students from college X are known for their smartness. II. Students should do MBA through correspondence in order to become smart. 15. Statement : Conveyance facility provided by organisation helps employees report to work on time. Assumptions : I. The conveyance facility which is provide by the organisation always reaches on time. II. It is not possible to report to work on time unless the convenience facility is provided by the organization.

16. Statement: A promotional-campaign-For healthy children encourage them to play in the playgrounds rather than video games at home. Assumptions: I. Health of at least some children would improve following this campaign. II. Most of the parents would not buy video games for their children after this promotional campaign. 17. Statement: ‘We should export the surplus of wheat in order to utilize its over-production this year‘. Assumptions : I. Knowing that the exported wheat is a part of the surplus of the total production, many countries would

not pay the desired amount of it. II. Some countries are willing to import wheat.

18. Statement : "Sometimes a person acting out of conviction, pursuing a dream, represents the spirit of an entire generation". Assumptions : I. One gets conviction out of one's action. II. Expectations of different generations are not the same.

19.Statement : "Take a recruitment system, for example, the company would make an offer to somebody it had rejected three months ago! There was no recruitment database that kept record of that". Assumptions : I. It is possible to devise a system which would indicate those applicants who were rejected earlier. II. This company considers three months time too short a period to reconsider recruitment decision about a candidate. 20. Statement : "Everyone does not need to build a 100 crore or 1000 crore company. Small vs Big vs Superbig is a choice that an entrepreneur makes herself/himself depending upon the ambitions, values and what she/he likes doing“. Assumptions: I. The figures are for the company. II. There is hardly any distinction a small or a big company.

21. Statement : Please use public transport to avoid traffic congestion during the Trade Fair. Assumptions: I. Only private, vehicles cause traffic congestion II. Many people possess private vehicles. 22. Statement : For better health eat fruits instead of vitamin supplementing tablets. Assumptions: I. Vitamins are essential for good health. II. Vitamin supplementing tablets do not help at all in attaining good health. 23. Statement : As the day for result declaration for board exams closes by, the students awaiting their result start feeling the heat. Assumptions : I. Anxiety levels are more during board exams than any other exam. ll. Some students are more anxious than other students.

24. Statement : Only first class Science graduate are eligible for the job.

CLAtutor Logical Reasoning 64

Assumptions : I. Students with graduation in fields other than Science are not as intellectual. II. Courses other than Science are available at the graduation level.

25, Statement : Any one found smoking in the ‘non-smoking‘ area would be penalized. Assumptions: I. There are separate smoking and non-smoking areas. II. There are some people who do not smoke.

Strong and Weak Arguments

Directions for questions 26 to 41: The questions given

below are followed by two arguments numbered I & ll .

Arguments in relation to the question are either weak or

strong.

Mark your answer as:

(a) if only argument I is strong

(b) if only argument ll is strong

(c) if neither argument l nor II is strong

(d) if both arguments I and ll are strong

(e) if either argument I or ll is strong

26. Main Statement: Should there be a ban on product

advertising?

Argument I: Yes. The money spent on advertising is

huge and it inflates the cost of the product.

Argument II: No. It is an age of advertising. Unless your

advertisement is better than that of your other

competitors the product will not sell.

27. Main Statement: Is clear focus the key to high achievements? Argument I: Yes. One with no goal or focus leads a

barren existence.

Argument II: No. Behind every successful man is a

woman.

28. Main Statement: Is Learning a continuous process?

Argument I: Yes. There is so much to learn and we have

so little time.

Argument II: No. Learning stops after one has obtained

the highest degrees in their subject.

29. Main Statement: Is common currency a necessary

ingredient for Europe’s growth?

Argument I: Yes. A common currency will enable entire

Europe to market itself as one.

Argument II: No. Similar experiments by Latin

American countries have not affected their economies.

30. Main Statement: Is winning more important than

participation?

Argument I: Yes. All sports person are now

professionals who solely depend upon the prize money

for their survival.

Argument II: No. Cricketers in India earn more from

endorsements than from match fee.

31. Main Statement: Should foreign films be banned in

India’?

Argument I: Yes. They depict an alien culture, which

adversely affects our values

Argument II: No. Foreign films are highly engrossing.

32. Main Statement: Should there be reservation of seats

on communal basis?

Argument I: Yes. It will check most of the inter-

communal biases.

Argument II: No. Ours is a secular state.

33. Main Statement: Should English be the medium of

instruction for higher education in India?

Argument I: Yes. Even in advanced countries like

England and USA the medium of instruction is English

for higher education.

Argument II: Yes. English is a much widely spoken

language in the World.

34. Main Statement: Should luxury hotels be banned in

India?

Argument I: Yes. They are places from where

international criminals operate.

Argument II: No. Affluent foreign tourists will not have

a place to stay.

35. Main statement: India should make voting in

elections compulsory for every eligible citizen.

Argument I: Yes, because India spends a lot on

elections, which is very important in the Indian context

and everyone should vote to make India a true

democracy.

CLAtutor Logical Reasoning 65

Argument II: No, because the moment you make things

compulsory it is not a truly democratic way of getting

things done.

36. Main statement: India should increase the number of

universities from 350 to 1500.

Argument I: Yes, it is crucial to sustain India's growth

because we have a large number of people seeking

education.

Argument II: No, the increase will cause a dilution in

academic standards and more corruption.

37. Main Statement: Should India install more nuclear

reactors for electricity generation?

Argument l: Yes, the government can earn more revenue from them. Argument II: No, this will increase the costs of the

government.

38. Main Statement: Plastic bags should be banned.

Argument I: Yes. Plastic Bags lead to clogging of water

resources like springs and rivers causing environmental

degradation.

Argument II: No, Ban of plastics would pose a threat to

the large number of small-scale plastic plants that

employ a lot of people.

39. Main Statement: Should the government abolish

income tax in India?

Argument I: Yes. Because most people default on

income tax payments.

Argument II: No. Because the governments gets a large

part of its revenue from tax collections.

40. Main Statement: Will virtual classrooms and robots

replace real classrooms and human teachers in the near

future?

Argument I: Yes. At the pace we are progressing there

will be a time when everyone will be taught by robots.

Argument II: No. Machines cannot emulate the human

touch, inspiration and motivation. Robots may not be

able to emulate the intelligent thinking process,

imagination, perception and insight of humans.

41. Statement: Should India remain secular?

Argument I: Yes, because that is the best way of

maintaining harmony in a diverse culture like India.

Argument II: No, because the different religious groups

do not see eye to eye.

Directions for questions 42-46: In making decisions

about important questions, it is desirable to, be able to

distinguish between strong‘ arguments and ‘weak’

arguments, ‘Strong’ arguments are those which are both

important and directly related to the questions. ‘weak’

arguments, ‘Strong’ arguments are those of minor

importance and also may not be directly related to the

question or may be related to a trivial aspect of the

question,

Each question below is followed by three arguments

numbered (A), (B) and (C). You have to decide which of

the arguments is a 'strong‘ argument and which is a

‘weak’ argument.

42. Statement : Should there be complete ban on setting

up of thermal power plants in India?

Arguments:

(A) Yes, this is the only way to arrest further addition to

environmental pollution.

(B) No, there is a huge shortage of electricity in most

parts of the country and hence generation of electricity

needs to be augmented.

(C) No, many developed countries continue to set up

thermal power plants in their countries.

(a)Only (A) is strong

(b)Only (B) is strong

(c)Only (C) is strong

(d)Only either (A) or (B) is strong

43. Statement : Should road repair work in big cities be

carried out only late at night?

Arguments:

(A) No, this way the work will never get completed.

(B) No, there will be unnecessary use of electricity.

(C)Yes, the commuters will face lot of problems due to

repair work during the day.

(a)None is strong

(b)Only (A) is strong

(c)Only (C) is strong

(d)Only (B) and (C) are strong f

(e)Only (A) and (B) are strong -

44. Statements : Should all the deemed universities be

derecognized and attached to any of the central of state

universities in India?

CLAtutor Logical Reasoning 66

Arguments:

(A) Yes many of these deemed universities do not

conform to the required standards of a full- fledged

university and hence the level of education is

compromised.

(B) No, these deemed universities have been able to

introduce innovative courses suitable to the requirement

of various industries as they are free from strict Govt,

controls.

(C) Yes, many such universities are basically money

spinning activities and education takes a backseat in

these institutions.

(a) Only A) and (B) are strong

(b) Only B) and (C) are strong

(c) Only A) and (C) are strong

(d) All (A (B) and (C) are strong

(e) None of the above

45. Statement : Should there be a cap on drawing groundwater for irrigation purposes in India? Arguments: (A) No, irrigation is of prime importance for food production in India and it is heavily dependent on groundwater in many parts of the country. (B) Yes, water tables have gone down to alarmingly low levels in some parts of the country where irrigation is primarily dependent on groundwater, which lead to serious environmental Consequences. (C) Yes, India just cannot afford to draw groundwater any further as the international agencies have cautioned India against it. (a) Only (A) and (B) are strong (b) Only (B) and (C) are strong (c) Only (A) and (C) are strong (d) All(A), (B) and (C)are strong (e) None of the above

46. Statement : Should there be a restriction on the

construction of high rise buildings in big cities in India?

(A) No big cities in India do not have adequate open

land plots to accommodate the growing population.

(B) Yes, only the builders and developers benefit from

the construction of high rise buildings.

(C) Yes the Govt should first provide adequate

infrastructural facilities to existing buildings before

allowing the construction of new high rise buildings.

(a)Only (B) is strong

(b)Only (C) is strong

(c)Only (A) and (C) are strong

(d)Only (A) is strong

(e)None of the above

Directions for questions 47 to 50: In making decisions

on important questions, it is desirable to be able to

distinguish between "strong" arguments and weak"

arguments insofar as they relate to the question "Strong"

arguments are those which are both important and

directly related to the question. “Weak” arguments are

those which are of minor importance and also may not

be directly related to the question or may be related to a

trivial aspect of question.

Each question below is followed by two arguments

numbered I and ll. You have to decide which of the

arguments is a “Strong” argument and which is the

“Weak” argument. Give answer.

(a) if only argument I is strong (b) if only argument II is strong (c) if either I or II is strong (d) if neither I nor II is strong (e) if both I and II are strong

47. Statement: Should the provision to dissolve the

prematurely be amended?

Arguments:

I.Yes. On many occasions the provision has been used by

ruling governments to fulfil their vested interests.

II. No. To fulfil the constitutional obligations and norms,

it sometimes becomes need of the hour to dissolve the

assembly prematurely.

48. Statement: Should HIV test be made mandatory in

India for brides and grooms before marriage?

Arguments:

I. Yes. India is one of the countries where the number of

HIV victims is the highest. It is essential for India to

make efforts to restrict the number of victims.

II. No. it will be a humiliation of brides and grooms

both.

49. Statement : Should there be a censor board for, supervising advertisements issued by different companies? Arguments: l. No. It is impractical to have censor boards for various purposes. ll. Yes. On many occasions it has been witnessed that an advertisement has created a lot of debate by showing such scenes in the advertisement as are detrimental to our social fabric.

CLAtutor Logical Reasoning 67

50. Statement : Should students be allowed to work while they are studying? Arguments: I.No. The economic background if every individual does not allow them to pursue education; such a permission therefore makes it difficult to fulfil one's dream. II. Yes. Work plays an important role in making a person self-sufficient. Besides, this practical knowledge becomes necessary to make studies meaningful

CLAtutor Logical Reasoning 68

Course of Action, Cause And Effect

Course of Action

Directions for questions 1 to 25: In each question below

a statement/s followed by two courses of action

numbered I and ll is given. A course of action is a

practicable and feasible step or administrative decision

to be taken for follow-up, improvement, or further

action in regard to the problem, policy, etc. On the basis

of the information given in the statement, you have to

assume everything in the statement to be true, and

decide which of the suggested courses of action logically

follow(s) for pursuing. Give your answer according to

the code given below:

(a) if only I follows

(b) if only ll follows

(c) if either l or ll follows

(d) if neither I nor ll follows

(e) if both I and ll follow

1. Statement : People see tax as a burden and thus

devise ways to underpay or avoid, it altogether.

Courses of action:

l. Govt, should educate & inform citizens about the ways

in which taxes help in development of the nation.

ll. Tax rates should be increased so that the under-

recovery in collection is compensated.

2. Statement : There is no motivation “among today‘s

generation to join the armed forces owing to frequent

transfers to risky areas. Perhaps they are not aware of

the good side of it.

Courses of action:

l. Short term internship should be introduced at high

school level to give students a peek into the adventurous

life of the forces and provide a more realistic job

purview.

ll. The salary levels of the defense forces should be

increased with immediate effect.

3. Statement : A plethora of bodies with no single one

which can be held accountable for city's development is

the key reason behind its rather stagnant state.

Courses of action :

I. Each body should be given a set of duties and there

found be no duplication in such allocation.

II. An umbrella committee should be appointed with

powers to regulate actions of all other bodies.

4. Statement: Number of hospital beds per 1000 people in India, which is a key indicator of healthcare infrastructure is much lower than many other nations which are thought to be poorer than India. Courses of action: l. The expenditure of importing medicines should be borne by the hosipital. ll. incentives should be given to entry of private players into building healthcare infrastructure. 5. Statement : The sales of ball-point pens manufactured

by company Lixus have gone down considerably ever

since the same company introduced a gel-ink pen in the

market.

Courses of Action:

l. Ball-point pens should not manufactured by Lixus

anymore.

ll. Lixus should immediately withdraw all gel-ink pens

from the market so as to force people to buy ball-point

pens.

6. Statement : Nearly 26 per cent of all engineering

graduated have been left unemployed owing to severe

recession worldwide.

Courses of Action :

l. All the unemployed students should be advised to

take up jobs in foreign countries.

ll. The unemployed students should be advised to take

up jobs only after the recession is over.

7. Statement : The government airline has been making

huge losses even as private airliners continue to prosper

and make substantial profit.

Courses of Action:

l. All the private airlines should be banned from their

operation in the country.

ll. The government airline should be instructed to

increase passenger fare significantly in order to increase

its profit.

8. Statement: Many private schools have been violating

government directives and have been charging more

fees than the specified limits.

Courses of Action:

CLAtutor Logical Reasoning 69

l. The government should identify such schools and take

stringent actions against them.

ll. The government should not limit school fees and keep

it flexible for the schools to decide.

9. Statement : A large number of patients admitted in a

particular hospital have been diagnosed with a serious

contagious disease.

Courses of Action :

l. These patients should be immediately evicted from the

hospital for fear of spread of the disease.

ii. The hospital should make appropriate quarantine

arrangements to control the spread of disease.

10. Statement : Many complaints have been registered

lately by commuters regarding unavailability of railway

tickets during the peak traveling season.

Courses of action :

l. The commuters should be advised to defer their travel

plans till the peak season gets over.

ll. Railways should try to accommodate as many

commuters as possible by adding extra coaches to the

trains.

11. Statement : Leader of a tribal community residing

near a proposed highway project has given an

application expressing his community's opposition to

construction of the highway through their residential

area.

Courses of action :

l. Government should convince the tribal community

stating the fact that building of the highway would bring

development in their area.

ll. Government should immediately stall the highway

project in response to the resistance faced by the tribal

community.

12. Statement: More and more farmers have been

shifting to cultivation of export quality rice rather than

the rice eaten daily by people as export quality rice helps

them earn more profit.

Courses of action:

l. Strict action should be taken against the farmers

growing only export quality rice in their fields.

ll. Procurement price of other qualities of rice should be

increased so as to make their cultivation a profitable

venture as well.

13. Statement : According to a recent report, almost 90 %

students from a premier college in India end up settling

in foreign countries for nearly thrice the salary drawn in

India.

Courses of action :

I. Indian organizations should pay salaries to students

from this college which are at par with the foreign

organizations.

Il. Signing of a bond which prevents them from taking a

job in any foreign country should be made mandatory

by the government.

14. Statement : The police reported that two armed

terrorists had entered the city a couple of days ago.

Courses of action :

l. Police should immediately circulate the photographs

of the terrorists using electronic and print media in

order to take public’s help in nabbing them.

ll. Public should be informed to stay indoors until the

terrorists are arrested.

15. Statement : There have been sporadic cases of stone

throwing and damaging vehicles in the locality during

the day following altercation between two local youth

clubs last night.

Courses of action:

l. The local police administration should arrest all those

who are caught committing these acts.

ll. The local police administration should call a meeting

of office bearers of both the clubs of the locality to bring

the situation under control.

16. Statement : A huge truck overturned on the middle

of the main road and blocked most part of the road

causing a huge traffic jam.

Courses of action:

l. The traffic department should immediately deploy its

personnel to divert traffic through other roads.

ll. The traffic department should immediately send men

and equipment to move the truck and clear the road.

17. Statement: Some workers of the company making

diamond jewellery were caught while they were leaving

the premises as they were trying to smuggle small pieces

of diamonds hidden in their purses.

Courses of action :

l. The management of the company should immediately

CLAtutor Logical Reasoning 70

put on hold all activities in the premises till a fool proof

security system is in place.

II. The belongings of all the workers should thoroughly

be searched before they leave the premises of the

company.

18. Statement : A huge tidal wave swept away many

fishing boats and hutments of the fishermen living along

the coastline.

Courses of action :

l. The fishermen should henceforth the restrained from

constructing their huts along the coast line.

II. The local administration should send a team of

officials to assess the extent of demand and suggest

remedial measures.

19. Statement: A large number of invitees who attended

the marriage function fell ill due to food poisoning and

were rushed to various hospitals located in the area.

Course of action:

I. The government should restrict such marriage

functions till further notice.

II. The local hospitals should be advised by the

government to provide best services to the affected

people.

20. Statement: Many pilgrims died in a stampede while

boarding a private ferry to the holy place on the first day

of the ten day long festival.

Courses of action

l. The Govt., should immediately cancel the licenses of

all the private ferry to with immediate effect.

II. The Govt. should deploy its personnel guard to guide

pilgrims on their journey to the holy place.

21. Statement : Some students of the local college were

caught travelling in the train without purchasing valid

tickets.

Courses of action :

l. The parents of these students should be informed

about the incident and requested to counsel their wards.

ll. These students should be put behind bars for

travelling without bonafide credentials.

22. Statement: A large part of the locality was flooded as

the main pipe supplying drinking water burst while the

workers of a utility company were laying cables in the

area.

Courses of action:

I. The civic authority should immediately arrange to

repair the damage and stop loss of water.

II. The civic authority should seek an explanation and

compensation from the utility company for the damage

caused by them.

23. Statement : Millions of pilgrims are expected to take

a dip in the Ganges at the holy place during the next

fortnight.

I. The government should restrict the number of

pilgrims who can take dip each day during that period.

II. The Government should deploy an adequate number

of security personnel to maintain law and order during

the next fortnight at the holy place.

24. Statement: The rate of inflation has reached its

highest in the last 20 years and there is no sign of it

softening in the coming months.

Courses of action:

I. Government should initiate steps like reducing

Government taxes on essential commodities with

immediate effect.

II. Farmers should be asked by the Government sell their

produce at lower Prices.

25. Statement: Many customers complained to the

manager of the local branch of the bank against the

highhandedness of the bank employees while dealing

with the customers.

Course of action:

I. The bank manager should immediately suspend all the

employees of the branch and deploy a different set of

employees.

II. The bank management should ask for a report from

the manager of the branch for taking necessary steps.

Cause and Effect

Directions for questions 26-31: Below in each question

are given two statements (A) and (B). These statements

may either be independent causes or effects of

independent causes or a common cause.

One of these statements may be the effect of the other

statement. Read both the statements and decide which of

the following answer choice correctly depicts the

relationship between these two statements. Mark your

answer according to the code given below:

(a) if statement (A) is the cause and statement (B) is its

CLAtutor Logical Reasoning 71

effect

(b) if statement (B) is the cause and statement (A) is its

effect

(c)if both the statements (A) and (B) are independent

causes

(d) if both the statements (A) and (B) are effects of

independent causes

(e) if both the statements (A) and (B) are effects of some

common causes.

26. A. The government has increased number of buses

plying between Mumbai and Surat.

B. The government has allowed private bus owners to

operate between Mumbai and Surat.

27. A. The association of management colleges

conducted a combined admission exam for all the

institutes this year.

B. As the dates for entrance exam for many management

colleges clashed last year, many candidates complained

that they could not appear for a number of entrance

exams.

28. A. The departmental store owner put cameras for

surveillance of the store.

B. Many products were reported missing by the staff

working in the departmental store.

29. A. Asthma cases in the city are on a continuous rise

since last year.

B. The number of commonly found animals such as

sparrows, squirrels and pigeons in the city has

continuously dwindled since last year.

30. A. Prices of food grains in India has increased

substantially.

B. Lakhs of farmers have quit farming owing to the lack

of governmental support in the form of subsidies.

31. A. The university decided to postpone the final

exams for course X by a month.

B. Nearly twenty five students out of sixty students in

course X failed to pass the final exams last year.

32. Effect: The temple at the religious site wears a deserted look with the number of devotees trickling down. Which of the following can be a possible cause of the above effect? (a) A structural engineer had visited the temple a month back and had declared the structure unsafe. (b) The temple is facing a drastic depletion of its funds

which had accumulated over the years due to offerings made by devotees. (c) The local corporation decided to donate a huge amount of money to the temple for its renovations. (d) The village housing the religious site has qualified priests to perform religious ceremonies. (e) A famous actor recently visited the temple and paid his respects to the deity. 33. Statement : The income tax authorities carried out raids at three different business houses in the city last week. Which of the following can be a possible effect of the above statement ? (a) The three business houses are regular defaulters in payment of their income tax. (b) The income tax department had received a tip off about the illegal activities going on in the three business houses. (c) The government decided to look into the matter and has appointed an inquiry committee. (d) Other business houses took immediate action to clear off all their income tax dues, in order to avoid a raid on their establishments. (e) The authorities intend to conduct raids in several other business houses in the vicinity. 34. Effect As a step to regulate private hospitals, the state health department is framing rules to ensure all such hospitals are registered with it. Which of the following can be a possible cause of the above statement? (a) The department realised that private hospitals charge much less for treatment as compared to government hospitals. (b) Government run hospitals do not maintain the same standards as private hospitals. (c) The department realised that several hospitals were rejecting cases stating lack of infrastructure (d) Apart from the number of doctors, nurses and beds, the kind of procedure a hospital can carry out based on its infrastructure will also be registered and detailed. (e) Private hospitals not registering with the department shall be forced to do so and will have to Pay hefty Penalties. 35. Statement The constable has been recommended for a suitable reward by his superior in recognition of his sincere duty and busting of several gangs of criminals actively involved in the loot and incidents of pick- picketing. Which of the following can be an assumption implicit in the above statement?

CLAtutor Logical Reasoning 72

(a) The superior is certain that the recommendation would be denied. (b) The number of criminals apprehended by this particular constable was exceptionally high. (c)The constable desires to be momentarily compensated for his efforts. (d) The superior wants to set an example to other juniors by recommending reward. (e) Rewards recognizing the sincerity and accomplishments of policemen are given. 36. Statement: The college has finally received accreditation and has gained the status of a deemed university. Which of the following can be a effect of the above statement ? (a) The principal of the college will now have to be a retired government official. (b) Number of students seeking admission to this college in the next academic year would drop significantly. (c) The college will charge lesser fees from all its students despite not getting a grant from the government. (d) The college will reduce the number of course that it runs by a significant margin. (e) The reputation of the college amongst the student population in general will improve. Directions for questions 37-50: In each question below are given two statements (A) and (B). These statements may either be independent causes or effects of independent causes or a common cause. One of these statements may be the effect of the other statement. Read both the statements and decide which of the following answer choice correctly depicts the relationship between these two statements. Mark your answer according to the code given below. (a)if statement (A) is the cause and statement (B) is its effect (b)if statement (B) is the cause and statement (A) is its effect (c)if both the statements (A) and (B) are independent causes: (d)if both the statements (A) and (B) are effects of independent causes (e)if both the statements (A) and (B) are effects of some common cause 37.A. The average day temperature of the city has increased by about 2 degrees in the current year over the average of the past ten years.

B. More people living in rural areas of the state have started migrating to the urban areas in comparison with the earlier year. 38.A. Most of the shopkeepers in the locality closed their shops for the second continuous day. B. Two groups of people living in the locality have been fighting with each other with bricks and stones forcing people to stay indoors. 39. A. The Govt., has decided to increase the prices of LPG gas cylinders with immediate effect. B. The Govt., has decided to increase the prices of Kerosene with immediate effect. 40. A cyclonic storm is expected to hit the coastline in the state during the next 48 hours. B. Warning has been issued that heavy rains are expected in the coastal region in the state during the next 48 hours. 41. A. Majority of the first year students of the engineering college failed in Mathematics in the semester examination' B. The college authority terminated the contract of the Professor who taught Mathematics to the first year students. 42.A. The government of India has allowed the private airline companies to operate on specified international routes. B. There has been a considerable increase in the flow of foreign tourists to India. 43. A. Many seats in the private engineering colleges in the state have remained vacant this year. B. The government engineering colleges in the state could not accommodate all the students who sought admission this year. 44. A. The banks have decided to give advances to the priority sector at the rate of interest at par with the corporate sector. B. The percentage of bad loans given by the banks to the priority sector is very low as compared to the corporate sector. 45. A. The stale government has now decided to increase the stamp duty on house purchases with immediate effect. B. The real estate prices have increased considerably during the last few months.

CLAtutor Logical Reasoning 73

46. A. Most of the steel manufacturing companies in the country have made considerable profit during the last fiscal year. B. Many Asian countries have been importing huge quantities of steel from India. 47. A. Three top players of the Indian cricket missed all the matches of the five match tournament due to injury. B. India lost the cricket tournament to the visitors with a 2 : 3 margin. 48.A. The prices of food grains have increased by about 20 percent over the same period last Year. B. The Prices of vegetables have substantially decreased during the past few weeks.. 49. A. A huge explosion rocked the housing complex as a cooking gas cylinder exploded in one of the flats in the complex. B. Residents of the housing complex came running out of their houses and assembled in the open courtyard. 50. A. Majority of the students of the school passed in Mathematics in the annual examination. B. Majority of the students of the school failed in English in the annual examination.

CLAtutor Logical Reasoning 74

Inference and Conclusion

Inference

1. Statement : In the university examination most of the

candidates write in Hindi medium.

Inferences:

(a) All the candidates who appear in this examination

write answers in Hindi.

(b) In this examination no candidate writes answers

other than in Hindi.

(c) Mostly candidates with Hindi medium appear in this

examination.

(d) Some candidates of this examination write in Hindi.

2. Two statements are given followed by four inferences.

Select the alternative which is most appropriate.

Statements:

India is becoming industrialized. Pollution is a problem

associated with industrialization

Inferences:

I. All industrial centres are polluted.

II. India is polluted.

III. Polluted nations are industrialized.

IV. India may become polluted.

(a) All are appropriate.

(b)None is appropriate

(c)Only IV is appropriate

(d)Only II is appropriate

3.Statements:

Many science collegiate are highly creative. Creative

persons are very intelligent.

Inferences:

(a) Many science collegiate are very intelligent.

(b) Creative science collegiate are very intelligent.

(c) All science collegiate are very intelligent.

(d) Persons who are very intelligent are highly creative

Science collegiate.

4.In this question, a statement is given followed by four

alternative inferences. Select the one which is most

appropriate.

Statement: A large number of language specialists write

novels.

Inferences:

(a) Some language specialists do not write novels.

(b) A novelist need not be a language specialist.

(c)Language specialty are language specialties.

(d) All novelists are language specialists.

5. Statement: “Electric supply in Anand colony will be

cut tomorrow after 12 O’clock for three hours because

repair work will be carried out”.

Inferences:

I. Residents of Anand colony are likely to use their

electrical appliances before 12 O’ clock tomorrow.

II. Residents of Anand Colony need training for using

electricity economically.

(a)Only inference I follows

(b)Only inference II follows

(c)Both the inference follow

(d) None of the inferences follows.

6. Statements:

All the watches sold in that shop are made of white

metal.

Some of Rico watches are sold in that shop.

Inferences:

I. All watches of white metals are manufactured by Rico.

II. Some of Rico watches are of white metal.

III. None of Rico watches are of white metal.

IV. Some of Rico watches of white metals sold in that

shop.

(a) I and II inferences only

(b) II and IV inferences only

(c) II and III inferences only

(d) I and IV inferences only

7. “If a person is rich, he has a lot of influence.” What

influence can you draw from the above statement?

(a) Kamala is rich, so she has lot of influence.

(b) Poor people cannot have influence.

(c) Ram has a lot of influence, so he is rich.

(d) Govind is not rich, so he does not have lot of

influence.

Conclusion

Directions for questions 8 to 17: Each of the following

questions consists of statements followed by two

CLAtutor Logical Reasoning 75

conclusions. Read the given statements carefully and

identify which of the conclusions directly follow

Mark your answer as:

(a) If only conclusion I follows

(b) if only conclusion II follows

(c) if both conclusions I and II follow

(d) neither conclusion I nor II follows

8.Statements:

Sheila is an MBA. Some MBAs specialize in information

Systems Management.

Conclusion:

I. Sheila specialized in information Systems Management. II. Not all MBAs specialize in information Systems

Management.

9.Statements: : Asphalt is very hard. Oil is of the same density as asphalt. Conclusion: I,.Oil is very hard. II. The density of asphalt and oil can be measured.

10. Statements: Murder is a moral offence. You have committed a moral offence. Conclusion: I. You are a murderer. II. Murderers are immoral.

11. Statements: Earth is a part of our solar system. Our solar system is part of the milky way galaxy. Conclusion: l. Earth is part of the milky way galaxy. ll. Milky way galaxy is in the solar system. 12. Statements:

Watching cricket is one of the major recreational activities in India. He has gone for recreation. Conclusion: l. He has gone to watch cricket. ll. Cricket is sometimes played for pleasure in India. 13. Statements: For the past 10 years people of Kashmir have lost their peace of mind because of fear of terrorist attack, My uncle who is residing in Kashmir has also lost his peace of mind. Conclusion l.My uncle may be fearing a terrorist attack.

II. People staying in Kashmir cannot lead a 'peaceful life. 14. Statements: Sachin owns a Ferrari. Some Ferraris have left-hand drive. Conclusion: I.sachin had bought a Ferrari for himself. ll. Sachin's car may have a left hand drive. 15. Statements:

The reason for the upsurge in the incidence of cardiac disease among young professionals is occupational stress. Mr. Singh, a young professional recently succumbed to cardiac arrest. Conclusion: l. Mr. Singh's death was due to occupational stress. ll. A lot of young professionals are facing occupational stress now days. 16. Statements: CET forms can be procured either by paying cash or by submitting a DD. Many people prefer cash. She has procured a CET form. Conclusion: I.She submitted a DD for the CET form. ll. Purchasing the form by making cash payment is more convenient than paying through a DD. 17. Statements: Polar bears do not have fur on them, Animal X doesn't have fur on it. Conclusion: l. Animal X is from the poles. ll. Polar bear's skin is thick enough to protect it from cold. Directions for questions 18 to 37: In the following questions, one/two statements are followed by two conclusions 1 and 2. You have to consider the statements to be true, even if they seem to be at variance from commonly known facts, You have to decide which of the given conclusions, if any. Follow from the given statements. Mark your answer as: (a) if neither conclusion 1 nor conclusion 2 follows (b) if both conclusions 1 and 2 follow (c) if only conclusion 1 follows (d) if only conclusion 2 follows 18. Statements:

1.Alexander visited Egypt. 2. Cleopatra was the queen of Egypt.

CLAtutor Logical Reasoning 76

Conclusions: 1. Alexander met Cleopatra. 2. Egypt was a monarchy, 19. Statements:

1. India is a democratic country. 2.Pakistan is a democratic country. Conclusions: 1. India and Pakistan are both democratic. 2. India and Pakistan are neighbors. 20. Statements:

1. Cricket is played with red balls. 2. Snooker uses many red balls Conclusions: 1.Snooker can be played with cricket balls. 2. Cricket can be played with snooker balls. 21. Statements:

1.Many people are rich and educated 2. Many educated people are poor. Conclusions: 1, Rich people are educated. 2, Poor people are educated. 22.Statement:

The government provides subsidy to help rich people. Conclusions: 1. Rich people need help. 2. Rich people get subsidies. 23. Statements:

1. Bills are discussed first in the Lok Sabha and then in the Rajya Sabha 2. lf both houses of Parliament pass the bill, it becomes a law. Conclusions: 1. There are two houses in the Parliament 2. The Lok Sabha is larger than the Rajya Sabha. 24. Statements:

1 . Many streets are named after politicians. 2, Many streets are named after social workers. Conclusions: 1.Streets have names. 2. Politicians are social workers,

25. Statements: 1.Joyce is a well-known literary figure. 2.Joyce has a distinct writing style. Conclusions: I.A distinct writing style leads to popularity. II.Joyce won the Nobel Prize for Literature

26.Statement: India, one-third of patients who have brain strokes are left permanently paralyzed. Conclusions: I.Cases of brain stroke are on the rise in India. II.Brain stroke might lead to permanent paralysis at times. 27. Statements:

1. Each student will read two books every week. 2.Each student will submit a report on one of the two books every week. Conclusions: I. The books provided for reading are interesting" II.Interest in reading can be developed by force. 28.Statements

1. The company reported losses in the first quarter. 2 The company has been profitable in the Past. Conclusions: 1. The company will make a profit in the second quarter. 2 The company is going to be bought by a bigger company. 29.Statements: 1.A flag has many colors. 2 India’s flag has 4 colors, including white. Conclusions: 1. India may not be a nation. 2. India has to be a nation. 30.Statements: 1.Rich people have lots of free time. 2. Rich people buy many cars. Conclusions: 1. Having many cars saves time. 2. Many rich people purchase cars, 31.Statements: 1, Air travel is more expensive than train travel. 2. Train travel takes less time than air travel. Conclusions: 1. To save time, one should use train travel. 2. To save money one should use train travel.

32. statements: 1.Athletes like David exercise for more than a quarter of every day, 2.For anyone to become an athlete, they must do more than 6 hours of exercise everyday. Conclusions:

CLAtutor Logical Reasoning 77

1. David exercises for more than 6 hours everyday. 2. Anyone can be an athlete.

33.Statements: 1. Watches are used to keep track of time. 2. Clocks are made all over the world to keep track of time. Conclusions: 1. Clocks and watches are substitutes of each other. 2. Watches are made all over the world.

34. Statements:

1. The government holds exams to select people for various posts lying vacant. 2. Many people give these exams to get their first jobs. Conclusions; 1. There are many posts that need to be filled. 2. Many people are unemployed.

35. Statements:

1.North Korea and South Korea are separated by water. 2. They threaten war on each other.

Conclusions: 1.North Korea and South Korea are islands, 2. They are fighting over who gets to keep the water between them. 36. Statements: 1. The internet is a web of interconnected computers. 2. All computers have input and output devices. Conclusions: 1. The internet is an output device. 2. A single computer cannot make an internet. 37. Statements: 1.Water is necessary for life to survive 2. Food is necessary for life to survive.

Conclusions:

1. Water and food are life.

2. Life cannot survive without water and food.

Directions for questions 38 to 43: In the following questions, two statements are given, followed by two conclusions I and Il. you have to consider the statements to be true even if they seem to be at variance from commonly known facts. you have to decide which of the given conclusions, if any, follow from the given statements. 38. Statements: 1. Due to contamination of water, large number of people were admitted to hospitals.

2. The symptoms were of Typhoid.

Conclusions:

I.Contamination of water may lead to Typhoid. ll. Typhoid is a contagious disease.

(a) Only conclusion I is true (b) Only conclusion II is true

(c)Both conclusions I and II are true

(d) Both conclusions I and II are false

39. Statements:

1.60% of the government employees went on strike. 2.Mr. Gopal is a government employee. Conclusions: I Mr.Gopal went on strike. ll. Mr. Gopal did not participate in the strike (a) Only conclusions l follows (b) Only conclusions ll follows (c) Both conclusions I and ll follow (d) Either conclusion I or ll follows 40. Statements:

1. Temple is a place of worship. 2. Church is also a place of worship. Conclusions: I. Hindus and Christians use the same place for worship. II. All churches are temples. (a) Neither conclusion I nor II follows (b) Both conclusions I and II follows (c) Only conclusion I follows (d) Only conclusion II follows 41. Statements: 1. India is becoming industrialized. 2. Pollution is a problem associated with industrialization. Conclusions I.Polluted nations are industrialized. II. India may become polluted. (a) All are appropriate (b) Only I is appropriate (c) Only ll is appropriate (d) None is appropriate 42. Statements:

1. Happiness derived from external materials is momentary. 2.Everlasting happiness has to come from within. Conclusion: I.Nobody can experience happiness from outside, ll. Happiness experienced from cinema is not lasting. (a) Only I follows

CLAtutor Logical Reasoning 78

(b) Only II follows ' (c) Neither I nor ll follows (d) Both I and il follow 43. Statements

1. Sachin is a good cricketer, 2. Sachin's sister is a good singer. Conclusions: I.Sachin and his sister are good cricketers. II.Sachin and his sister are talented. (a) Only I follows (b) Only II follows (c) Neither I nor II follows (d) Both I and II follow Directions for questions 44 to 50: Each of the following questions consist of statements followed by two conclusions. Read the given statements carefully and identify which of the conclusions directly follow. Mark your answer as: (a) if only conclusion I follows (b) if only conclusion II follows (c) if both conclusions I and II follow (d) if neither conclusion I nor II follows 44.Statements:

Free and fair elections have been held in Taliban. The extremists could not stop them The government must be complimented for it. Conclusions:

I.The government could have taken advantage of the situation. ll. The government has tackled the problem of extremists decently enough. 45.Statements: Both ,India and Pakistan are democratic countries. Both have democratically elected Prime Ministers. Both the prime Ministries want peace. Conclusions: l. India and Pakistan are unlikely to follow the path of confrontation. ll. India and Pakistan are neighbouring countries. 46.Statements: Many people are endowed with plentiful time, idling away time is a sign of high status for them. Conclusions: I.Affluent people idle away time. II.Good fortune makes a person idle,

47.Statements: Time saving devices constitutes a new work culture. This culture removes sloth and India needs it badly. I. All Indians are very slow and slothful. II. India does not have time-saving machines 48 Statements: The Planning commission is opposed to the proposal it feels that the cost of subsidizing helicopter operations will be exorbitant. Conclusions: I. The Planning Commission wants the Government to spend less on itself. II. Helicopter operations must be subsidized. 49. Statement:

The population below the poverty line is computed on the basis of minimum daily calorie requirement of food and actual consumption Forty-eight percent of India’s population lives below the poverty line. Conclusions: I.Forty-eight percent of the people in India do not get the required calories of food II. Poverty is India’s biggest problem 50.Statements:

The policy of liberalization will make the rich richer and the poor poorer. The disparity between the rich and the poor will widen. Conclusions: l. Liberalization is not good for India ll. The rich believe in liberalization.

CLAtutor Logical Reasoning 79

Critical Reasoning

Directions for questions 1 to 4: Read the following

statements carefully and answer the questions which

follow.

1.Real estate builders have refused to bring down the

property prices as proposed by the government this

year.

Which of the following arguments would strengthen the

stance taken by the builders?

(a) Very few people ventured into buying properties this

year owing to exorbitant prices set by the builder.

(b) With the ever increasing cost of basic materials such

as cement and steel, the profit of the builders has gone

down by 48% this year.

(c) The builders have earned huge profit in a

governmental scheme for building low cost housing

societies.

(d) In a report published by a national daily the margin

of profit earned by a builder per square foot is as much

as eighty percent of its cost price. .

(e) The builders have to necessarily abide by the decree

of the government which controls the prices of the real

estate.

2.The ministry of sports has been advised by a

committee to take the highest award in the field of sports

back from two players who were allegedly involved in

Match-fixing.

Which of the following statements would weaken the

argument put forward by the committee to the sports

ministry ?

(a) A good conduct in the past and a lack of evidence

against the players make the case against them very

weak.

(b) The ministry of sports has never declined the

recommendations made by the committee earlier.

(c) Taking the award back from the players would set a

good example to other players for avoiding such actions

in the future.

(d) There have been past cases where the award had to

be taken back from the players owing to some

misconduct later on.

(e) The committee is constituted of some of the most

respected and esteemed members from the fields of

sports and politics.

3.Many organizations have been resorting to recruitment

based upon performance at graduate/post-graduate

level exams rather than conducting exam for the same

purpose.

Which of the following statements would strengthen the

argument given in the above statement?

(a) A recent study shows no link of past performance

with the performance in A recruitment exams.

(b) The graduate/post-graduate exams are considered to

be severely deficient in training in job related

environment.

(c) Organizations which had undertaken recruitment on

the basis of graduate/post graduate exams report a

significant drop in the quality of the recruited

employees.

(d) Such policies would did to unemployment among

students having below average performance in

graduation or post-graduation.

(e) Such policies could save time, money and resources

of the organization which are wasted in the conduct of

recruitment examinations.

4. According to a recent government directive, all bank

branches in rural areas should be computerised.

Which of the following statements would weaken the

government's argument?

(a) Computerization of bank branches in urban areas has

helped in making their performance more efficient and

fast.

(b) Lack of skilled and qualified manpower has been

suitably substituted by computers in banks.

(c) Non—computerised bank branches in the rural areas

have been proved to be as efficient as their

computerized counterparts.

(d) The government has introduced a special test for

computer knowledge in all recruitment exams for banks.

CLAtutor Logical Reasoning 80

(e) Unemployment in the rural areas could be controlled

by training more and more professionals in computers.

5. Statement : The government has decided to instruct

the banks to open new branches in such a way that

there is one branch of any of the banks in every village

of population 1000 and above or a cluster of villages

with population less than 1000 to provide banking

services to all the citizens.

Which of the following weaken the step taken by the

government?

(a) The private sector banks have stepped up their

branch expansion activities in rural India.

(b) Many government owned banks have surplus

manpower in its urban branches.

(c) All the banks including those in private sector will

follow the government directive.

(d) Large number of branches of many government

owned banks in the rural areas are making huge losses

every year due to lack of adequate business activities.

(e) None of the above

6. Cause The government has recently increased its taxes

on petrol and diesel by about 10 percent.

Which of the following can be a possible effect of the

above cause’?

(a) The petroleum companies will reduce the prices of

petrol and diesel by about 10 percent.

(b) The petroleum companies will increase the prices of

petrol and diesel by about 10 percent.

(c) The petroleum companies will not change the prices

of petrol and diesel.

(d) The petrol pumps will stop selling petrol and diesel

till the taxes are rolled back by the government.

(e) None of the above.

7. Statement : Most of the companies in IT and ITES

sectors in India have started hiring from engineering,

college campuses this year and are likely to recruit much

more than yearly recruitment the earlier years.

Which of the following substantiates the facts stated in

the above statement ?

(a) IT and ITES are the only sectors in India which are

hiring from engineering college campuses.

(b) Government has stepped up recruitment activities

after a gap of five years.

(c) The IT and ITES companies have now decided to visit

the engineering college campuses for tier II cities in

India as well.

(d) Availability of qualified engineers will substantially

increase in the near future

(e) None of the above

8. Effect Majority of the employees of the ailing

organization opted for voluntary retirement scheme and

left the organization with all their retirement benefits

within a fortnight of launching the scheme.

Which of the following can be a probable cause of the

above effect?

(a) The company has been making huge losses for the

past five years and is unable to pay salary to its

employees in time.

(b) The management of the company made huge

personal gains through unlawful activities.

(c) One of the competitors of the company went

bankrupt last year.

(d) The company owns large tracts of land in the state

which will fetch huge sum to its owners.

(e) None of the above.

9. Read the following information and five statements

given below it carefully and answer the questions which

follow.

Excerpt from a research report-‘Average life expectancy

in southern part of India is far more than that in Western

India. While the average life of a native of South India is

82 years, the average life of a native of Western India is

only 74 years.

Based on the above fact, the proposal that the above

study makes is that if an individual moves from Western

India to South India, his/her life expectancy would

immediately increase by 8 years?

Which of the following statements would weaken the

above-mentioned study‘s proposal that people

belonging to Western parts of India should move to

South India to increase their life expectancy?

(a) The average life expectancy of population living in

Eastern part of the country is also less than the

population living in South India.

(b) Nearly 80% of the population in Southern India has

a minimum age of 83 years.

(c) Higher life expectancy in Southern India can be

ascribed to the genetic makeup of the population

belonging to that area.

(d) The average life expectancy of South India is

comparable to the best averages in the world.

CLAtutor Logical Reasoning 81

(e) Higher life expectancy in Southern India can be

attributed to better environmental conditions and better

healthcare facilities.

Directions for questions 10 to 13: Read the following

information carefully and answer the questions which

follow:

Fortunately more and more countries are shifting their

focus away from industrial development to control of

climate changes these days.

A.The counties which focus more on controlling climate

change than industrial development are only the richer

ones, which can afford to concentrate on area other than

industrial development.

B.Many countries had once prioritised industrial

development which proved to be harmful to the

environment in the long run.

C. Some experts are of the view that climate change is

not as alarming an issue as it is made to be because it is a

natural phenomenon and has been occurring regularly

throughout the history of earth.

D. If climate change continues at the present rate, it

would bring in large-scale destruction to human

habitation in a very short time.

E.Industrial development is one of the biggest but

definitely not the only reason behind global warming.

10. Which of the statements numbered A, B, C, and D

mentioned above would weaken the argument for the

need of a shift away from industrial development to that

of controlling climate changes?

(a) A (b) B (C) C (d) D (e) Both B and D

11. Which of the statements numbered (A), (B), (C), (D)

and (E) can be assumed/inferred from the

facts/information given in the statement?

(An assumption is something supposed or taken for

granted and an inference is something which is not

directly stated but can be inferred from the given facts.)

(a) E (b)C (c)A (d) B (e) Either D or A

12. Which of the following can be inferred from

statement (E) if it is considered to be true with regard to

the given information?

(a) Nations also need to focus on sources other than

those generated due to industrial development.

(b) Other sources of pollution have more adverse effects

as compared to those generated due to industrial

development.

(c) Unlike older times, industrial development has

ceased to be a reason behind global warming these days.

(d) if industrial development stops, global warming

would automatically come to an end.

(e) If sources other than, industrial global warming will

end completely.

13. Which of the statements numbered A, C, D and E

mentioned above represents a reason behind curtailing

industrial development by some of the countries?

(a) Either C or E (b) D (c) C

(d)Either A or C (e)A

14. Statement:

The chairman of a car company announced in a meeting

that all trials of its first product, a new model of car ‘X’,

are over and the company is planning to launch its car in

the market after six months.

Which of the following would strengthen the chairman's

decision?

I. Material management and other resources will have to

be in fine tune to maintain production schedule.

ll. Company should also make plans products other than

car.

III. The network of dealers must be finalized and all

legal, financial and other matters in this regard should

be finalized at the earliest.

(a) Only l (b) Only II (c) Only I and Ill

(d) All the three (e) None of these

15. Statement :

Over the last twenty years, the rate of increase in total

population of City ‘X’ has been second to none in the

world. Hence the economic growth is more modest per

capita of total population is calculated. Over the last ten

years, the progress has been very slow.

According to the passage, which of following must be

true?

(a) For the past few years, the birth rate declined.

(b) City ‘X’ has a very large population.

(c) The per capita production rate has not declined.

(d)Productivity per capita has not grown as fast during

the past ten years.

(e) None of these

Directions for questions 16 to 18: Study the following

information to answer the given questions.

CLAtutor Logical Reasoning 82

Science is a sort of news agency comparable in principle

to other news agencies. But this news agency gives us

information which is reliable to an extraordinarily high

degree due to elaborate to an spread over centuries. So,

science should be read with the same interest with

which we read news.

16. Which of the following will weaken the above

argument’?

(a) Man is an intelligent creature.

(b) Science gives information.

(c) Scientific information is revised.

(d) News agencies cannot verify news.

(e) None of these

17. Which of the following may be regarded as an

assumption in the above passage?

(a) Verification of news is necessary.

(b) Science encourages investigative spirit.

(c) Science is objective in approach.

(d) Science gives us news and not any other information

regarding national phenomenon.

(e) None of these

18 Which of the following strengthens the argument?

(a) Agricultural research is scientific.

(b) Science gives abstract theories.

(c) Verified information is reliable.

(d) Science is a compulsory subject.

(e) None of these.

19. Statement: The management of School ‘X’ has

decided to give free breakfast from next academic year

to all the student in its primary section through its

canteen, even though they will not get any govt grant.

Which of the following may weaken the step taken by

the management?

I. Funds will have to be raised to support the scheme for

the years to come.

II. The school will have to admit many poor students at

negligible fee for the next academic year.

III. The canteen facilities and utensils will have to be

checked and new purchases to be made up to equip it

properly.

(a)I and III (b)II or III (c) I and II

(d) Only II (e) None of these

20. The Central Govt has directed the state govt to

reduce govt expenditure in view of serious resource

crunch and it may not be able to sanction any additional

grant to the states for the next six months.

Which of the following may be an assumption/s of the

Central Govt?

l. The state govt will abide by the directive.

II. The state govt is totally dependent on Central Govt for its expenditure. III. The Central Govt has reviewed the expenditure account of the state govt. IV. The state govt may appeal against such directions. (a)I and III (b)II or IV (c) I and II

(d) Only II (e) None of these

21. Statement: The average family income in the metros has increased today as compared to earlier in most families as these days husbands and wives both are working. Which of the following inferences cannot be drawn from the above statements? (a) It was less prevalent for the spouse to work earlier than today. (b) Wives earn more than their husbands today. (c)The family income level was lower a few years ago (d) More husbands and wives are working today as compared to last few years (e) None of these

22. Any further increase in the pollution level in the city

by way of industrial effluents and automobile

exhaustions would pose a severe threat to the

inhabitants.

Which of the following would be an appropriate move

to control the pollution level in the city.

(a) All the factories in the city should immediately be

closed down

(b) The govt should restrict the issue of fresh licenses to

factories and automobiles

(c) The automobiles should not be allowed to ply on the

road for more than four hours a day

(d) Factories must not be allowed to work for more than

15 days in a month

(e) None of these

Directions for questions 23 to 25: Answer the following

questions based on the statement given below.

Statement: If a person reads a lot, he will increase his

vocabulary.

CLAtutor Logical Reasoning 83

23. The given statement is based on the assumption that.

(a) People should read books containing unfamiliar

words.

(b) The books to be read should be entertaining.

(c) Strong vocabulary is important for writing and

speaking.

(d) Reading a lot is the only way to increase vocabulary.

(e) None of these

24. Which of the following would weaken the given

statement?

(a) Only a well read person can make proper use of

English language.

(b) The style of a person is inferred by the style of the

author he reads.

(c) Those who passed vocabulary tests could pass any

test.

(d) Educated people can express their thoughts better.

(e) None of these

25. Which of the following will strengthen the given

statement?

(a) Everyone must study in an English-medium school.

(b) Slow readers cannot do well in competitive

examinations.

(c) In order to become a good speaker in English, one

must increase one’s vocabulary.

(d) It is not a big deal to increase one’s vocabulary.

(e) None of these

CLAtutor Logical Reasoning 84

Non-Verbal

Directions(1-5): In each of the following questions, find out which of the figures (a), (b), (c) and (d) can be formed from the pieces given in (x) 1.

2.

3.

4.

5.

Directions (6-10): In each of the following questions, complete the missing portion of the given pattern by selecting from the given alternatives (a), (b), (c) and (d) 6.

7.

8.

9.

10.

Directions (11-25): In each of the following questions given below which one of five answer figures should come after the problem figures if the sequence were continues. 11.

12.

CLAtutor Logical Reasoning 85

13.

14.

15.

16.

17.

18.

19.

20.

21.

22.

CLAtutor Logical Reasoning 86

23.

24.

24.

25.

CLAtutor Logical Reasoning 87